National Board Practice Exam – Flashcards

Unlock all answers in this set

Unlock answers
question
A 42-year-old woman taking no medications and with no significant medical history is scheduled for comprehensive dental hygiene care. Initially she has no complaints about her health, and her vital signs are within normal limits. After you explain the procedure to be performed and initiate the dental hygiene intervention, she begins to complain of "feeling faint." What should be the initial emergency management for this patient?
answer
Reclining the dental chair to the Trendelenburg position RATIONALE: Reclining the chair to the Trendelenburg position will often abort a near-syncopal episode and should be the initial emergency management for this patient. Vital signs should be retaken once the patient is reclined in the Trendelenburg position. Providing a sugar-containing drink is not indicated unless there is reason to believe that the patient is hypoglycemic. Office staff may be alerted, but the chair should be reclined first.
question
In judicial law, the party bringing a lawsuit is referred to as the
answer
plaintiff RATIONALE: In judicial law, determined by state, provincial, and federal courts, the party bringing a lawsuit is known as the plaintiff. The defendant is the party against whom the lawsuit is filed. The attorney is the individual serving as an advocate for either the plaintiff or the defendant. An expert witness is an individual retained by either the plaintiff or the defendant to explain specialized information to jurors.
question
Each of the following is a characteristic of acidulated phosphate fluoride EXCEPT one. Which one is the EXCEPTION?
answer
Discolors tooth structure RATIONALE: Acidulated phosphate fluoride (APF) does NOT discolor tooth structure. Stannous fluoride (SnF2) is the formulation that causes staining of tooth structure. Although the low pH (3.5) of acidulated phosphate fluoride (APF) enhances fluoride uptake, it may cause etching of dental sealants, porcelain, and some composite restorations. Acidulated phosphate fluoride has a high level of patient acceptance and is the most commonly used in-office fluoride treatment. The enhanced fluoride uptake of acidulated phosphate fluoride (ACP) is a result of its low pH (3.5).
question
A periodontal examination conducted on a 48-year-old man reveals probing depths of 5 to 6 mm in proximal surfaces of all posterior teeth, normal gingival contour, radiographic evidence of bone loss, and generalized gingival inflammation. What disease classification is present?
answer
Moderate chronic periodontitis RATIONALE: With normal gingival contour, average probing depth in moderate chronic periodontitis is 5 to 6 mm, with 4- to 5-mm attachment loss and radiographic evidence of bone loss. Attachment loss and bone loss do not occur in gingivitis, whether it is acute or chronic. With normal gingival contour, average probing depth in early chronic periodontitis is 3 to 4 mm. With normal gingival contour, average probing depth in advanced chronic periodontitis is 7 mm or greater. Periodontal pockets do not occur in necrotizing ulcerative periodontitis, because soft tissue necrosis is so rapid that it coincides or precedes bone loss, sometimes exposing bone.
question
Dental practice acts are designed to perform each of the following functions EXCEPT one. Which one is the EXCEPTION?
answer
Amend relevant laws RATIONALE: Amending any law, relevant to oral care or not, is not a function of a dental practice act. Laws are written, amended, and enacted by the legislative branch of government. Dental practice acts are designed to protect the public by requiring would-be providers to demonstrate minimum proficiency before obtaining a license. Adoption of rules and regulations for dentistry, dental hygiene, and dental assisting, as well as for dental laboratory technologists and denturists, is a function of dental practice acts. A function of dental practice acts is to regulate applications for licensure by implementing mechanisms for measuring the competence of applicants.
question
A 69-year-old man is a new patient to your practice. He states that he has a heart condition but has never had a heart attack. He periodically experiences swelling of the lower legs and is taking a medication called digoxin (Lanoxin). He is also taking a "water pill." His vital signs are normal, but you notice some edema around his ankles. The MOST likely name for this patient's "heart condition" is
answer
congestive heart failure. RATIONALE: Swelling of the extremities, especially the lower legs, is a classic sign of congestive heart failure (CHF). Digoxin (Lanoxin) is often used to treat CHF. Shortness of breath and cough are other signs and symptoms of CHF. Cerebrovascular accident (CVA), or stroke, is not a heart condition. The patient's symptoms are not consistent with stroke. This patient exhibits no symptoms of angina, which include chest pain or discomfort and pain in the arms, neck, jaw, shoulder, or back accompanying chest pain. Also absent are symptoms of myocardial infarction (heart attack). Chest pressure is the most common symptom of myocardial infarction, although the diverse symptoms include a sensation of fullness or squeezing, jaw pain, toothache, headache, shortness of breath, nausea, and vomiting.
question
Radiographic diagnosis contributes significantly to the final diagnosis of all the following EXCEPT one. Which one is the EXCEPTION?
answer
Cementoma RATIONALE: Although cementomas are detected radiographically, the most reliable factors for diagnosis are the age, gender, and race of the patient, plus the fact that the associated teeth are vital. Internal resorption can only be seen radiographically. Odontoma is evaluated radiographically. Unerupted supernumerary teeth are diagnosed from a radiograph. Impacted third molars are seen radiographically.
question
Chronic liver disease may follow infection with each of the following viruses EXCEPT one. Which one is the EXCEPTION?
answer
Hepatitis A RATIONALE: Chronic liver disease can result from infection with hepatitis B, C, or D. Note that hepatitis literally means inflammation of the liver (hepa = liver; itis = inflammation). In the absence of other complicating diseases, hepatitis A alone does not cause chronic liver disease. Hepatitis A is the least serious of the numerous strains of hepatitis but can last several weeks to 6 months. It does not have a chronic form.
question
The Health Insurance Portability and Accountability Act pertains to the universal principle of autonomy because HIPAA was designed to protect patients' confidential information.
answer
The statement is NOT correct, but the reason is correct RATIONALE: The Health Insurance Portability and Accountability Act (HIPAA) pertains to the universal principle of confidentiality and was specifically designed to protect all patient-related confidential data and information.
question
While working in private dental practice you volunteered extensively at a small state-funded dental clinic in a rural county whose primary employer is a large tobacco-growing operation. Community water sources are fluoridated, but a significant portion of the population relies on well water. Recently you were hired by a nonprofit organization through a state-funded grant to develop tobacco-cessation strategies and provide dental hygiene care for underserved local adults. Before you initiate services, the grant mandates that you assess the oral health status and needs of the population. An epidemiologist is available for you to consult, as is an environmental health specialist. Disparities in the early diagnosis and treatment of oral and pharyngeal cancer occur equally among racial and ethnic groups because health professionals have successfully educated adults about the signs, symptoms, and risk factors.
answer
NEITHER the statement NOR the reason is correct. RATIONALE: Recent National Health Interview Survey (NHIS) results indicate racial and ethnic disparities related to early detection and treatment of oral pharyngeal cancers. Consideration needs to be given to income levels, education, availability of proper health care, and use of tobacco and alcohol by different population groups.
question
You have been asked to evaluate the federally funded dental program at a local senior citizen center in a northeastern state. Located in a diverse inner-city neighborhood of a large metropolitan city, the center serves an ethnically diverse population of varying socioeconomic status. Community water supplies are fluoridated, and students from local dental hygiene programs occasionally provide oral health education and oral screenings under the supervision of a licensed dentist. Both a registered nurse and a dietitian are available for you to consult with, as are a dentist and a gerontologist. Breakfast and lunch are served five days a week, and movies, complete with soft drinks and snacks, are available most afternoons. The ideal treatment for inactive or arrested root surface caries is the placement of traditional permanent esthetic restorations because the more conservative approach that involves self-care and fluoride is ineffective on root surfaces.
answer
NEITHER the statement NOR the reason is correct. RATIONALE: Because the ideal restorative material for the root surfaces has not been found, oral health professionals may better serve patients with inactive or arrested root surface caries by advocating meticulous self-care and fluoride therapy. This avoids the removal of tooth structure in close proximity to the dental pulp. Although the most effective concentration and frequency of fluoride use for root caries remain undetermined, fluoride has beneficial effects on roots. Fluoride does have a beneficial effect on root surfaces.
question
Which of the following blood cells function as phagocytes?
answer
Neutrophils RATIONALE: Neutrophils, the most abundant type of white blood cells, are highly phagocytic and very active in the initial stages of infection. Note that phagocytes, essentially scavengers, are involved in all stages of infection by ingesting harmful foreign particles, microbes, and dead or dying cells. The remaining selections—eosinophils, basophils, lymphocytes, and erythrocytes—do not function as phagocytes. Eosinophils and basophils are both a specialized type of white blood cell called a leukocyte. Erythrocytes (distinguished by the prefix "erythro") are a type of red blood cell that carries oxygen and contains hemoglobin.
question
Which of the following blood cells is an agranular leukocyte?
answer
Lymphocyte RATIONALE: A lymphocyte is a specialized type of white blood cell called an agranular leukocyte. (An agranular lymphocyte, also known as a mononuclear leukocyte, is defined as a white blood cell with a one-lobed nucleus.) Note that the letter "a" before a description typically means "not." Referring to the remaining selections: Although a basophil is also a leukocyte (white blood cell), it is considered granular (as opposed to agranular). Platelets, erythrocytes, and thrombocytes are not leukocytes; they have neither agranular nor granular characteristics. A platelet is a cell fragment that circulates in the blood and is involved in hemostasis by forming blood clots. Again, the prefix has significance; erythro- (red) means that an erythrocyte is a red blood cell, whereas a thrombocyte is a specialized type of platelet.
question
While working in private dental practice you volunteered extensively at a small state-funded dental clinic in a rural county whose primary employer is a large tobacco-growing operation. Community water sources are fluoridated, but a significant portion of the population relies on well water. Recently you were hired by a nonprofit organization through a state-funded grant to develop tobacco-cessation strategies and provide dental hygiene care for underserved local adults. Before you initiate services, the grant mandates that you assess the oral health status and needs of the population. An epidemiologist is available for you to consult, as is an environmental health specialist. Working with the environmental health specialist, you learn that 27.2% of adults in this county use tobacco products, compared with 18.2% nationwide. While studying the incidences of oral and pharyngeal cancer, you discover that data to measure the number of deaths due to cancer of the oral cavity or pharynx can be obtained from each of the following sources EXCEPT one. Which one is the EXCEPTION?
answer
Surveys frequently mailed to private dental offices by the American Cancer Society RATIONALE: Surveys mailed to private dental offices, regardless of the entity conducting the research, are limited to a select subset of the general population and do not reflect dentally underserved populations. The National Vital Statistics System of the National Center for Health Statistics (NCHS) within the Centers for Disease Control (CDC) provides data regarding deaths due to cancer of the oral cavity or pharynx. The National Cancer Institute (NCI) within the National Institute of Health (NIH) tracks the proportion of oral and pharyngeal cancer lesions diagnosed at the earliest stages. Self-reported studies were used to track the incidences of oral and pharyngeal cancer by The National Health Interview Survey (NHIS), a component of Healthy People 2010.
question
Liquid polishes provide a long-term finish to composite restorations because their filler component contains zinc oxide-eugenol.
answer
NEITHER the statement NOR the reason is correct. RATIONALE: Liquid polishes provide only short-term smooth surface coating to composite restorations. By contrast, microfill and anofill composites develop the smoothest finishes because they employ small filler particles. Zinc oxide-eugonol is not a component of composite restoration material but is often added as a sedative component to cement liners.
question
Determination of who should receive treatment when all cannot be treated is known as
answer
distributive justice. RATIONALE: Distributive justice, also known as allocation of scare resources, is defined as the determination of who should receive treatment when all cannot be treated. Closely related and integral to distributive justice is equity. This is the concept that all persons receive equal treatment. A factor considered in all distributive justice determinations is the prioritization of the needs of the individual, legally referred to as need. Public policy differs from distributive justice and is defined as a system of plans of actions, regulatory measure, laws, and funding priorities.
question
Each of the following is classified as a complex carbohydrate EXCEPT one. Which one is the EXCEPTION?
answer
Monosaccharides RATIONALE: Monosaccharides (trioses, pentoses, hexoses) are classified as simple carbohydrates. Mucopolysaccharides (protein and carbohydrate in a loose binding) are classified as complex carbohydrates. Glycolipids (carbohydrate and lipid, as in gangliosides) are classified as complex carbohydrates. Glycoproteins (carbohydrate and protein in a specific functional arrangement, such as blood group substances and many hormones) are classified as complex carbohydrates.
question
The marginal gingiva of a patient with malignant neutropenia are likely to demonstrate generalized enlargement. The oral tissues of a patient with acute leukemia are likely to exhibit necrosis and ulcerated marginal gingiva.
answer
Both statements are false. RATIONALE: Patients with malignant neutropenia are likely to have necrotic oral tissues and ulcerated marginal gingiva. Oral symptoms of acute leukemia include generalized gingival enlargement, swelling, and bleeding.
question
The silver halide crystals in the emulsion of radiographic film determine the speed of the film. Larger silver halide crystals in the emulsion of dental radiographic film result in slower film but a more detailed, high contrast image.
answer
The first statement is true, the second is false. RATIONALE: The speed of dental radiographic film is determined by the size of the silver halide crystals in the emulsion. Larger crystals result in a faster film. However, this faster film produces a grainy image with less detail and sharpness. To solve this dilemma, manufacturers have altered the shape of the silver halide crystals to produce images with higher contrast and increased sharpness.
question
A woman in her last trimester of pregnancy becomes unresponsive during dental hygiene care. No spontaneous respirations are detected, and attempts to ventilate her are unsuccessful. The following question pertains to this scenario. The appropriate emergency treatment for a pregnant woman is to perform sharp back blows, because abdominal thrusts are not used in late pregnancy.
answer
The statement is NOT correct, but the reason is correct. RATIONALE: The appropriate emergency treatment is to kneel beside the patient and perform chest thrusts at the base of the sternum, also called the breastbone. Sharp back blows are not indicated. The reason correctly states that abdominal thrusts are not used in late pregnancy. Note that administration of oxygen is not advised, because this will be of no value if the airway is obstructed.
question
The material used to record the occlusal pattern of the opposing dentition in a dental impression is called
answer
bite registration wax. RATIONALE: To record the occlusion (form a registration pattern) of the opposing dentition, bite registration wax is used. Utility wax, as its name implies, has many uses. Utility wax is soft and commonly used to modify such appliances as alginate impression trays. Baseplate wax is used to establish the vertical dimension, plane of occlusion, and initial arch form of a complete denture. Corrective impression wax is used to form a registration pattern of soft tissues on an impression.
question
As a requirement for your graduate degree in dental hygiene, you are conducting statistical analysis of data from a community oral screening program in a local homeless shelter. Socioeconomic status is low and the incidence of homelessness is high in this community since a factory closed recently and layoffs were made in related industries. Homelessness presents many challenges in obtaining accurate research data, partly because of transportation difficulties experienced by patients and partly by the seasonal and transient nature of this population as individuals seek employment. The shelter provides a limited supply of toothpaste, toothbrushes, and dental floss. A related aspect of this assignment includes presenting a brief introduction of statistical concepts to dental hygiene students at a nearby community college. An asymmetrical distribution of statistical data is known as
answer
a skewed distribution RATIONALE: A skewed distribution is one in which the distribution is asymmetrical. An ordinal scale is a scale of measurement in which a class or category is ranked in an empirical order such as "strongly disagree, disagree, agree, strongly agree." A bell (normal) curve represents a symmetrical distribution of scores in which the mean, median, and mode have the same value and the scores fall within a standard relationship to the mean. A scattergram is used to graphically represent the relationship between two variables.
question
Michael, a 12-year-old Asian adolescent, is scheduled for his first dental hygiene care appointment and dental examination in 5 years. His mother reports that he is generally in good health but that he takes Adderall XR 30 mg/day for attention deficit hyperactivity disorder and still exhibits clumsiness, irritability, and an inability to focus for even short periods. He is not doing well in school, and the teachers have recommended that he be held back a year. His mother also reports that the "goose egg" on his forehead was sustained while he was roughhousing under the kitchen table. She says that he is always getting bumps and bruises because of his inability to control his behavior. All of the following statements about attention deficit hyperactivity disorder (ADHD) are true EXCEPT one. Which one is the EXCEPTION?
answer
Children with ADHD have a higher incidence of intellectual and developmental disabilities (IDDs). RATIONALE: In children with a higher incidence of intellectual and developmental disabilities, the diagnosis would NOT be attention deficit-hyperactivity disorder (ADHD). Instead, the diagnosis would be intellectual and developmental disability (IDD). The incidence of ADHD is higher in males, although this is partially because of more aggressive behavior. ADHD can affect any CNS function. ADHD most often causes problems with reading, writing, and math. Because it is labeled an attention deficit disorder, concentration and memory are problems.
question
Anthrax is categorized as a zoonotic infection because it primarily occurs in nonhumans and is not transferred to humans.
answer
The statement is correct, but the reason is NOT. RATIONALE: Anthrax is categorized as a zoonotic infection. Note that a zoonotic infection, also called zoonosis, is a category of disease that occurs primarily in nonhumans but can be transmitted to humans. Examples of other zoonotic infections include tularemia, brucellosis, and toxoplasmosis. By contrast, diphtheria, primarily a disease of humans, is not a zoonotic infection.
question
A 6-year-old girl arrives for her 6-month dental hygiene care appointment with just one complaint. She feels "itchy all over." It started about 10 minutes ago, when she was outside playing in the grass. Initially, her vital signs are normal except for slight tachycardia. She has some skin lesions that appear to be urticaria, and her lower lip is beginning to swell. Within a minute she complains of difficulty breathing and her respiratory rate is increased. The normal respiratory rate for this child is
answer
approximately 20 respirations/minute. RATIONALE: The normal respiratory rate for a child is approximately 20 respirations/ minute. Ten to 14 respirations/minute is incorrect. Approximately 16 respirations/minute is incorrect. Sixteen to 20 respirations per minute is incorrect.
question
The precentral gyrus is located in the frontal lobe and is involved with sensory perception. The precentral gyrus is responsible for executing involuntary movements.
answer
Both statements are false. RATIONALE: Although the precentral gyrus is located in the frontal lobe of the brain, it is not involved with sensory perception. The precentral gyrus is involved in voluntary muscle control.
question
Fluoride application before sealant placement is advised because the mechanical bonding necessary for retention is enhanced.
answer
NEITHER the statement NOR the reason is correct. RATIONALE: Fluoride application is contraindicated before sealant placement because the bonding necessary for retention is disturbed.
question
An informed consent form should contain the appropriate technical language because without technical language the form does not constitute a legal document.
answer
NEITHER the statement NOR the reason is correct. RATIONALE: An informed consent form should not contain technical language. Technical language may prevent a patient from understanding aspects of a procedure or therapy. A patient needs to sign a consent form indicating that he or she fully understands the information.
question
Manifestations of bulimia include all of the following EXCEPT one. Which one is the EXCEPTION?
answer
Consumption of 300 to 600 calories per day RATIONALE: Intake of calories in bulimia ranges from 3,500 to 20,000. Recurrent bingeing followed by vomiting is a major characteristic of bulimia. Dehydration results from frequent vomiting. Vomiting is induced to reduce caloric intake from the bingeing. Depression is an integral component to this psycho-psychologic condition.
question
Dental enamel is considered more radioresistant than blood cells because enamel has a lower metabolic rate and a slower proliferation rate than blood cells.
answer
Both the statement and reason are correct and related. RATIONALE: Enamel is considered radioresistant because both its metabolic and proliferative rates are relatively low. By contrast, tissues with higher metabolic and proliferative rates, such as blood cells, are considered radiosensitive. Note that the lens of the eye and the thyroid gland are also considered radiosensitive. Blood cells are more radiosensitive than either the lens of the eye or the thyroid gland.
question
Clubbing of the fingers is MOST often seen in
answer
congenital heart disease. RATIONALE: Clubbing of the fingers coupled with cyanosis is a cardinal sign of congenital heart disease. Although stroke victims are deprived of oxygen and are therefore cyanotic, they do not have clubbing of the fingers. Hypertensive disease, also associated with oxygen deprivation (cyanosis), causes increased blood pressure but does not cause clubbing of the fingers. Acquired immune deficiency syndrome (AIDS), a spectrum that kills or damages the body's immune system, is yet another disease that does not cause clubbing of the fingers. Cerebral palsy, also not associated with clubbing of the fingers, is a neuromuscular disorder that results from brain damage.
question
Which of the following findings would necessitate retreatment with active periodontal therapy if discovered on a reevaluation during a periodontal maintenance care visit?
answer
Localized attachment loss of 2 mm or more RATIONALE: Because localized attachment loss of 2 mm or more indicates advancing disease, active therapy should be resumed. Generalized bacterial plaque biofilm requires oral self-care education. Gingival inflammation indicates the necessity of an evaluation to determine etiology. If related to local factors, such as plaque biofilm and calculus, periodontal maintenance procedures should be employed. If inflammation is generalized and without local etiology, a referral for evaluation of systemic disease is warranted. Gingival recession, with its resultant tissue shrinkage and/or repositioning, is common after periodontal therapy. Progressive recession should be addressed on the basis of etiology. Gingival bleeding, a sign of inflammation, indicates the need for an evaluation to determine etiology.
question
As a requirement for your graduate degree in dental hygiene, you are conducting statistical analysis of data from a community oral screening program in a local homeless shelter. Socioeconomic status is low and the incidence of homelessness is high in this community since a factory closed recently and layoffs were made in related industries. Homelessness presents many challenges in obtaining accurate research data, partly because of transportation difficulties experienced by patients and partly by the seasonal and transient nature of this population as individuals seek employment. The shelter provides a limited supply of toothpaste, toothbrushes, and dental floss. A related aspect of this assignment includes presenting a brief introduction of statistical concepts to dental hygiene students at a nearby community college. Data from the oral screening program indicate that the target population has such a low incidence of gingivitis that you suspect incorrect methodology. Each of the following is true of concepts within the experimental design to ensure validity EXCEPT one. Which one is the EXCEPTION?
answer
The independent variable is the outcome variable of interest. RATIONALE: The outcome variable of interest is called the dependent variable. The dependent variable is believed to be caused or influenced by the independent variable. Stratified sampling, a method of randomly selecting subjects from two or more strata within the population of study, increases the likelihood that the study will be focused on randomly selected subjects. Inclusion of every "nth" person in a randomly ordered list, known as systematic sampling, increases the likelihood of random selection of subjects.
question
The finger that serves as a "support beam" during calculus removal is the
answer
ring finger RATIONALE: The ring finger of the dominant hand, also called the fulcrum finger, stabilizes the clinician's hand during calculus removal and is often described as a "support beam." Together with the thumb, the role of the index finger is to apply lateral pressure. The role of the middle finger is to detect root surface irregularities during periodontal instrumentation; this finger should always rest on the shank of the curet or explorer. Together with the index finger, the role of the thumb is to apply lateral pressure.
question
Penicillin would be LEAST effective when used against
answer
Mycoplasma RATIONALE: Of the listed pathogens, penicillin is LEAST effective against Mycoplasma. This is because the target of penicillin's action is the cell wall, and Mycoplasma does not have a cell wall. Penicillin is the drug of choice (DOC) for infections caused by Actinomyces, Bacillus, and Treponema. Note that Actinomyces is a gram-positive organism that typically lives in the nose and throat but often causes oral infections. Bacillus is an extremely diverse genus of gram-positive spore-forming rods found in the oral cavity. Treponema are gram-negative and spiral (spirochetes). Treponema pallidum causes syphilis.
question
Osteoarthritis primarily affects weight-bearing joints in adults. Small joints are more susceptible to osteoarthritis than larger joints, which benefit from greater resistance to damage.
answer
The first statement is true, the second is false. RATIONALE: Weight-bearing joints such as hips and knees in adults are primarily affected by osteoarthritis. Note that osteoarthritis does not affect children. Osteoarthritis primarily affects large joints, not small ones.
question
Frictional keratosis should be biopsied and excised as soon as it is discovered, because untreated frictional keratosis has the potential for malignant transformation.
answer
NEITHER the statement NOR the reason is correct. RATIONALE: Frictional keratosis, the oral counterpart of a callus on the skin, is a common tissue anomaly that forms as a result of mild mechanical trauma or irritation from malposed teeth, dental prosthetics, or patient habit. Linea alba, found on the buccal mucosa and the product of chronic cheek biting, is a common form of frictional keratosis. No treatment is required for frictional keratosis. Although the potential for malignant transformation is considered minimal, patients should be advised to discontinue any contributory behavior that is under their control.
question
When CPR is performed on an adult, the appropriate depth of chest compressions is 1.5 to 2 inches because deeper compressions may cause damage to surrounding tissues and organs.
answer
NEITHER the statement NOR the reason is correct. RATIONALE: In 2010 the American Heart Association Guidelines for depth of chest compressions was changed to at least 2 inches in adults. Note that for infants and children, compression depth recommendation is now at least one third of the anterior-posterior diameter of the chest. For infants, this is approximately 1.5 inches. For children, this depth is approximately 2 inches. Compression depth has been changed to increase blood flow primarily by increasing intrathoracic pressure and directly compressing the heart.
question
Your next patient, Mrs. Smith, an African-American woman who is 50 years old, has been waiting for a kidney transplant for 9 months. Three times a week, she must undergo kidney dialysis as a lifesaving therapy. She takes nifedipine (Procardia) for hypertension and Wellbutrin (bupropion) for depression. Her chief complaint is "My gums look big, and they keep bleeding." All of the following oral manifestations are associated with end-stage renal disease or its treatment EXCEPT one. Which one is the EXCEPTION?
answer
Rampant dental caries RATIONALE: Rampant dental caries is not caused by renal insufficiency. However, a high prevalence of moderate to severe periodontitis is documented in patients with chronic kidney disease, especially those in the dialysis population. Mucosal anemia (also called anemic mucosa) is a reflection of general anemia. Herpes is an opportunistic infection resulting from immunosuppression due to her condition. Calculus deposits tend to be increased in end-stage renal disease. The alveolar bone takes on an appearance of ground glass because calcium leaches out of the alveolar bone.
question
Although tissues have the capacity to repair some degree of damage from ionizing radiation, accumulated unrepaired damage is called the cumulative effect. The somatic effect of ionizing radiation refers to exposure resulting in injury to future generations.
answer
The first statement is true, the second is false. RATIONALE: Over time, ionizing radiation accumulates in the cells, tissues, and organs. Accumulated damage that the body cannot repair is called the cumulative effect. The somatic effect refers to injury occurring in the person exposed to the ionizing radiation that does not involve reproductive tissue. By contrast, the genetic effect refers to exposure of reproductive tissue that results in injury to future generations.
question
The doctrine of law whereby the court bases its decisions on previous case law is known as
answer
stare decisis. RATIONALE: In Latin, stare decisis means "Let the decision stand." Stare decisis applies to the doctrine of law whereby the court bases its decisions on previous case law (a prior case with similar facts). Landmark decisions depart from precedent; on the basis of the same facts, a "landmark," or different conclusion, is made. Criminal law, a classification of law that includes both misdemeanors and felonies, relates to acts considered offensive to society as a whole. Civil law, the civil code system developed by legislative bodies and enforced by the court system, is based on rules and regulations that protect the legal rights of private persons.
question
Before a culture for suspected angular cheilitis is scheduled, application of an antifungal ointment to give the area a chance to resolve illustrates the clinical diagnosis, because a key feature of clinical diagnosis is based solely on the clinical picture.
answer
The statement is NOT correct, but the reason is correct. RATIONALE: Applying an antifungal ointment before scheduling a culture from an area of suspected angular cheilitis is an example of therapeutic diagnosis. This conservative approach involves use of a therapeutic medication to see whether the suspected condition resolves. Although it is true that a key feature of the clinical diagnosis is based solely on the clinical picture, this was not the method used.
question
While probing a patient with moderate periodontitis, you notice that pus flows from the gingival margin in response to pressure. This substance is referred to as
answer
purulent exudate. RATIONALE: Purulent exudate (pus), sometimes observed during routine probing, may be "milked" with a coronal finger stroke over the tissues. Purulent exudate, often referred to as suppuration, is a sign of secondary infection. It consists of living, degenerated, and necrotic leukocytes, as well as living and dead bacteria, serum, and fibrin. Gingival crevicular fluid, an exudate present in clinically normal sulci, is typically detected minimally or not at all. Erythema, defined as redness of the skin or mucous membranes, is caused by hyperemia of surface capillaries. The term erythema is used to describe redness within gingival, periodontal, and oral tissues. Percussion is a method of tapping a tooth with a solid object, such as the handle of an intraoral mirror, to assess tooth sensitivity.
question
Radiation damage from the radiolysis of water to hydrogen peroxide within DNA macromolecules is referred to as
answer
indirect. RATIONALE: Damage to DNA macromolecules that is caused by radiolysis of intracellular water (H2O) to hydrogen peroxide (H2O2) is called indirect damage. Radiolysis is defined as the dissociation of molecules by radiation. The term indirect damage refers to the damage to DNA caused by a free radical. A direct effect of radiolysis involves the actual transfer of ionizing energy directly from the x-ray to the DNA. Contrast this with the indirect effect, which occurs during the dissociative process of radiolysis. Collumnation is the mechanism within a PID that restricts the cross-sectional area of the X-ray beam. Free radicals are involved in radiolysis; indeed, free radical damage to DNA occurs as a result of exposure to radiation, including ionizing and UV radiation.
question
Regenerative techniques in periodontal surgery are recommended for
answer
two- and three-walled vertical defects. RATIONALE: Regenerative techniques in periodontal surgery are most effective in two- and three-walled vertical defects in the alveolar bone. One-walled defects are not indicated for regenerative techniques in periodontal surgery, however periodontal flap surgery is often performed. Gingival hyperplasia, if severe enough to warrant surgery, is treated with gingivectomy. The etiology of gingival recession indicates that it is not an indication for regenerative surgical techniques.
question
What term is used to describe a lesion that is stem-like?
answer
Pedunculated RATIONALE: "Pedunculated" describes a lesion that is stem-like or on a "pedestal." The term "sessile" describes a lesion that is flat at the base. A papule is a small elevated lesion that is above the surface of the normal surrounding tissue. A bulla is an elevated lesion that looks like a blister. "Lobulated" describes a lesion whose segments are fused to make one combined lesion.
question
Michael, a 12-year-old Asian adolescent, is scheduled for his first dental hygiene care appointment and dental examination in 5 years. His mother reports that he is generally is good health but takes Adderall XR 30 mg/day for attention deficit hyperactivity disorder and still exhibits clumsiness, irritability, and an inability to focus for even short periods. He is not doing well in school, and the teachers have recommended that he be held back a year. His mother also reports that the "goose egg" on his forehead was sustained while he was roughhousing under the kitchen table. She says that he is always getting bumps and bruises because of his inability to control his behavior. Questions 55 through 60 refer to this situation. If you were planning Michael's first visit to the dental treatment area, all of the following actions would be appropriate EXCEPT one. Which one is the EXCEPTION?
answer
Asking him to sit perfectly still for 30 minutes and then giving him a break RATIONALE: A child with ADHD cannot physically remain motionless for 30 minutes. Placing the instruments out of Michael's reach will ensure his safety and the integrity of your infection control procedures. Gradually introducing this patient to the dental treatment area will reduce his fear and curiosity. Desensitization is important to minimize oral and startle reflexes. Providing short breaks every few minutes is a wise strategy that will impose some structure but also accommodate his hyperactivity disorder.
question
A community health clinic located in an underserved rural community has hired a dental hygienist to design and implement a program to reduce early childhood caries (ECC). Most of the population does not benefit from community fluoridated water, nor are school-based fluoridation programs available. During initial research, the dental hygienist learns that a strong coalition within the community opposes any type of fluoridation. To prioritize needs, goals and objectives the program administrator has recommended use of the SMART formula. The clinic, funded by a state agency, hopes to expand oral services to older children and adults in the next few years. While researching early childhood caries (ECC), the dental hygienist wants assurance that the studies she consults are valid. Validity measures
answer
what is intended to be measured. RATIONALE: With regard to a study, validity is considered to be the degree to which a study measures what it intends to measure. In research design, validity refers to whether a study is able to scientifically answer the questions it intends to answer. Measuring data consistently at different times is called reliability. Reliability is also called reproducibility and stability of measurement. Conditions that can be reversed are measured with a reversible index; this tool is designed to assess a disease or condition that can be reversed (e.g., gingivitis). An irreversible index, designed to assess a disease or condition, measures cumulative conditions that cannot be reversed (e.g., dental caries, fluorosis).
question
An area of angular cheilitis that heals after treatment with fluconazole and terconazole is bacterial in origin, because both fluconazole and terconazole are antibacterial preparations.
answer
NEITHER the statement NOR the reason is correct. RATIONALE: Angular cheilitis that responds to antifungal preparations is caused by a fungal infection. Both fluconazole, also called diflucan, and terconazole are antifungal medications used to treat infections throughout the body, including the oral cavity. They are commonly dispensed as creams or ointments. A bacterial infection will not respond to antifungal treatment.
question
While working in private dental practice you volunteered extensively at a small state-funded dental clinic in a rural county whose primary employer is a large tobacco-growing operation. Community water sources are fluoridated, but a significant portion of the population relies on well water. Recently you were hired by a nonprofit organization through a state-funded grant to develop tobacco-cessation strategies and provide dental hygiene care for underserved local adults. Before you initiate services, the grant mandates that you assess the oral health status and needs of the population. An epidemiologist is available for you to consult, as is an environmental health specialist. The measure of central tendency used to describe the score or value that occurs most frequently in a distribution is the
answer
Mode RATIONALE: The mode is the measure of central tendency used to describe the score or value that occurs most frequently in a distribution of scores. Note that the mode is most useful for categorical data, when it is useful to determine the most common category of an event or habit. The mean, determined by dividing the sum of all values by the number of items, represents the arithmetic average of the values within the distribution. The median represents the exact middle score or value when the distribution is placed in an ordered sequence. Correlation is not a measure of central tendency; it is a statistical method for determining variation between variables.
question
You have been asked to evaluate the federally funded dental program at a local senior citizen center in a northeastern state. Located in a diverse inner-city neighborhood of a large metropolitan city, the center serves an ethnically diverse population of varying socioeconomic status. Community water supplies are fluoridated, and students from local dental hygiene programs occasionally provide oral health education and oral screenings under the supervision of a licensed dentist. Both a registered nurse and a dietitian are available for you to consult with, as are a dentist and a gerontologist. Breakfast and lunch are served five days a week, and movies, complete with soft drinks and snacks, are available most afternoons. You wish to describe variability of plaque scores taken from members of your target population during a screening session, using measures of dispersion. Each of the following is a measure of dispersion EXCEPT one. Which one is the EXCEPTION?
answer
Correlation RATIONALE: Correlation is not a measure of dispersion. Rather, correlation is a statistical measure for determining the strength of the linear relationship between two or more variables. Range, a measure of dispersion, is the spread between the highest and the lowest scores in a distribution. Variance, a measure of dispersion, is the measure of average deviation or spread of scores around the mean. Standard deviation, a measure of dispersion, indicates the distribution of scores in relation to the mean.
question
The orbicularis oris has two parts, the palpebral and orbital portions. The depressor supercilii pulls the eyebrow medially and is responsible for the vertical folds that develop between the eyebrows.
answer
Both statements are false. RATIONALE: It is the orbicularis oculi that can be divided into two main parts, the palpebral and orbital portions. This muscle surrounds the eye in wide sweeping arches. Note that the orbicularis oris is the complex of muscles, formerly thought of as a sphincter, that surrounds the oral cavity. The depressor supercilii is the muscle that depresses the brow. The action of the corrugator supercilii pulls the eyebrow medially and is responsible for the vertical folds that develop between the eyebrows. Also note that these muscles are classified among the muscles of facial expression.
question
The disadvantages of using povidone iodine during periodontal therapy include each of the following EXCEPT one. Which one is the EXCEPTION?
answer
May not be used on immunocompromised patients RATIONALE: Povidone iodine, effective against many organisms, is often specifically recommended for use in the treatment of immunocompromised patients. A disadvantage of this antiseptic during periodontal therapy is concern for its toxicity with prolonged use. Patients in whom povidone iodine is definitely contraindicated are those with thyroid dysfunction, as well as those with an iodine sensitivity or allergy. Patients who are pregnant or lactating should not be exposed to povidone iodine.
question
The sternohyoid muscle is classified as a suprahyoid muscle. The sternohyoid muscle is involved in the action of swallowing.
answer
The first statement is false, the second is true. RATIONALE: The sternohyoid muscle is not classified as a suprahyoid muscle. Located in the anterior section of the neck and inserting into the lower border of the hyoid bone, the sternohyoid is one of the four pairs of infrahyoid muscles. The action of the sternohyoid muscle depresses the hyoid bone during the terminal phase of swallowing. Note that depression of the hyoid bone by the sternohyoid muscle also contributes to phonation (speech).
question
An adult with partial airway obstruction and poor air exchange should not be treated with the Heimlich maneuver because this intervention is intended to treat upper-airway obstructions and symptoms of choking.
answer
The statement is NOT correct, but the reason is correct. RATIONALE: The Heimlich maneuver, often referred to as abdominal thrusting, is a procedure used to treat airway obstruction (or choking), whether partial or total. This maneuver compresses the lungs and exerts pressure to (optimally) cause expulsion of the lodged. Any partial airway obstruction can quickly become a complete airway obstruction and should therefore be treated with the Heimlich maneuver.
question
A patient with acute fluoride poisoning should first be given milk to drink, followed by measures to induce vomiting. Charcoal compounds followed by large quantities of water are recommended to induce vomiting in acute fluoride poisoning.
answer
Both statements are false. RATIONALE: For a patient with acute fluoride poisoning, vomiting should be induced immediately (before any other measure) with ipecac or manually. A patient may be given a calcium solution or milk to slow the absorption rate, but only after vomiting has occurred. Neither charcoal compounds nor large quantities of water are advised.
question
Home use of a cannula is safe for most patients without additional instruction because, as with standard and subgingival jet irrigation tips, the safety features are well documented.
answer
NEITHER the statement NOR the reason is correct. RATIONALE: A cannula is a needlelike irrigation tip, and there are safety concerns regarding its home use. Conversely, with proper instruction, both standard and subgingival (or marginal) irrigation tips can be safely used for self-care.
question
If you wanted to decrease patient radiation exposure, which of the following PIDs (position-indicating devices or cones) would you choose?
answer
16-inch rectangular RATIONALE: The additional length of the PID combined with its beam-restrictive rectangular shape make the 16-inch rectangular PID the best selection for decreasing patient radiation exposure. In general, a longer PID (position indicating device) lessens divergence of the primary x-ray beam. This decreases patient radiation exposure. Rectangular PIDs expose the patient to less radiation than circular PIDs do. The very short closed-ended, 4-inch pointed PID is commonly called a cone. It significantly increases the patient's radiation exposure and should not be used.
question
The portion of the target bombarded by electrons is called the
answer
focal spot. RATIONALE: The focal spot is the point on the target where X-radiation is produced by the interaction of accelerating electrons and atoms from target material. The target is embedded in a copper stem. The focusing cup surrounds the tungsten filament on the cathode side of the vacuum tube. The tungsten filament produces the electrons when heated.
question
Gray teeth bleach more rapidly than do yellow teeth. Whereas gingival margin irritation from tooth whitening agents is transient, tooth sensitivity is a more permanent condition.
answer
Both statements are false. RATIONALE: Yellow teeth bleach more rapidly than do gray teeth. The most common side effects of tooth whitening agents, gingival margin irritation and tooth sensitivity, are transient (short lived).
question
An example of an infectious disease and possible sequela is
answer
Lyme disease and arthritis. RATIONALE: Arthritis is permanent damage resulting from Lyme disease. Scarlet fever, which manifests as a bright-red rash covering most of the body, is a disease resulting from infection with group A Streptococcus. Jaundice is a symptom of infection with the hepatitis B virus. Emphysema is a disease of the lungs unrelated to tuberculosis. Giardiasis is the name of a prolonged diarrheal disease caused by the protozoa Giardia lamblia.
question
While working in private dental practice you volunteered extensively at a small state-funded dental clinic in a rural county whose primary employer is a large tobacco-growing operation. Community water sources are fluoridated, but a significant portion of the population relies on well water. Recently you were hired by a nonprofit organization through a state-funded grant to develop tobacco-cessation strategies and provide dental hygiene care for underserved local adults. Before you initiate services, the grant mandates that you assess the oral health status and needs of the population. An epidemiologist is available for you to consult, as is an environmental health specialist. The epidemiologist informs you that many individuals have tooth discoloration, ranging from whitish spots to dark yellow and brownish stains, as a result of excess fluoride in water from private wells. While studying Dean's Fluorosis Index as a potential tool for preventing further fluorosis, you note that each of the following concepts is correct EXCEPT one. Which one is the EXCEPTION?
answer
NHANES uses Dean's Fluorosis Index as designed in 1942 RATIONALE: The original Dean's Fluorosis Index (1942) has been slightly modified for use in the National Health and Nutrition Examination Survey (NHANES) to more efficiently assess only the maxillary anterior teeth of participants 6 to19 years of age. However, the original 1942 version, which classifies each tooth on the basis of specific characteristics of enamel, remains one of the most universally accepted classifications for dental fluorosis. In addition to Dean's Fluorosis Index, dental fluorosis has been classified with the use of the Thylstrup-Fejerskov Index, the Fluorosis Risk Index, and the Tooth Surface Index of Fluorosis.
question
Which one of the following stages of syphilis is properly matched with the clinical manifestation?
answer
Tertiary-gumma RATIONALE: The tertiary stage of syphilis is associated with gummas. These inflammatory granulomatous lesions have a central zone of necrosis. Chancres are characteristic lesions in the primary stage of syphilis. Rashes appear in the secondary stage of syphilis. The primary stage of syphilis is associated with chancres, not gummas.
question
A decrease in arterial blood pH below 7.45 is called alkalosis. A decrease in arterial blood pH below 7.35 is called acidosis.
answer
The first statement is false, the second is true. RATIONALE: Alkalosis describes a condition that develops when arterial blood pH is above normal. The normal range for arterial pH is 7.35 to 7.45. Correctly stated, an increase in arterial blood pH (above 7.45) is called alkalosis. Acidosis describes a condition that develops when the arterial blood pH is below normal. The normal range for arterial pH is 7.35 to 7.45.
question
Professionally administered (in-office) oral irrigation with 0.12% chlorhexidine gluconate is
answer
of little value because of the low substantivity of irrigating agents in a periodontal pocket. RATIONALE: The benefits of professional oral irrigation, also referred to as in-office irrigation, are not supported by research. Regardless of the agent used, research discredits the value of in-office irrigation because of the low substantivity of all antimicrobial agents in a periodontal pocket. Serum, proteins, and gingival crevicular fluid quickly break down professionally applied antimicrobial applications. Professional (in-office) oral irrigation is not routinely recommended in nonsurgical periodontal therapy because antimicrobial agents have low substantivity in periodontal pockets. The destructive effects on periodontal pathogens and beneficial effects on tissue are a result of mechanical debridement, NOT professional irrigation with any agent. Controlled-release agents such as chlorhexidine chip, minocycline microspheres, and 10% doxycycline hyclate gel are more effective than professional (in-office) oral irrigation with any agent.
question
Gracey curets are accurately described as "self-angulated" because when the lower shank is parallel to the tooth surface to be instrumented, the lower cutting edge is automatically at the correct angulation for periodontal debridement.
answer
Both the statement and reason are correct and related. RATIONALE: This statement accurately describes a major feature of Gracey curets. Considered "self-angulated," the lower cutting edge of a Gracey curet will automatically be at the correct angulation for periodontal debridement when the lower (terminal end) shank is parallel to the surface to which the instrument is to be applied.
question
An at-home oral irrigator is beneficial for a patient with a periodontal abscess because the pulsating stream of water over gingival tissue and teeth helps reduce the concentration of microorganisms and cellular end products.
answer
The statement is NOT correct, but the reason is correct. RATIONALE: At-home oral irrigation is contraindicated in patients with periodontal abscesses, ulcerative lesions, and other contraindications. However, for patients who do not have contraindications, this question correctly states that the pulsating (or steady) stream of water over gingival tissue and teeth helps reduce pathogens and cellular end products that linger in the area.
question
Bronchial asthma is characterized by all of the following conditions EXCEPT one. Which one is the EXCEPTION?
answer
Airway hyporesponsiveness to stimuli RATIONALE: During an episode of bronchial asthma, the airway is hyperreactive to stimuli, NOT hyporeactive. Bronchial asthma is characterized by airway obstruction. Bronchospasm, sudden constriction of the muscles in the walls of the bronchioles, can lead to airway obstruction. Airway inflammation can be produced by irritation. Note that airway inflammation and irritation can cause bronchospasm that can result in airway obstruction. Increased secretion (hypersecretion) of mucus glands is an overreaction to stimuli that can lead to inflammation, increased mucus production, bronchospasms, and airway obstruction.
question
Which of the following cancers has the BEST 5-year survival rate (all ages and races included)?
answer
Prostate RATIONALE: Cancer survival rates, also called survival statistics, are based on the percentage of people who survive a certain type of cancer for a specific amount of time, typically 5 years. Prostate cancer, the only selection generally characterized by slow progression, has a nearly 100% 5-year survival rate if it is detected early and treated appropriately. By contrast, lung cancer has one of the lowest 5-year survival rates (14%). Because of intensive research, expanded funding sources, and digital mammography, 5-year survival rates in breast cancer are improving and now approach 86% or higher when cancer is detected early and treated appropriately. The 5-year survival rates for ovarian cancer is still only 45% because of the difficulty of early detection and ineffective treatment protocols. The 5-year survival rate for colorectal cancer ranges from 6% to 74%, depending on the stage at the time of diagnosis. If detected in a precancerous polyp stage, colorectal cancer is considered curable.
question
The device used to measure blood pressure is called a
answer
sphygmomanometer. RATIONALE: A sphygmomanometer is the device used to measure blood pressure. This device is commonly known as a blood pressure cuff. A stethoscope is used to listen to the rush of the blood through the artery that is compressed by the cuff. Korotkoff sounds are what are heard as the blood flows through the artery as it is compressed by the blood pressure cuff. A dosimeter is used to measure radiation exposure.
question
Fetal alcohol syndrome, like Bell's palsy, involves multiple malformations and malocclusions. Cretinism, Down syndrome and both bilateral cleft lip and palate cause malformation in the eye spacing and in jaw structure.
answer
Both statements are false. RATIONALE: Whereas fetal alcohol syndrome involves multiple malformations and malocclusions, Bell's palsy does not. Although Bell's palsy involves paralysis in muscles, this occurs after bone formation is complete. Cretinism causes malformation in eye spacing and jaw structure, but Down syndrome and cleft lip/palate do not. Down syndrome causes craniofacial malformation and malocclusions. Bilateral cleft lip and palate create major developmental malformations of the palate and alveolar bone. Note that these disruptions can cause malocclusion.
question
The mechanism that converts excess dietary carbohydrate to fat and stores it in adipose tissue as a long-term energy source is called
answer
Lipogenesis RATIONALE: Lipogenesis is the metabolic process that converts excess carbohydrates to fat and stores it in adipose tissue (fat) for long-term energy needs. Glyconeogenesis is the synthesis (formation) of glucose from non-carbohydrate sources, such as amino acids and glycerol. Active transport is an absorptive mechanism of the digestive process that requires energy to enable molecules to pass against a concentration gradient with the aid of an ion channel or carrier protein at the brush border. Anabolism is a metabolic process that builds, or synthesizes, cells, tissues, and organs. (Catabolism is the opposite mechanism of anabolism.)
question
Lipid-soluble endocrine secretions that are classified by chemical structure and synthesized from cholesterol are known as
answer
steroid hormones. RATIONALE: Steroid hormones, classified by their chemical structure, are synthesized from cholesterol. As with all hormones, steroids are secreted by the endocrine system and regulate physiologic activity of most cells. Carried to almost every point in the body, hormones last longer and work more slowly than neurotransmitters do. Tropic hormones, classified by general function, target other endocrine glands and stimulate their growth and secretion. Glycoprotein hormones, classified by chemical structure, are long folded chains of amino acids. Insulin and parathyroid hormone are examples of glycoprotein hormones. Anabolic hormones, classified by general function, stimulate anabolism (the synthesis of complex molecules).
question
Patients with rheumatoid arthritis are at risk for Addison's disease because Addison's disease is caused by an insufficient level of cortisol.
answer
The statement is NOT correct, but the reason is correct. RATIONALE: Although patients with rheumatoid arthritis have many health risks, they are not at risk for Addison's disease. Addison's disease is caused by insufficient production of cortisol by the adrenal glands. Also called adrenal insufficiency, this condition can progress to adrenal crisis, a life-threatening emergency. Adrenal crisis is also called Addisonian crisis and acute adrenal crisis.
question
The protective features of Gracey curets include each of the following EXCEPT one. Which one is the EXCEPTION?
answer
Triangular cross-section RATIONALE: Gracey curets are not triangular in cross-section. Instead, it is the cross-sectional design of sickle scalers that creates both a pointed back and toe that makes "sickles" unacceptable for use subgingivally or near the gingival margin. By contrast, curets have numerous features, including a rounded toe and a rounded back, that are specifically designed to protect surrounding gingival tissues. Unique to Gracey curets is the protective design feature in which the face of the working end is tilted in relation to the lower shank.
question
Which of the following fluoride-releasing restorative material is of limited use?
answer
Metal-modified glass ionomer RATIONALE: Although they have fluoride-releasing properties, metal-modified glass ionomer restorative materials have only limited use in dentistry. Glass ionomer fluoride-releasing restorative materials are widely used in Class V restorations. Because of the location of Class V restorations near the gingival margin, glass ionomer restorative materials are widely used in geriatric dentistry. Resin-modified restorative materials are widely used, including applications in both geriatric and pediatric dentistry. Compomer, a widely used fluoride-releasing restorative material, is used in Class I, Class II, and Class III restorations.
question
A 42-year-old woman taking no medications and with no significant past medical history is scheduled for comprehensive dental hygiene care. Initially, she has no complaints about her health, and her vital signs are within normal limits. After you explain the procedure to be performed and initiate the dental hygiene intervention, the woman begins to complain of "feeling faint." Once this woman states that she is feeling fine, what should the dental hygienist do?
answer
Once vital signs are normal, discharge the patient and allow her to go home RATIONALE: Once vital signs are normal and the woman feels well, she may go home. Continuing with care is not advised after a near-syncopal episode. There is no need to transport this patient to a hospital unless she does not recover fully or there are other signs or symptoms that make you feel that the episode was more serious than just a syncopal episode. There is no need to encourage the patient to be evaluated by her primary care physician unless she does not recover fully or there are other signs or symptoms that make you feel that the episode was more serious than just a syncopal episode.
question
Under tort law, the degree of care that a reasonably prudent profession should exercise is called
answer
standard of care. RATIONALE: Standard of care is defined as the degree of care that a reasonably prudent professional should exercise, for example, in practicing within the rules and regulations of the state or provincial dental practice act. Minimum requirements of acceptable care are a component of the concept of standard of care. Res ipsa loquitur is a Latin term that means "the thing speaks for itself." Appropriation is the use of another's name or likeness for financial gain.
question
A community health clinic located in an underserved rural community has hired a dental hygienist to design and implement a program to reduce early childhood caries (ECC). Most of the population does not benefit from community fluoridated water, nor are school-based fluoridation programs available. During initial research, the dental hygienist learns that a strong coalition within the community opposes any type of fluoridation. To prioritize needs, goals and objectives the program administrator has recommended use of the SMART formula. The clinic, funded by a state agency, hopes to expand oral services to older children and adults in the next few years. Successful community oral health programs decrease the need for urgent dental care because they effectively decrease demand for dental services.
answer
The statement is correct, but the reason is NOT. RATIONALE: The goal of community oral health programs is to decrease the need for urgent dental care by increasing the demand for dental services. In contrast, in underserved areas, the need for dental care is high because the demand for care is low.
question
Application of fluoride varnish after dental hygiene treatment is considered macro-allocation of resources because it exemplifies fair allocation of resources on the basis of individual needs.
answer
The statement is NOT correct, but the reason is correct. RATIONALE: Application of fluoride varnish after dental hygiene treatment is considered micro-allocation of resources. The application of fluoride varnish after dental hygiene treated is an example of fair allocation of resources on the basis of individual needs.
question
During the first moments of dental hygiene treatment, a 42-year-old woman with no significant medical history, no current health concerns, and normal vital signs who is taking no medications begins to complain of "feeling faint." Even after the dental chair is reclined, the woman becomes very pale and loses consciousness. During the loss of consciousness, all of the following signs may be noted EXCEPT one. Which one is the EXCEPTION?
answer
"Fruity" breath odor RATIONALE: On the basis of the situation described, the most likely diagnosis is syncope due to psychogenic factors, which is not associated with a fruity breath odor. Syncope, associated with a diabetic emergency (hyperglycemia), is commonly known as fainting. This short loss of consciousness and muscle strength is the result of a decrease in blood flow to the brain, usually because of low blood pressure. Common characteristics of syncope include a weak pulse, flaccid muscles, and shallow respirations.
question
All of the following accurately describe chlorhexidine gluconate EXCEPT one. Which one is the EXCEPTION?
answer
Chlorhexidine gluconate is an over-the-counter chemotherapeutic mouthrinse RATIONALE: Each of the statements above is correct EXCEPT that chlorhexidine gluconate is NOT available as an over-the-counter (OTC) mouthrinse. Because chlorhexidine can have side effects, it is available by only prescription and is recommended for short-term use, and its use should be carefully monitored by a dental hygienist or dentist. Whether in its original formulation or the newer alcohol-free formulation, chlorhexidine gluconate may cause brownish-yellow staining, as well as increased supragingival calculus.
question
All of the following muscles move the head EXCEPT one. Which one is the EXCEPTION?
answer
Brachialis RATIONALE: The brachialis muscle does not move the head; rather, its function is to flex the pronated forearm. Of the numerous muscles that move the head, the sternocleidomastoid muscle is specifically involved in flexion of the head. Similarly, contraction of the semispinalis capitus muscle makes it possible to extend the head as well as to bend it laterally. Also involved in extension, the splenius capitus muscle enables the head to bend and rotate toward the same side.
question
Both allergic reactions and psychological disorders may result in anaphylactic shock. Septic shock is usually caused by an infectious agent, whereas cardiogenic shock may be caused by heart failure.
answer
The first statement is false, the second is true. RATIONALE: Allergic reactions, but not psychological disorders, may result in anaphylactic shock. Neurogenic shock is associated with psychological disorders. Septic shock is usually caused by an infectious agent. Cardiogenic shock may be caused by heart failure.
question
The protein efficiency ratio (PER), an indication of a protein's ability to support protein synthesis, is defined as the
answer
measure of a protein's ability to support growth. RATIONALE: The protein efficiency ratio (PER) is a measure of a protein's ability to support growth. This is one of many measures of a protein's ability to support protein synthesis. The expression of the percentage of nitrogen retained for maintenance and growth compared with the amount absorbed is the biologic value (BV) of a protein. The expression of the percentage of retained nitrogen compared with the amount ingested is called the net protein utilization (NPU). The NPU differs from the biologic value (BV) because it takes into account the protein's digestibility. The comparison of the amino acid balance of a food protein with the amino acid requirements of preschool-age children and then corrected for digestibility is called the protein digestibility-corrected amino acid score (PDCAAS).
question
Coughing and aspiration resulting from mucus or saliva accumulation or impaired oral reflexes is seen in all of the following conditions EXCEPT one. Which one is the EXCEPTION?
answer
Autism spectrum disorders RATIONALE: Autism spectrum disorders create no problems with mucus accumulation or swallowing. Bronchial asthma causes hypersecretion of mucous. In emphysema, mucus obstructs airways. Mucus obstructs airways in cystic fibrosis. People with Parkinson's disease have difficulty swallowing.
question
Michael, a 12-year-old Asian adolescent, is scheduled for his first dental hygiene care appointment and dental examination in 5 years. His mother reports that he is generally is good health but takes Adderall XR 30 mg/day for attention deficit hyperactivity disorder and still exhibits clumsiness, irritability, and an inability to focus for even short periods. He is not doing well in school, and the teachers have recommended that he be held back a year. His mother also reports that the "goose egg" on his forehead was sustained while he was roughhousing under the kitchen table. She says that he is always getting bumps and bruises because of his inability to control his behavior. All of the following are appropriate actions on the part of the dental hygienist in relation to Michael's history of minor injuries EXCEPT one. Which one is the EXCEPTION?
answer
Suggesting that the parents are abusing him RATIONALE: There is no evidence that the parents have abused Michael. The fact that he is taking Adderall suggests that he is receiving medical care for ADHD. Because he is prone to accidents and hasn't seen a dentist, he should be checked for oral injuries. Dental hygienists should be providing anticipatory guidance to parents about injury-prevention behaviors. More information is needed about the causes and types of oral injuries before counseling is attempted. This information is needed for counseling and to ensure his safety during dental hygiene appointments.
question
Ethical duties and legal duties are usually of the same magnitude. Ethics are societal mandates.
answer
Both statements are false. RATIONALE: Ethical duties are usually more important than legal duties; for example, a health care provider who is acquitted in civil or criminal court after being charged with illegal conduct did not necessarily act ethically. Ethics are professionally based, whereas laws are societal mandates.
question
Electronic periodontal probes
answer
reduce the time required for probing. RATIONALE: Electronic periodontal probes (collectively called controlled-force probes) have many advantages, including a reduction in the time required for probing. Disadvantages include a decrease in both tactile sensitivity and patient comfort. With the elimination of mouth-to-paper chart recordings, infection control efficacy is enhanced by electronic periodontal probes.
question
A collection of sebaceous glands commonly found on the gingiva is called a mucocele. A mucocele may result from trauma to a minor salivary duct.
answer
The first statement is false, the second is true. RATIONALE: A collection of sebaceous glands commonly found on the gingiva is referred to as Fordyce granules. A mucocele is a benign mucous-containing cystic lesion of a minor salivary gland. The most common cause of a mucocele is trauma to a minor salivary gland.
question
A 6-year-old girl arrives for her 6-month dental hygiene care appointment with just one complaint. She feels "itchy all over." It started about 10 minutes ago, when she was outside playing in the grass. Initially, her vital signs are normal except for slight tachycardia. She has some skin lesions that appear to be urticaria, and her lower lip is beginning to swell. Within a minute she complains of difficulty breathing and her respiratory rate is increased. This child is MOST likely experiencing
answer
an anaphylactic reaction. RATIONALE: Respiratory distress from an immediate allergic reaction is progressing to an anaphylactic reaction. Anaphylaxis is defined as a severe, potentially life-threatening allergic reaction. This is not a mild reaction, nor is it delayed. There is not a mild form of anaphylaxis. Although this child is having a skin reaction, she is not experiencing hyperventilation.
question
A narrow slit in gingival, mucosal, or epidermal tissue is called an exostosis. An area of hardened or abnormally hard tissue is described as idiopathic.
answer
Both statements are false. RATIONALE: TOP: A narrow slit or cleft in gingival, mucosal, or epidermal tissue is called a fissure. A fissured tongue is an example. Note that such areas harbor microbial growth and are highly susceptible to infection, ulceration, inflammation, and pain. Conversely, an exostosis is a benign bony growth, such as a mandibular tori, that projects from the surface of a bone. An area of hardened or abnormally hard tissue is described as indurated. Note that the term idiopathic refers to a condition or lesion of unknown origin.
question
Which of the following interventions is NOT a treatment for leukemia?
answer
Renal surgery RATIONALE: Renal surgery is not a treatment for leukemia. Irradiation is used to destroy malignant cells. The combination of chemotherapy with blood transfusions and antibiotics destroys malignant cells, injects normal cells, and reduces the chance of infection. Interferon therapy, first described in 1957 but not widely used until more recently, has been shown to be an effective treatment for leukemia. Bone marrow transplant replaces abnormal cells with normal cells.
question
When periodontal instrumentation is completed on two quadrants at one appointment, treatment of a maxillary and mandibular quadrant on the same side of the mouth is recommended because this approach provides the patient with one side of the mouth on which to comfortably chew.
answer
Both the statement and reason are correct and related. RATIONALE: Selecting two quadrants on the same side of the mouth is the preferred option for several reasons. Providing the patient with one side of the mouth to comfortably chew is important. Also validating this choice is the fact that when anesthesia is used, the patient's tongue and tissues are only numb on one side.
question
Tooth mobility can be caused by all of the following EXCEPT one. Which one is the EXCEPTION?
answer
Gingival enlargement RATIONALE: Gingival enlargement that does not progress to periodontal damage does not cause tooth mobility. Tooth mobility results from forces that cause the loosening of a tooth within its socket. Abnormal stresses to the periodontium during occlusal trauma can cause tooth mobility. Inflammation from an abscess can cause tooth mobility if it involves the periodontal ligament. The same is true of inflammation from periodontal infections. Hormonal changes such as those seen in puberty, pregnancy, and menopause can cause tooth mobility.
question
Which of the following cartilages is signet ring-shaped?
answer
Cricoid RATIONALE: The cricoid cartilage is signet ring-shaped and is attached by membranes to the upper part of the trachea. The arytenoid cartilages, a pair of small three-sided pyramids, are located on the back of the larynx. The corniculate cartilages are two tiny cones, one on the apex of each arytenoid cartilage. The cuneiform cartilages are two tiny rods, placed in the mucous membrane folds, that connect the arytenoids to the epiglottis.
question
While working in private dental practice you volunteered extensively at a small state-funded dental clinic in a rural county whose primary employer is a large tobacco-growing operation. Community water sources are fluoridated, but a significant portion of the population relies on well water. Recently you were hired by a nonprofit organization through a state-funded grant to develop tobacco-cessation strategies and provide dental hygiene care for underserved local adults. Before you initiate services, the grant mandates that you assess the oral health status and needs of the population. An epidemiologist is available for you to consult, as is an environmental health specialist. Four components necessary to address and develop address before the initiation of an oral health program are assessment, planning, implementation, and evaluation. A community oral health assessment is a narrowly constructed process designed to extend data collected in private dental offices to wider segments of the population.
answer
The first statement is true, the second is false. RATIONALE: Before an oral health program is initiated, the four vital components of assessment, planning, implementation, and evaluation must be addressed and developed. A community oral health assessment is neither narrow nor relies on data collected in private dental offices. A community oral health assessment is a multifaceted process that is both community oriented and community directed.
question
An infection acquired during a hospital stay is referred to as both subclinical and opportunistic. The absence of recognizable symptoms is typical in focal, primary, and subclinical infections.
answer
Both statements are false. RATIONALE: An infection acquired during a hospital stay is referred to as a nosocomial infection. A subclinical infection is one in which no symptoms are recognized. Opportunistic infections are those that occur when an organism that does not usually cause disease becomes pathogenic. Although it is true that no symptoms are recognized in subclinical infections, this is not the case for focal or primary infections. A focal infection is initially localized to one area and spreads elsewhere. A primary infection is an original infection.
question
Michael, a 12-year-old Asian adolescent, is scheduled for his first dental hygiene care appointment and dental examination in 5 years. His mother reports that he is generally in good health but that he takes Adderall XR 30 mg/day for attention deficit hyperactivity disorder and still exhibits clumsiness, irritability, and an inability to focus for even short periods. He is not doing well in school, and the teachers have recommended that he be held back a year. His mother also reports that the "goose egg" on his forehead was sustained while he was roughhousing under the kitchen table. She says that he is always getting bumps and bruises because of his inability to control his behavior. Which of the following is the MOST important consideration in providing dental and dental hygiene care for Michael?
answer
Ensuring a safe and structured environment in the dental treatment area during the appointment RATIONALE: Because of his hyperactivity and tendency to injuries, minimizing risk and ensuring safety is your MOST important consideration in providing care for this patient. Radiographs, especially a complete set, may be difficult to obtain. At this time, an exam is more important. Using oral premedication or nitrous oxide and oxygen conscious sedation to control behavior could be considered, but it is not your first priority. Determining the etiology of Michael's behavioral disorder is the role of the physician or educational psychologist. Flossing is most likely an unrealistic goal at this point; toothbrushing is likely to be more successful.
question
External root resorption can occur as a result of orthodontic treatment because excess forces caused by tooth movement can stimulate osteoclasts.
answer
Both the statement and reason are correct and related. RATIONALE: If excess orthodontic forces cause teeth to move too quickly, external root resorption may occur.
question
All of the following are etiologic factors for gingival recession EXCEPT one. Which one is the EXCEPTION?
answer
Alveolar bone loss RATIONALE: Alveolar bone loss can occur with or without apical migration of the gingival margin. Position of a tooth within the arch is an etiologic factor for gingival recession. Dehiscence, the loss of alveolar bone on the facial (rarely lingual) aspect, is an etiologic factor for gingival recession. Faulty toothbrushing causes gingival trauma and is a common etiologic factor for gingival recession. Gingival inflammation often causes changes within the gingival architecture; it can be an etiologic factor for gingival recession.
question
Which of the following organs is able to tolerate the greatest degree of ischemia?
answer
Skin RATIONALE: Ischemia is defined as restriction of blood flow. Of the organs listed, the skin can tolerate the greatest degree of ischemia. The brain can least tolerate restriction of blood flow. Skeletal muscle is easily damaged by ischemia. The heart can be easily damaged by ischemia. Note that ischemic heart disease is characterized by reduced blood supply to the heart.
question
The administration of oxygen is NOT indicated in the treatment of
answer
hyperventilation. RATIONALE: Administration of oxygen is contraindicated in hyperventilation. This patient should be rebreathing air with decreased oxygen content to improve symptoms. Patients with asthma should receive oxygen. Patients with myocardial infarction should receive oxygen. Patients with chronic obstructive pulmonary disease (COPD) should receive oxygen.
question
Removal of subgingival calculus during periodontal debridement is critical to successful therapy because in the presence of bacteria from the plaque biofilm, subgingival calculus has the ability to degrade cementum and invade the junctional epithelium.
answer
The statement is correct, but the reason is NOT. RATIONALE: Removal of subgingival calculus during periodontal debridement is critical to successful therapy because the bacterial plaque biofilm that always overlays calculus causes periodontal infection in humans. Therefore it is well established that the role of calculus in harboring pathogens is the critical reason for its disruption and removal during periodontal debridement. Two important distinctions: Bacteria from the plaque biofilm enter the junctional epithelium, but calculus remains coronal to the junctional epithelium and does not invade this barrier. Although both plaque and calculus can degrade cementum within a periodontal pocket, this is not the primary reason for its removal.
question
An anaphylactic reaction in a child can quickly progress to congestive heart failure because the immediate allergic reaction of anaphylaxis produces constriction of the airways and a weak, rapid pulse.
answer
The statement is NOT correct, but the reason is correct. RATIONALE: An anaphylactic reaction, in a patient of any age, produces constriction of airways and a weak, rapid pulse. However, this condition can quickly progress to cardiovascular collapse, not congestive heart failure. An important distinction: Whereas an anaphylactic reaction is an immediate allergic reaction, congestive heart failure occurs over time as the heart's pumping power becomes increasingly impaired.
question
Which of the following conditions causes a delay in dental development?
answer
Hypothyroidism RATIONALE: Hypothyroidism delays growth in many areas, including the teeth and jaws. Autism is a developmental disorder that affects behavior, communication, speech, language, social interactions, and sensory impairment; it does not affect dental development. Multiple sclerosis is a neuromuscular disorder of adults and doesn't affect dental development. Hyperthyroidism causes accelerated dental development. Spina bifida has no effect on dental development.
question
During root planing, the need for extensive cementum removal to attain a glassy, smooth surface is widely accepted. While performing root planing with anesthesia, a skilled dental hygienist should be able to accomplish complete removal of calculus in areas of deep, restricted pockets.
answer
Both statements are false. RATIONALE: The need to obtain a glassy, smooth surface through the removal of extensive cementum during root planing is still being debated. Relevant issues include a lack of clarity about its absolute necessity, as well as the possibility of overtreatment and resultant hypersensitivity. Even when anesthesia has been administered, complete removal of calculus from root surfaces with deep, restricted pockets is unlikely because of inaccessibility. Note the distinction―complete removal of detectible calculus remains the initial clinical endpoint. Furthermore, thorough instrumentation remains paramount to the success of therapy, both nonsurgical and surgical.
question
Which one of the following is NOT considered a variant of normal?
answer
Hairy leukoplakia RATIONALE: Hairy leukoplakia, most often associated with HIV or AIDS, is considered a condition rather than a variant of normal. In contrast, white hairy tongue, fissured tongue and geographic tongue are each considered variants of normal. Migratory glossitis is another name for geographic not considered a variant of normal because its more classified as a condition Rather it is considered a condition most often associated with HIV or AIDS. White hairy tongue is a variant of normal. Fissured tongue is considered a variant of normal. Geographic tongue is considered a variant of normal tongue.
question
Which of the following diagnostic imaging systems uses a conventional x-ray unit and an anatomically adapted charge-coupled device (CCD) sensor to produce an x-ray image on a computer monitor or video screen?
answer
Digital imaging RATIONALE: Digital imaging uses computer-based devices and a conventional x-ray unit to produce radiographic images. Arthrography involves the injection of contrast medium to image joint dysfunction. Magnetic resonance involves the use of radio frequency emitted from tissue, not ionizing radiation. Nuclear medicine involves the injection of isotopes to record radiation emitted from tissues. Sialography involves the injection of contrast agents to evaluate ductal and acinar systems.
question
The four to five modular pieces in the coccygeal vertebral area are normally fused with several of the lumbar vertebrae. There are seven cervical vertebrae, several of which are fused with the thoracic vertebrae.
answer
Both statements are false. RATIONALE: Although there are four to five modular pieces in the coccygeal vertebral area, in the adult they may be fused with the sacrum. Note that none of the coccygeal vertebrae is fused to any of the five lumbar vertebrae. Though it's true that there are seven cervical vertebrae, none is fused. Nor is any cervical vertebra fused with any structure within the thoracic area.
question
Products that contain amorphous calcium phosphate (ACP) can be appropriately substituted for fluoride therapy because ACP contains the same minerals as the hydroxyapatite crystals of tooth enamel.
answer
The statement is NOT correct, but the reason is correct. RATIONALE: Although amorphous calcium phosphate (ACP) contains the same minerals as the hydroxyapatite crystals of tooth enamel, ACP should not be substituted for fluoride therapy. Rather, ACP should be used in conjunction with fluoride to enhance fluoride uptake.
question
Ganglions of the sympathetic division of the autonomic nervous system are classified as either sympathetic trunk ganglia or prevertebral ganglia. Ganglions of the parasympathetic division of the autonomic nervous system are called terminal ganglia
answer
Both statements are true. RATIONALE: Ganglions of the sympathetic division of the autonomic nervous system are classified as sympathetic trunk ganglia (lateral to the vertebral column) and prevertebral ganglia (anterior to the vertebral column). Parasympathetic ganglia, called terminal ganglia, are located near or within visceral effectors.
question
The rate of expansion or contraction of one dimension of a dental cement with temperature change is known as its
answer
linear coefficient of thermal expansion. RATIONALE: The rate of expansion or contraction of one dimension of a dental material as the temperature changes is called the linear coefficient of thermal expansion (LCTE, α). This is considered a thermal property of the material. The mixing interval (the duration of the stage of mixing a material), is a category within the stages of manipulation as defined for the particular material. Thermal conductivity, a one of the thermal properties of a dental material, is influenced by factors including insulation and conduction. The resistance of a material to acceleration, or being moved, is called mass.
question
Which of the following components of the digestive process is responsible for the mechanical breakdown and mixing of food?
answer
Teeth and tongue RATIONALE: The teeth and tongue are responsible for the mechanical breakdown and mixing of food. Saliva is responsible for hydration and lubrication of food. Salivary amylase, also known as ptyalin, begins the initial enzymatic hydrolysis of starch. Keratin sulfate is a complex carbohydrate that contributes to formation of nails and teeth.
question
A community health clinic located in an underserved rural community has hired a dental hygienist to design and implement a program to reduce early childhood caries (ECC). Most of the population does not benefit from community fluoridated water, nor are school-based fluoridation programs available. During initial research, the dental hygienist learns that a strong coalition within the community opposes any type of fluoridation. To prioritize needs, goals and objectives the program administrator has recommended use of the SMART formula. The clinic, funded by a state agency, hopes to expand oral services to older children and adults in the next few years. For a child aged 12 to 23 months, more than one decayed, missing, or filled primary tooth surface constitutes early childhood caries. For a child aged 72 months, more than one smooth decayed, missing, or filled surface constitutes severe early childhood caries.
answer
The first statement is true, the second is false. RATIONALE: The American Academy of Pediatric Dentistry categorizes early childhood caries (ECC) both by age and by the decayed, missing, and filled primary tooth surface (dmfs) index. Children ages 12 to 23 months with more than one decayed, missing, or filled primary tooth surface (>1 dmfs) are considered to have ECC. Age 72 months (6 years) is no longer considered early childhood in the classification of ECC.
question
The official name of a drug determined by the U.S. Adopted Names Council is called the
answer
generic name. RATIONALE: The generic name is the official name of a drug that is determined by the U.S. Adopted Names Council. The generic name is used by all manufacturers of a particular drug. The chemical name of a drug is based on the drug's chemical formula. Each drug company makes up its own trade name for its product. The trade name is also known as the proprietary name. The brand name is technically the name of the drug company itself but is often used interchangeably with the trade name.
question
A dental hygiene diagnosis interprets data collected during the assessment phase of care. A dental hygiene diagnosis identifies a specific oral disease.
answer
The first statement is true, the second is false. RATIONALE: A dental hygiene diagnosis, defined as the interpretation of data collected during the assessment phase of care, identifies significant findings, recognizes deviations from normal, describes abnormalities, and analyzes the significance of abnormalities. Note that although the dental diagnosis identifies a specific oral disease, the dental hygiene diagnosis identifies human needs deficits related to dental hygiene care.
question
You have been asked to evaluate the federally funded dental program at a local senior citizen center in a northeastern state. Located in a diverse inner-city neighborhood of a large metropolitan city, the center serves an ethnically diverse population of varying socioeconomic status. Community water supplies are fluoridated, and students from local dental hygiene programs occasionally provide oral health education and oral screenings under the supervision of a licensed dentist. Both a registered nurse and a dietitian are available for you to consult with, as are a dentist and a gerontologist. Breakfast and lunch are served five days a week, and movies, complete with soft drinks and snacks, are available most afternoons. Factors influencing adherence to health promotion and preventive regimens include each of the following EXCEPT one. Which one is the EXCEPTION?
answer
Autocratic messages RATIONALE: Autocratic messages and authoritarian delivery of information by the dental hygienist are an ineffective health promotion strategy. In contrast, messages delivered in a genuine, positive, and considerate manner have the highest likelihood of achieving patient adherence. The quality of communication between the patient, or the community, and the hygienist is critical to achieving patient adherence to healthy behaviors. Whether addressing private practice patients or a large community, shared responsibility/active participation by each patient influences the outcome. Support systems are important, especially for children and patients with disabilities. Support systems should be considered in the design of community oral health programs.
question
Disorders with the strongest evidence linking genetic mutations and periodontitis include each of the following EXCEPT one. Which one is the EXCEPTION?
answer
Down syndrome RATIONALE: Although Down syndrome is a genetic disorder that is associated with periodontitis, it is not among those with the strongest evidence of a link between genetic mutations and periodontitis. Among genetic disorders with established links to periodontitis are Papillon-Lefèvre syndrome, Chédiak-Higashi syndrome, and Ehlers-Danlos syndrome . In Papillon-Lefèvre syndrome, the severe destruction of the periodontium results in loss of most primary teeth by the age of 4 and most permanent teeth by age 14. In some cases of Chédiak-Higashi syndrome, periodontitis can be the clinical finding leading to the correct diagnosis. Early-onset generalized periodontitis is one of the most significant oral findings of Ehlers-Danlos syndrome.
question
Diastolic pressure is
answer
the lower reading in the blood pressure measurement. RATIONALE: Diastolic blood pressure, which is represented by the lower, or "bottom" reading, measures pressure in the arteries between heartbeats. It's important to note that the diastolic reading registers pressure when the heart muscle is resting. Diastolic pressure is not a form of hypertension but rather an indicator representing the blood pressure when the heart is at rest. Diastolic pressure is not a form of hypotension. Diastolic blood pressure is not the higher, or "upper" reading. The "upper" reading is known as the systolic pressure.
question
While working in private dental practice you volunteered extensively at a small state-funded dental clinic in a rural county whose primary employer is a large tobacco-growing operation. Community water sources are fluoridated, but a significant portion of the population relies on well water. Recently you were hired by a nonprofit organization through a state-funded grant to develop tobacco-cessation strategies and provide dental hygiene care for underserved local adults. Before you initiate services, the grant mandates that you assess the oral health status and needs of the population. An epidemiologist is available for you to consult, as is an environmental health specialist. Although tobacco-cessation programs in private practice are beneficial, tobacco-cessation programs in community health settings are ineffective because they lack one-to-one communication. A campaign to teach regular self-examination of the oral cavity is a vital part of a community oral health program.
answer
The first statement is false, the second is true. RATIONALE: No community oral health program is complete without tobacco-prevention strategies. This community in particular, with an economic base dependent on tobacco growth, desperately needs such preventive intervention. It is indisputable that a campaign to teach regular self-examination of the oral cavity is beneficial for all involved. Statistics clearly demonstrate that early detection and prompt treatment vastly improve outcomes in cases of oral and pharyngeal cancer.
question
Which of the following interactions of x-rays and matter results in low-energy radiation produced by outer shell electrons shifting to an inner vacancy?
answer
Photoelectric effect RATIONALE: The photoelectric effect, resulting in low-energy radiation production, occurs when an outer shell electron is ejected from its orbit and it shifts to an inner vacancy. The Compton effect results from an incident electron ejecting an outer shell electron. Thomson scattering occurs when an incident electron passes near an outer shell electron, causing vibration and ionization. Coherent scattering is another name for Thomson scattering.
question
Which of the following is NOT an oral manifestation of HIV infection?
answer
Pneumocystis jiroveci RATIONALE: Pneumocystis jiroveci (formerly known as Pneumocystis carinii) is not an oral manifestation of HIV infection. Instead, P. jiroveci is the name of the fungus that causes pneumocystis pneumonia (PCP), one of the most frequent and severe opportunistic infections in people with weakened immune systems, particularly those with HIV/AIDS. Oral manifestations of HIV disease are very common and include oral lesions with such unique presentations that they were previously known as opportunistic diseases. Examples include hairy leukoplakia, candidiasis, linear gingival erythema, and papillomavirus (warts). Because these conditions may be indicative of other underlying diseases, careful history taking and detailed examination of the patient's oral cavity are important parts of the physical examination. Early recognition, diagnosis, and treatment of HIV-associated oral lesions may reduce morbidity.
question
A community health clinic located in an underserved rural community has hired a dental hygienist to design and implement a program to reduce early childhood caries (ECC). Most of the population does not benefit from community fluoridated water, nor are school-based fluoridation programs available. During initial research, the dental hygienist learns that a strong coalition within the community opposes any type of fluoridation. To prioritize needs, goals and objectives the program administrator has recommended use of the SMART formula. The clinic, funded by a state agency, hopes to expand oral services to older children and adults in the next few years. After a research question is formulated, the next step is the development of a hypothesis. A null hypothesis assumes that there is statistically significant correlation among the groups being studied.
answer
Both statements are false. RATIONALE: After formulating a research question, the next step is the development of a hypothesis. A null hypothesis assumes that there is no statistically significant difference between the groups being studied. Correlation is measured with the correlation coefficient.
question
During the first moments of dental hygiene treatment, a 42-year-old woman with no significant medical history, no current health concerns, and normal vital signs who is taking no medications begins to complain of "feeling faint." Even after the dental chair is reclined, the woman becomes very pale and loses consciousness. The next measures the hygienist should initiate are first assessing the airway, opening it if necessary, and then placing a bite block in the patient's mouth. In this situation, after reclining the dental chair, the hygienist should next take the action of activating the emergency medical system (EMS). The next measure the hygienist should take is moving the patient from the dental chair.
answer
Both statements are false. RATIONALE: Although it is true that after reclining the chair (to the Trendelenburg position), the hygienist should next assess the airway, opening it if necessary, this statement in its entirety is false because the situation does not warrant placing a bite block in the mouth. This could cause harm. Although EMS should be activated if there is any suspicion that the patient is suffering a major respiratory or cardiac event, the most likely diagnosis, on the basis of the situation described, is syncope due to psychogenic factors. The patient should not be moved from the dental chair. She will benefit from remaining in the dental chair (reclined in the Trendelenburg position), where her head will be lower than her legs, thus increasing blood flow to her brain.
question
The law, as it relates to dental hygiene practice, classifies each of the following EXCEPT one as an intentional tort. Which one is the EXCEPTION?
answer
Negligence RATIONALE: As related to the practice of dental hygiene, the law classifies negligence as an unintentional tort. Assault and battery, defamation, breach of confidentiality, and misrepresentation are classified as intentional torts. Tort law is a body of rights, obligations, and remedies that is applied by courts in civil proceedings to provide relief to persons who have suffered harm from a tort; a tort is simply a civil wrong.
question
An example of an acid-producing bacteria that is also aciduric is
answer
Lactobacillus. RATIONALE: Lactobacilli not only produce acid but are also aciduric in nature, because they survive in a relatively acidic environment (low pH). Aciduric bacteria are also known as "acid-loving." Although Streptococcus mitis produces hydrogen peroxide, a relatively weak acid, it is not aciduric, because it cannot survive an acidic environment (low pH). Streptococcus sanguis produces numerous acids, including hydrogen peroxide, but is not considered aciduric, because it cannot survive an acidic environment (low pH). Streptococcus viridans produces numerous acids, including the mildly acidic hydrogen peroxide. However, it cannot survive in an acidic environment (low pH), so it is not considered aciduric.
question
A community health clinic located in an underserved rural community has hired a dental hygienist to design and implement a program to reduce early childhood caries (ECC). Most of the population does not benefit from community fluoridated water, nor are school-based fluoridation programs available. During initial research, the dental hygienist learns that a strong coalition within the community opposes any type of fluoridation. To prioritize needs, goals and objectives the program administrator has recommended use of the SMART formula. The clinic, funded by a state agency, hopes to expand oral services to older children and adults in the next few years. Which preventive program has the BEST potential for success with this population?
answer
Sodium fluoride varnish RATIONALE: Sodium fluoridate varnish is recommended by the ADA, the CDA, and the CDC for use in preschool-age children. Studies have demonstrated a 45% reduction in caries with public health sodium fluoride varnish programs. Community water fluoridation, an optimal long-term goal, has met with community opposition and is not likely to be implemented soon. School water fluoridation, once recommended in areas with suboptimal fluoride, provides no benefit until children begin school. Although fluoridated milk shows promise in research studies, milk is not an ideal vehicle for fluoride delivery.
question
All of the following are characteristic of supernumerary teeth EXCEPT one. Which one is the EXCEPTION?
answer
Never present in posterior regions RATIONALE: Although most supernumerary teeth occur in the anterior region, they are often present in posterior regions. Because most supernumerary teeth are unerupted, it is significant that supernumerary teeth are most often discovered radiographically. Although there are numerous causes of supernumerary teeth, they are usually genetic/hereditary in etiology. Supernumerary teeth are associated with certain malformations and syndromes. This is especially true when multiple supernumerary teeth are present. Cleft lip and palate, cleidocranial dysplasia, and Gardner syndrome are among the syndromes/malformations associated with supernumerary teeth. The most common supernumerary tooth in the maxillary midline is called a mesiodens.
question
The aluminum filter narrows the x-ray beam to a specific diameter because aluminum effectively filters (removes) low-energy, non-penetrating wavelengths from the beam.
answer
The statement is NOT correct, but the reason is correct. RATIONALE: The aluminum filter does not narrow the x-ray beam. However, its function is to remove, or filter, low-energy, non-penetrating wavelengths from the beam. Note that it is the collimator or diaphragm that controls the shape of the beam.
question
Both mast cells and basophils are associated with hypersensitivity reactions of an anaphylactic type. Neutrophils are associated with inflammatory responses to irritation.
answer
Both statements are true. RATIONALE: Anaphylactic and anaphylactoid (similar to anaphylactic) reactions result from systemic release of mediators by mast cells and basophils. Neutrophils, often called the human body's first line of defense, respond to the first phase of insult, which is called irritation. The insult may be microbial, physical, chemical, or neoplastic in origin. Unresolved insult progresses to inflammation.
question
Each of the following is a desirable characteristic of tray systems for use in professional fluoride therapy EXCEPT one. Which one is the EXCEPTION?
answer
Can be used with gel or solution fluoride formulation RATIONALE: Although fluoride tray systems are ideal for fluoride gels, solution-based fluoride formulations cannot be adequately retained within tray boundaries. Optimal fluoride coverage is unlikely when solution-based fluoride formulations are used in trays. The capacity of a fluoride tray system to adapt to facial and lingual surfaces without extreme gapping is desirable and helps ensure optimal coverage. The depth of a fluoride tray system must be sufficient to reach exposed root surfaces to ensure optimal coverage. Selecting a fluoride tray system that is comfortable for the patient ensures compliance and is likely to provide optimal treatment.
question
Fever and erythema are considered systemic signs of inflammation. Pain, swelling, and edema are considered local signs of inflammation.
answer
The first statement is false, the second is true. RATIONALE: Fever is considered a systemic sign of inflammation. However, erythema, defined as redness of the skin or mucous membranes, is considered a local sign of inflammation. Pain, swelling, and edema are considered local signs of inflammation, but all three have the potential to become systemic.
question
Internal resorption is caused by trauma, pulpal inflammation, or both. External resorption does not invade the dental pulp.
answer
The first statement is true, the second is false. RATIONALE: Trauma and pulpal inflammation/infection are two of the major causes of internal resorption. Typically detected in radiographs, internal resorption can occur in all areas of the pulp but is most common in cervical pulpal areas. External resorption may or may not invade the dental pulp space.
question
As a requirement for your graduate degree in dental hygiene, you are conducting statistical analysis of data from a community oral screening program in a local homeless shelter. Socioeconomic status is low and the incidence of homelessness is high in this community since a factory closed recently and layoffs were made in related industries. Homelessness presents many challenges in obtaining accurate research data, partly because of transportation difficulties experienced by patients and partly by the seasonal and transient nature of this population as individuals seek employment. The shelter provides a limited supply of toothpaste, toothbrushes, and dental floss. A related aspect of this assignment includes presenting a brief introduction of statistical concepts to dental hygiene students at a nearby community college. Variables along a continuum, such as temperature, scores from a survey, and time are examples of discrete data. Continuous data are considered quantitative in nature.
answer
The first statement is false, the second is true. RATIONALE: Variables along a continuum are considered continuous data; note the similarity in terminology. By contrast, discrete data have only one of a limited set of values and include hair color, gender, and political preference. Discrete data are counted only in whole numbers, whereas continuous data, quantitative in nature, may be stated in fractions or decimals.
question
Autonomy is the provider's commitment to the health and welfare of the patient above all other considerations. Fidelity is based on an individual's right to privacy.
answer
Both statements are false. RATIONALE: Autonomy, also known as personal liberty, is the freedom of an individual to make decisions regarding his or her own health. The provider's commitment to the health and welfare of the patient above all other considerations is known as beneficence. Fidelity requires that health care providers only practice within their scope of practice. An individual's right to privacy is protected by the universal principle of confidentiality.
question
As a requirement for your graduate degree in dental hygiene, you are conducting statistical analysis of data from a community oral screening program in a local homeless shelter. Socioeconomic status is low and the incidence of homelessness is high in this community since a factory closed recently and layoffs were made in related industries. Homelessness presents many challenges in obtaining accurate research data, partly because of transportation difficulties experienced by patients and partly by the seasonal and transient nature of this population as individuals seek employment. The shelter provides a limited supply of toothpaste, toothbrushes, and dental floss. A related aspect of this assignment includes presenting a brief introduction of statistical concepts to dental hygiene students at a nearby community college. The chi-square (χ2) test is an appropriate statistical tool with which to assess probability. Both the χ2 test and the t test are nonparametric inferential statistical tools used to aid in the interpretation of findings.
answer
Both statements are false. RATIONALE: The chi-square test (χ2) is used not to assess probability but instead to determine statistical significance between frequencies of data. The χ2 test is a nonparametric inferential statistical tool; the t test is a parametric inferential statistical tool. Both the χ2 and the t test are statistical tools used to aid in the interpretation of research findings.
question
A 42-year-old woman who is taking no medications and has no significant past medical history is scheduled for comprehensive dental hygiene care. Initially, she has no complaints about her health, and her vital signs are within normal limits. After you explain the procedure she will undergo and initiate the dental hygiene intervention, the woman begins to complain of "feeling faint." As this woman regains consciousness, what is MOST likely to occur?
answer
Her pulse rate and blood pressure will return to normal. RATIONALE: After this patient regains consciousness, both her pulse rate and blood pressure should return to normal and she should not feel sleepy. Note that sleepiness is associated with post-seizure patients but not with syncope (fainting). Although both hunger and agitation can cause fainting, if these conditions are not an initiating cause (as in this case), neither hunger nor agitation is likely to occur as a patient recovers consciousness.
question
Long, labored breaths and coughing occur frequently with congestive obstructive pulmonary disease because low blood sugar, a key feature of COPD, inhibits the respiratory system.
answer
NEITHER the statement NOR the reason is correct. RATIONALE: Although coughing is common in COPD (congestive obstructive pulmonary disease), shortness of breath due to respiratory distress is characteristic. Low blood sugar is not a sign or symptom of COPD. Note that other symptoms of COPD include inflammation (swelling), especially of the lower extremities and prominent jugular veins.
question
An abnormal reaction to a drug by the immune system is considered a side effect, because a side effect produces an undesirable effect from drug action on a nontarget organ.
answer
The statement is NOT correct, but the reason is correct. RATIONALE: An abnormal reaction to a drug by the immune system is considered an allergy. Allergy is defined as an altered or enhanced immune reaction. A side effect occurs when a drug acts on a nontarget organ and produces undesirable effects. A side effect is also described as an effect, whether therapeutic or adverse, that is secondary to the one intended. The term side effect can be applied to beneficial, but unintended, consequences of the use of a drug.
question
Mouth-formed athletic mouthguards are the preferred classification of protectors because they are both highly durable and comfortable, as well as the least likely classification to impair speech.
answer
NEITHER the statement NOR the reason is correct. RATIONALE: Custom-made protectors are the preferred classification of protectors because they are the most durable and comfortable and not as likely as other classifications of mouthguards to impair speech. By contrast, mouth-formed protectors ("boil-and-bite") are essentially made by the patient. Note that the fit of a "boil-and-bite" guard is better than that of a stock protector.
question
A community health clinic located in an underserved rural community has hired a dental hygienist to design and implement a program to reduce early childhood caries (ECC). Most of the population does not benefit from community fluoridated water, nor are school-based fluoridation programs available. During initial research, the dental hygienist learns that a strong coalition within the community opposes any type of fluoridation. To prioritize needs, goals and objectives the program administrator has recommended use of the SMART formula. The clinic, funded by a state agency, hopes to expand oral services to older children and adults in the next few years. The SMART formula for use in program planning for developing goals and objectives includes each of the following components EXCEPT one. Which one is the EXCEPTION?
answer
Assessable RATIONALE: SMART is acronym composed of letters that represent the ideal components of goals and objectives for a community based health program. The SMART formula does not include "assessable" as a component. Rather, it contains the component "appropriate." Under the SMART formula, the individual components of well-designed community health goals and objectives should be: Specific: based on clearly stated objectives Measurable; based on objectives that can be measured Appropriate: the needs of the population are identified Realistic or related: based on achievable goals related to anticipated outcomes Time bound: includes a timeline
question
During periodontal instrumentation, a face-to-tooth surface angulation of 40 degrees will result in
answer
burnished calculus. RATIONALE: A face-to-tooth surface angulation of 40 degrees will result in burnished calculus. The ideal face-to-tooth surface angulation for periodontal instrumentation is between 70 and 80 degrees. Please note that calculus can be removed with an angulation between 45 and 80 degrees. Surrounding tissues will not likely be damaged by a face-to-tooth surface angulation of 40 degrees, because this is the correct angulation for insertion of a curet. Root surfaces will be damaged if the face-to-tooth surface angulation is greater than 80 degrees. The angulation required for calculus removal is between 45 and 80 degrees, with most experts stating that 70 to 80 degrees is ideal.
question
Dark stains on radiographic films may result from contamination from all of the following EXCEPT one. Which one is the EXCEPTION?
answer
Fixer splash before processing RATIONALE: Contamination by fixer splash results in light or clear stains. Fluoride contamination results in dark stains on radiographic images. This is especially true of stannous fluoride. Wearing clean gloves prevents contamination of radiographic films from fluoride. Powder-free gloves are recommended because glove powder contamination results in dark stains on radiographic images. Static electricity can cause a radiographic film to develop lightning-like streaks, dark streaks, or dark smudges. These unique defects are caused by overly rapid opening of the film packet or transfer of static electricity from the user to the film. Static electricity results in dark streaks or smudges of exposed areas on the film. Contamination of the film with saliva before processing results in dark stains on the radiographic image.
question
Which of the following is NOT a bacterial virulence factor?
answer
Cytokine RATIONALE: A cytokine is a proinflammatory mediator produced by cells of the immune system that affect other cells. It is not a bacterial product. Capsules are bacterial virulence factors that resist host defenses by impairing phagocytosis. Endotoxin is a bacterial virulence factor produced by gram-negative cells. Kinase, an enzyme produced by bacteria, aids in establishing virulence. Exotoxin is a bacterial virulence factor produced by gram-positive cells.
question
You have been asked to evaluate the federally funded dental program at a local senior citizen center in a northeastern state. Located in a diverse inner-city neighborhood of a large metropolitan city, the center serves an ethnically diverse population of varying socioeconomic status. Community water supplies are fluoridated, and students from local dental hygiene programs occasionally provide oral health education and oral screenings under the supervision of a licensed dentist. Both a registered nurse and a dietitian are available for you to consult with, as are a dentist and a gerontologist. Breakfast and lunch are served five days a week, and movies, complete with soft drinks and snacks, are available most afternoons. The registered nurse confides that she suspects that numerous patients are abusing alcohol as an escape from depression and lifestyle changes associated with aging. During oral health screenings, which of the following oral conditions would you NOT associate with chronic alcohol abuse and dependence?
answer
Fordyce granules RATIONALE: Fordyce granules, also called Fordyce spots, are asymptomatic white or yellow-white papules of 1 to 3 mm in greatest dimension. They are nonpathologic and are not associated with alcohol abuse and dependence. There is no surrounding mucosal change, and the granules remain constant throughout life. The most common sites of occurrence are the buccal mucosa (often bilateral), the upper lip vermilion, and the mandibular retromolar pad and tonsillar areas, but any oral surface may be involved. Some patients will have hundreds of granules, but most have smaller numbers. Dental caries is associated with chronic alcohol abuse and dependence. Alcohol causes dehydration of the oral tissues, known as xerostomia, which is a contributing factor to dental caries. Also contributing to dental caries are nausea and vomiting resulting from alcohol consumption. Leukoplakia, a white patch or plaque that cannot be rubbed off, is associated with many systemic conditions, including chronic alcohol abuse and dependence. Attrition, a result of bruxism, is associated with chronic alcohol abuse and dependence.
question
An x-ray produced by the slowing down of an accelerating electron as it passes near the nucleus of the target atom is called
answer
Bremsstrahlung RATIONALE: Bremsstrahlung radiation, also called "breaking" radiation, is produced when the slowing- down process results in a transference of the electron's kinetic energy into x-ray energy. Characteristic radiation, also called "discrete" radiation, results from the restabilization that occurs in the target material atom after an orbiting electron is dislodged. Discrete radiation is also called "characteristic" radiation and occurs when outer shell electrons move to fill voids left by the ejection of electrons from the inner shells of the target material atoms. Particulate radiation is a type of ionizing radiation.
question
At an evaluation 6 weeks after completion of his dental hygiene treatment, your patient reports that tooth #6 is still very sensitive to cold and air. The labial class V area of this tooth has severe erosion and/or abrasion; consequently, a desensitizing agent was applied at the patient's last three visits. Which of the following methods should be considered to alleviate the dentinal hypersensitivity?
answer
Applying a composite resin RATIONALE: If dentinal hypersensitivity persists after several applications of a desensitizing agent in an area that has severe erosion or abrasion, application of a composite resin should be considered as a permanent solution. This will permanently seal the dentinal tubules and eliminate dentinal hypersensitivity. Application of a fluoride varnish may not be effective, because a desensitizing agent has already been used several times without positive results. Extracting the tooth is too radical for the symptoms. Reapplication of the same desensitizing agent after three treatments is unlikely to be effective.
question
Human immunoglobulin IgG, which responds in allergic reactions, is the most abundant of the various types in response to bacterial toxins. Human immunoglobulin IgA is found in exocrine secretions, whereas immunoglobulin IgD triggers B-cell response.
answer
The first statement is false, the second is true. RATIONALE: In response to bacterial toxins, human immunoglobulin IgG is the most abundant of the various types. Enhanced phagocytosis enables IgG acts to neutralize bacterial toxins. However, this statement in its entirety is false, because human immunoglobulin IgE responds during allergic reactions. Human immunoglobulin IgA is found in exocrine secretions such as tears and saliva, whereas immunoglobulin IgD triggers B-cell response.
question
Your treatment plan for a patient with moderate periodontitis (case type III) is likely to include each of the following therapeutic interventions EXCEPT one. Which one is the EXCEPTION?
answer
Extraction of teeth that have a poor prognosis RATIONALE: Extraction of teeth that have a poor prognosis is more likely to be an intervention recommended for a patient with advanced periodontitis (case type IV) than for a patient with moderate periodontitis (case type III). Elimination, alteration, or control of systemic diseases and other risk factors is a likely therapeutic intervention for a patient with moderate periodontitis (case type III). Oral self-care education is an integral therapeutic treatment planning component for patients with moderate periodontitis (case type III). Supragingival and subgingival debridement, including scaling and root planing with a sextant or quadrant approach, is an integral treatment-planning component for patients with moderate periodontitis (case type III).
question
Your next patient, Mrs. Smith, an African-American woman who is 50 years old, has been waiting for a kidney transplant for 9 months. Three times a week, she must undergo kidney dialysis as a lifesaving therapy. She takes nifedipine (Procardia) for hypertension and Wellbutrin (bupropion) for depression. Her chief complaint is "My gums look big, and they keep bleeding." Which of the following situations is most likely NOT an issue in planning a dental appointment for Mrs. Smith?
answer
Seizures RATIONALE: Seizures are not directly associated with end-stage renal disease. Hypertension is a clinical manifestation of impaired renal function and will most likely be an issue in the planning of Mrs. Smith's treatment. Bleeding tendencies are likely of concern, because heparinization during hemodialysis involves blood-thinning agents. Your treatment plan for Mrs. Smith will most likely address prophylactic antibiotic premedication; many physicians recommend this precaution because of the AV shunt in dialysis patients. Drug interactions and the ability of the patient to clear drugs are factors in treatment planning for patients with renal dysfunction of any level.
question
Placing a patient who is taking digoxin (Lanoxin) and a diuretic for congestive heart failure in a horizontal position will make breathing more comfortable but may induce lightheadedness. Placing a patient who is taking digoxin (Lanoxin) and a diuretic for congestive heart failure in a horizontal position may increase ankle edema.
answer
The first statement is false, the second is true. RATIONALE: For dental hygiene therapy, patients with CHF will breathe more easily when placed in a more upright position. A horizontal position is likely to increase fluid retention and edema around the heart and lungs. For this reason, persons with congestive heart failure (CHF) sleep with extra pillows or sitting upright in a chair. Note that lightheadedness should not occur when a person is in a horizontal position. A patient with CHF should not be placed in a horizontal position, because this may increase edema in the extremities, especially the lower extremities. A more upright position is recommended.
question
Viruses that persist in the body and cause recurrent disease are called
answer
latent RATIONALE: Latent viruses are capable of persisting in the body for long periods and causing recurrent disease. A disease is said to be of latent nature when it is caused by a latent virus. Resistant refers to the ability of an organism to ward off the host defenses or antibiotics. Cytopathic refers to the ability of a virus to cause cells to deteriorate. Opportunistic infections occur when an organism that does not usually cause disease becomes pathogenic. Oncogenic viruses are those capable of causing tumors.
question
Neuroglia, specialized tissue cells that support neurons, conduct nerve impulses away from the cell body. Neuroglia produce the myelin sheath around the axons of the central nervous system (CNS).
answer
The first statement is false, the second is true. RATIONALE: Neuroglia are specialized tissue cells that support neurons; however, nerve impulses are conducted away from the cell body by axons. Neuroglia not only produce the myelin sheath around the axons of the central nervous system (CNS), but also attach neurons to blood vessels and carry out phagocytosis.
question
Infrequent dental visits and emotional stress are risk factors for necrotizing ulcerative gingivitis (NUG). Leukemia, age, and gender are risk factors for NUG.
answer
Both statements are false. RATIONALE: Although emotional stress is a risk factor for necrotizing ulcerative gingivitis (NUG), infrequency of dental visits is not a risk factor for NUG. However, infrequent dental visits increase the risk of periodontitis. Leukemia, a systemic disease risk factor associated with periodontitis, is not a risk factor for NUG. Neither age nor gender is a risk factor for necrotizing ulcerative periodontitis.
question
You have been asked to evaluate the federally funded dental program at a local senior citizen center in a northeastern state. Located in a diverse inner-city neighborhood of a large metropolitan city, the center serves an ethnically diverse population of varying socioeconomic status. Community water supplies are fluoridated, and students from local dental hygiene programs occasionally provide oral health education and oral screenings under the supervision of a licensed dentist. Both a registered nurse and a dietitian are available for you to consult with, as are a dentist and a gerontologist. Breakfast and lunch are served five days a week, and movies, complete with soft drinks and snacks, are available most afternoons. A strategy to replace the sodas served during afternoon movies with less cariogenic beverages is an initial health promotion goal. Soda consumption contributes to each of the following EXCEPT one. Which one is the EXCEPTION?
answer
Increased salivary pH RATIONALE: Soda, containing 10 to 12 tsp of sugar per 12-ounce serving, decreases salivary pH. As the pH transitions from a neutral (healthy) acid/base balance, the oral environment becomes more acidic (lower pH) and conducive to microbial proliferation and disease. The sugar and resulting acidity triggered by soda consumption contribute to xerostomia, or dry mouth. The high sugar content in soda shifts the oral pH to a more acidic environment (lower pH), which increases demineralization of teeth. The high sugar content in soda provides nutrition for the microbes that initiate and perpetuate gingivitis.
question
A peripheral giant cell granuloma usually arises from the periodontal ligament and is characterized histologically by the presence of multinucleated giant cells. Osteoblasts and remnants of enamel and cementum are typical components of peripheral giant cell granulomas.
answer
The first statement is true, the second is false. RATIONALE: Peripheral giant cell granulomas, typically arising from the periodontal ligament, are commonly characterized by the presence of multinucleated giant cells. Osteoblasts are bone-forming cells and are not a component of peripheral giant cell granulomas. Remnants of enamel and cementum are not components of peripheral giant cell granulomas.
question
Consumer and industrial products are the major contributor of ionizing radiation to our population because radiation produced in these industries account for more than 80% of the ionizing radiation to which our population is exposed.
answer
NEITHER the statement NOR the reason is correct. RATIONALE: Consumer and industrial products are not the major contributor of ionizing radiation to our population. In fact, consumer and industrial products account for only a small percentage of the 18% of exposure that comes from artificial sources. Likewise, production of nuclear energy contributes only a small percentage of the 18% coming from artificial sources. Note that the major contribution to our population's exposure to ionizing radiation arises from naturally occurring radionuclides in the soil, as well as radiation from cosmic energy. More than 80% of the exposure to ionizing radiation comes from natural sources. Especially pertinent to dental hygienists, the healing arts, including dental x-rays, account for 11% of the total exposure to ionizing radiation.
question
A dental composite that is described as "flowable" is within which classification of filler content?
answer
Moderate RATIONALE: Composite resins that are described as flowable are low-viscosity light-cured resins that are classified in the moderate range of filler content. The filler content of modern hybrid-type resin-based composites usually varies between 60 and 70 volume percent (vol. %) or 70 and 85 weight percent (wt. %). Although composite resins that are described as very high, high, and low are not considered flowable, they have other valuable attributes. A dental composite classified as very high in filler content is described as "packable," whereas a dental composite classified as high in filler content is described as "hybrid" or "all-purpose." Along the spectrum of filler content, a dental composite classified as low in filler content is described as "microfill."
question
Which drug prescribed for hypertension has been found to cause the LEAST amount of gingival connective tissue enlargement?
answer
Isradipine RATIONALE: Gingival overgrowth represents an overexuberant response to a variety of local and systemic conditions. Certain anticonvulsants and immunosuppressive drugs and a number of calcium channel blockers have been shown to produce similar gingival overgrowth in certain susceptible patients. Among the alternatives, the calcium channel-blocking medication isradipine, also known as DynaCirc, has been found to cause the LEAST amount of gingival connective tissue enlargement. Although the calcium channel blocker Norvasc (generic complex is named amlodipine) causes more tissue response than does isradipine (DynaCirc), it does not cause the overgrowth associated with nifedipine. Note that nifedipine (generic name for Procardia), is also a calcium channel blocker. Of the medications listed, Procardia causes the MOST gingival connective tissue proliferation. Nitroglycerine, a nitrate prescribed to control angina pectoris, is taken only on an emergency basis and is not associated with gingival connective tissue enlargement.
question
A woman in her last trimester of pregnancy becomes unresponsive during dental hygiene care. No spontaneous respirations are detected, and attempts to ventilate her are unsuccessful. The following question pertains to this scenario. Once this patient regains spontaneous respirations, it will be important to roll her toward her right side to improve circulation because this action decreases compression by the uterus on the aorta and vena cava.
answer
The statement is NOT correct, but the reason is correct. RATIONALE: The patient should be rolled onto her left side. This decreases compression by the uterus of both the aorta and vena cava. This action will increase cardiac output. Conversely, rolling the patient toward her right side will decrease cardiac output.
question
A community health clinic located in an underserved rural community has hired a dental hygienist to design and implement a program to reduce early childhood caries (ECC). Most of the population does not benefit from community fluoridated water, nor are school-based fluoridation programs available. During initial research, the dental hygienist learns that a strong coalition within the community opposes any type of fluoridation. To prioritize needs, goals and objectives the program administrator has recommended use of the SMART formula. The clinic, funded by a state agency, hopes to expand oral services to older children and adults in the next few years. Tactics used by anti-fluoridationists include each of the following EXCEPT one. Which one is the EXCEPTION?
answer
Promoting reliance on governmental intervention RATIONALE: Anti-fluoridationists do not promote reliance on governmental intervention but instead create suspicion of government programs, officials, or both. Claiming that fluoridation causes fluorosis is a tactic widely used by anti-fluoridationists. Implying that water fluoridation is poisonous and causes health hazards such as allergies, cancer, heart disease, increased death rates, and Alzheimer's disease is a tactic used by anti-fluoridationists. Emotion and scare tactics are widely used by anti-fluoridationists to gain support for their opinions.
question
Of which hypersensitivity reaction is a positive tuberculin skin test an example?
answer
Type IV RATIONALE: Of the four categories of reactions, only a Type IV hypersensitivity reaction is an indication of a positive tuberculin skin test. A Type I hypersensitivity reaction is positive for the potentially life-threatening allergic reaction known as anaphylaxis. A Type II hypersensitivity reaction indicates the presence cytotoxic reaction such as a poison, drug overdose, or food poisoning. A Type III hypersensitivity reaction occurs when there is an excess of antigen, creating small immune complexes that are not easily cleared from the circulation. This response frequently induces inflammation.
question
Your next patient, Mrs. Smith, an African-American woman who is 50 years old, has been waiting for a kidney transplant for 9 months. Three times a week, she must undergo kidney dialysis as a lifesaving therapy. She takes nifedipine (Procardia) for hypertension and Wellbutrin (bupropion) for depression. Her chief complaint is "My gums look big, and they keep bleeding." Hemodialysis puts Mrs. Smith at risk for hepatitis B, hepatitis C, hepatitis D, and HIV infection. Hemodialysis puts Mrs. Smith at risk for hepatitis A, hepatitis B, hepatitis E, and hemophilia.
answer
The first statement is true, the second is false. RATIONALE: The hepatitis B, hepatitis C, and hepatitis D viruses can be transmitted in blood and blood-contaminated items. The same is true of HIV. Hepatitis A is transmitted in contaminated food and water. Hepatitis E is transmitted in contaminated water. Note that although bleeding problems in patients with hemophilia may be exacerbated by hemodialysis, hemophilia does not develop as a consequence of hemodialysis.
question
Informed consent is basic to the universal principle of autonomy. Informed refusal is basic to the universal principle of veracity.
answer
The first statement is true, the second is false. RATIONALE: Both informed consent and informed refusal are basic to the universal principle of autonomy. Respect for the individual autonomy of others is basic to the health care provider-patient relationship. Autonomy represents personal liberty and the concept that individuals are free to make decisions regarding their own health. Informed refusal is not basic to veracity; rather, informed consent is basic to the universal principle of autonomy. Veracity, another universal principle of ethics, is truthfulness and applies mutually to both the health care provider and the patient.
question
Which of the following radiographic projections would be MOST helpful in imaging the maxillary sinus?
answer
Water's projection RATIONALE: Water's projection is used to image the maxillary sinuses. The cephalometric projection is better used for orthodontic and facial reconstruction. The lateral oblique mandible projection images the mandible from the canine posteriorly to the body and ramus. The posterior-anterior projection is better used to image changes in the cranial bones. The transcranial projection is better used to image the temporomandibular joint.
question
The malleus, incus, and stapes
answer
are ear ossicles. RATIONALE: The ear ossicles comprise three tiny bones—the malleus, incus, and stapes. Located in the middle ear, the ossicles are not associated with linear acceleration. The ossicles are involved in vibrations of the tympanic membrane, also known as the eardrum. It is not the ossicles that maintain equilibrium but rather the semicircular canals.
question
Different classes of diuretics have targeted activity in major sites in the kidney. Which pair accurately expresses the class of diuretic with its major site of action?
answer
Class: potassium-sparing diuretics/site: distal convoluted tube RATIONALE: The major site of action for potassium-sparing diuretics is the distal convoluted tube. Note that spironolactone is an example of a potassium-sparing diuretic that has a major effect on the distal convoluted tubule. Carbonic anhydrase inhibitors affect the proximal tubules. Loop diuretics affect the thick segments of ascending limbs. Thiazides affect the distal convoluted tubules.
question
Fluoride-releasing restorative materials should not be used in Class V restorations because neither glass ionomers nor resin-modified glass ionomers area appropriate for incisal edges of teeth.
answer
NEITHER the statement NOR the reason is correct. RATIONALE: Fluoride-releasing restorative materials are frequently used in Class V restorations and are especially beneficial in geriatric dentistry. Note that Class V restorations are located in/on the cervical area of a tooth. By contrast, Class III and Class IV restorations involve the incisal edges of teeth.
question
Endogenous factors influencing the microbial composition of the oral flora include all of the following EXCEPT one. Which one is the EXCEPTION?
answer
Diet RATIONALE: Diet is an exogenous factor because dietary sources, like all exogenous sources, originate outside the body. By contrast, an endogenous factor is one that originates within the body. Oral pH, a measure of the acidity level within the oral environment, fits the definition of an endogenous factor. Saliva is also an endogenous factor, because it originates within the salivary glands. Oxygen concentration, also endogenous, represents the concentration of oxygen in a blood sample. Similarly, the biochemistry involved in oral microbial interactions is considered endogenous in origin.
question
Controlled drug delivery with minocycline hydrochloride (1 mg) therapy is recommended during periodontal flap surgery because this controlled antibiotic delivery system is effective in reducing periodontal pathogens, inflammation, and probing depths in sites that have not responded to initial therapy.
answer
The statement is NOT correct, but the reason is correct. RATIONALE: Although local delivery of a controlled slow-release antibiotic such as minocycline hydrochloride is considered effective in reducing periodontal pathogens, inflammation, and probing depths in sites that have not responded to initial therapy, local delivery is not recommended during periodontal flap surgery. However, when administered before periodontal surgery this agent is thought to precondition the tissue and decrease the severity of infection.
question
Your next patient, Mrs. Smith, an African-American woman who is 50 years old, has been waiting for a kidney transplant for 9 months. Three times a week, she must undergo kidney dialysis as a lifesaving therapy. She takes nifedipine (Procardia) for hypertension and Wellbutrin (bupropion) for depression. Her chief complaint is "My gums look big, and they keep bleeding." In light of her current health, it is likely that Mrs. Smith's gums appear large and tend to bleed because inflammation, accumulation of urea, and water retention are common feature of end-stage renal disease.
answer
Both the statement and reason are correct but NOT related. RATIONALE: Mrs. Smith's gums appear large and continue to bleed as a result of drug-induced gingival enlargement (hypertrophy). This is a side effect of the nifedipine (Procardia) that she takes for hypertension. A inflammation, accumulation of urea, and water retention cause serious consequences in end-stage renal disease, these conditions are not the cause of the gingival enlargement. Again, gingival enlargement (hypertrophy) is a side effect of the antihypertensive medication nifedipine.
question
Essential oil mouthrinses can effectively replace mechanical plaque biofilm control. Oxygenating agents may be added to mouthrinses to debride and release oxygen.
answer
The first statement is false, the second is true. RATIONALE: Mouthrinses, including essential oil-based ones, are recommended as adjuncts to, but not as replacements for, mechanical plaque biofilm control. Numerous agents are added to commercially prepared mouthrinses, including oxygenating agents (hydrogen peroxide) that debride and release oxygen.
question
All of the following conditions are linked to an oral manifestation EXCEPT one. Which one is the EXCEPTION?
answer
Spina bifida RATIONALE: Spina bifida, a birth defect involving incomplete closure of the backbone and spinal cord, has no associated oral manifestations. Bulimia causes multiple oral problems, including chronic sore throat, parotid gland swelling, xerostomia, burning tongue, dentinal hypersensitivity, and perimyolysis (decalcification of the teeth from exposure to gastric acid). Sickle cell anemia creates multiple oral problems, including loss of bone trabeculations, delayed eruption, and dental pain. Down syndrome is a craniofacial syndrome with multiple oral manifestations. Leukemia causes oral problems, such as infections, mucosal pallor, spontaneous gingival bleeding, petechiae, loss of lamina dura, resorption of alveolar bone, and mucositis.
question
A health care provider who reports a case of suspected child abuse without obtaining parental consent is in compliance with the law because the principle of beneficence requires health care providers to place the health and welfare of the patient above all other considerations.
answer
Both the statement and reason are correct and related. RATIONALE: This question illustrates ethical and legal dilemmas that may arise. In this case, the health care provider is acting in compliance with the law because of an exception to the law. The key is, although health care providers are ethically and legally bound to keep confidential all patient communications and records, exceptions are made for cases of suspected child abuse and other specific situations. Beneficence, the universal ethical principle mandating that the health and welfare of the patient be placed above all other considerations, is the driving principle of this ethical situation.
question
Modified zinc oxide-eugenol periodontal surgical dressings contain all of the following substances EXCEPT one. Which one is the EXCEPTION?
answer
Methylmethacrylate RATIONALE: Methylmethacrylate is a substance utilized in the fabrication of acrylic appliances such as orthodontic space maintainers. Tannic acid, rosin, and various oils are components of zinc oxide-eugenol surgical dressings.
question
A radicular cyst is most frequently located at the apex of the root and is most likely associated with supernumerary teeth. A dentigerous cyst is most likely associated with carious lesions.
answer
Both statements are false. RATIONALE: Radicular cysts, usually found at the apex of the root, are most likely associated with carious teeth. A radicular cyst is not commonly seen with a supernumerary tooth, unless the tooth is erupted and a carious lesion is present. Dentigerous cysts are most likely associated with unerupted teeth. Note that in impacted third molars, a dentigerous cyst is most likely to form in the area surrounding the crown of an unerupted tooth.
question
The blood pressure may be artificially increased if the cuff is too small, because the stethoscope cannot be properly placed over the carotid artery.
answer
The statement is correct, but the reason is NOT. RATIONALE: If the cuff is too small, and less than 80% of the arm is covered by the air bladder, the cuff may not be able to completely close off the blood vessels. In this case, the bladder will inflate too much and the monitor will give a higher reading than it should. The carotid arteries are the major blood vessels in the neck that supply blood to the brain, neck, and face. The stethoscope should be placed over the brachial artery. Conversely, a cuff that is too large may result in an artificially low blood pressure reading.
question
Drugs that produce adverse effect on a fetus are considered
answer
teratogenic RATIONALE: Teratogenic drugs produce adverse effects on a fetus. For example, alcohol intake during pregnancy results in fetal alcohol syndrome. Idiosyncratic is a term used to describe an abnormal drug response that is unpredictable and thought to be genetically related. Benzodiazepines are sedative-hypnotic medications used to treat anxiety disorder, situational anxiety, and insomnia. Commonly prescribed benzodiazepines include Xanax, Versed, and Valium. Aspirin products are salicylates.
question
Sudden momentary loss of awareness without loss of postural tone, accompanied by a blank stare lasting less than 90 seconds, BEST describes a
answer
nonconvulsive-absence (petit mal) seizure. RATIONALE: Nonconvulsive-absence seizures (known as petit mal) are characterized by sudden momentary loss of awareness without loss of postural tone. This condition is accompanied by a blank stare lasting less than 90 seconds. A grand mal seizure, formally called a generalized tonic-clonic seizure, usually lasts 1 to 3 minutes and involves both continuous contraction of voluntary muscles (tonic movements) and intermittent muscular contraction and relaxation (clonic movements). A complex partial (psychomotor) seizure involves a transient clouding of consciousness, behavioral alterations, and purposeless, repetitive, and stereotypical movements of actions. A stroke, also known as a cerebrovascular disease, is defined as the sudden loss of brain function resulting from interference with the blood supply to a portion of the brain. Signs and symptoms of a stroke include syncope, headache, altered consciousness, paralysis, speech problems, and alterations in reflexes.
question
As a requirement for your graduate degree in dental hygiene, you are conducting statistical analysis of data from a community oral screening program in a local homeless shelter. Socioeconomic status is low and the incidence of homelessness is high in this community since a factory closed recently and layoffs were made in related industries. Homelessness presents many challenges in obtaining accurate research data, partly because of transportation difficulties experienced by patients and partly by the seasonal and transient nature of this population as individuals seek employment. The shelter provides a limited supply of toothpaste, toothbrushes, and dental floss. A related aspect of this assignment includes presenting a brief introduction of statistical concepts to dental hygiene students at a nearby community college. Which health education and promotion theory is based on the underlying principle that accurate information enables better health choices?
answer
Health Belief Model RATIONALE: The major premise of the Health Belief Model (HBM) is that individuals need accurate information to make better health choices. The Transtheoretical Model (Stages of Change) recognizes that a state of readiness is required to change; this model follows an orderly process that progresses through predictable stages. The Theory of Reasoned Action posits that people make rational decisions based on knowledge, values, and attitudes. Social Cognitive Theory (SCT) identifies knowledge, behavior, and environment acting in a reciprocal (circular) manner to continually affect one another.
question
Atrioventricular (AV) valves are thin one-way structures composed of endocardium and located between the atrium and the ventricles. The right atrioventricular valve is also called the tricuspid valve.
answer
Both statements are true. RATIONALE: The atrioventricular (AV) valves are thin structures of endocardium that separate the atria from the ventricles. There is both a right and a left atrioventricular valve. These valves ensure that blood continues to flow in one direction only. The right AV valve is also called the tricuspid valve. Note that the left AV valve is called the mitral valve.
question
Hydrogen peroxide, an over-the-counter oxidizing agent, is safe for daily use. Oxygenating agents decrease clinical signs of inflammation.
answer
The first statement is false, the second is true. RATIONALE: Although it is an over-the-counter (OTC) oxidizing agent, hydrogen peroxide is not considered safe for daily use. Safety issues such as tissue injury and co-carcinogenicity have been raised with long-term use of hydrogen peroxide. It is true that oxidizing agents decrease clinical signs of inflammation, but it should be noted that bacterial pathogens may not be reduced.
question
As a requirement for your graduate degree in dental hygiene, you are conducting statistical analysis of data from a community oral screening program in a local homeless shelter. Socioeconomic status is low and the incidence of homelessness is high in this community since a factory closed recently and layoffs were made in related industries. Homelessness presents many challenges in obtaining accurate research data, partly because of transportation difficulties experienced by patients and partly by the seasonal and transient nature of this population as individuals seek employment. The shelter provides a limited supply of toothpaste, toothbrushes, and dental floss. A related aspect of this assignment includes presenting a brief introduction of statistical concepts to dental hygiene students at a nearby community college. Community-based type II examinations and inspections include thorough radiographic surveys and pulp vitality tests. Community-based type III examinations and inspections include posterior bitewing radiographs and, when indicated, periapical radiographs.
answer
Both statements are false. RATIONALE: Community-based type II examinations and inspections are considered limited in scope. They include the use of a mouth mirror, explorer, adequate illumination, and posterior bitewings, as well as periapical radiographs when indicated. Thorough radiographic surveys and pulp vitality tests are components of type I examinations and inspections. Community-based type III examinations and inspections involve the use of a mouth mirror, explorer, and adequate illumination only.
question
Your next patient, Mrs. Smith, an African-American woman who is 50 years old, has been waiting for a kidney transplant for 9 months. Three times a week, she must undergo kidney dialysis as a lifesaving therapy. She takes nifedipine (Procardia) for hypertension and Wellbutrin (bupropion) for depression. Her chief complaint is "My gums look big, and they keep bleeding." It is important to schedule dental or dental hygiene treatment
answer
the day after dialysis. RATIONALE: The optimal scheduling for dental or dental hygiene treatment in a dialysis patient is the day after the most recent dialysis treatment. This assumes that this is a non-dialysis day. This scheduling ensures the absence of circulating heparin, which has a half-life of about 4 hours. Scheduling dental hygiene treatment on the day of dialysis is contraindicated because heparin remaining in the bloodstream would increase bleeding. Dental or dental hygiene treatment immediately before dialysis is not advisable. It is not possible to schedule dental hygiene treatment to coincide with dialysis; dental hygiene care cannot be administered at a dialysis center. Intentionally scheduling dental hygiene treatment while heparin is present in the bloodstream is contraindicated because of increased bleeding.
question
Penicillin blood level peaks between 45 minutes and 2 hours when the drug is administered orally or intramuscularly. When penicillin is administered intravenously, the level peaks within 10 to 20 minutes.
answer
Both statements are false. RATIONALE: When penicillin is administered orally or intramuscularly, its blood level peaks between 30 minutes and 1 hour. When administered intravenously, the penicillin level peaks immediately.
question
Each of the following accurately describes an aspect of professionally applied fluoride varnish EXCEPT one. Which one is the EXCEPTION?
answer
Has thixotropic characteristics RATIONALE: Thixotrophic is a term that describes characteristics of some fluoride gels, however it is not a characteristic of fluoride varnish. The chemistry of a thixotropic formulation converts a gel to liquid when it is placed in the mouth, enabling the fluoride ion to more readily flow into confined spaces within an application tray. Professionally applied fluoride varnish contains a 5% sodium fluoride (NaF) concentration and has FDA approval for use as a desensitizing agent and cavity liner. Although not approved as a decay prevention agent, professionally applied fluoride varnish (NAF 5%) is retained on the tooth surfaces for 3 to 4 hours and maintains a fluoride reaction with the underlying enamel.
question
Michael, a 12-year-old Asian adolescent, is scheduled for his first dental hygiene care appointment and dental examination in 5 years. His mother reports that he is generally in good health but takes Adderall XR 30 mg/day for attention deficit hyperactivity disorder and still exhibits clumsiness, irritability, and an inability to focus for even short periods. He is not doing well in school, and the teachers have recommended that he be held back a year. His mother also reports that the "goose egg" on his forehead was sustained while he was rough housing under the kitchen table. She says that he is always getting bumps and bruises because of his inability to control his behavior. All of the following statements are true EXCEPT for one. Which one is the EXCEPTION?
answer
The parents do not have a legal right to ask that services be provided in their local school system. RATIONALE: National law states that parents have a right to seek free public education, inclusive of specialized education and support services, for a child with an identified disability. Attending school in the local community is an example of mainstreaming. ADHD qualifies this patient for special education purposes. All public buildings must be accessible to people with disabilities. ADHD is an example of educational labeling for the purpose of acquiring services.
question
While performing root planing, you encounter a through-and-through furcation involvement with loss of some bone between roots but not complete communication from one surface to another. This is a
answer
class II furcation. RATIONALE: A Class II furcation occurs when there is some loss of bone between roots but there is not complete communication from one surface to another. A Class I furcation involves exposure of an area of furcation with bone remaining between the roots. In a Class III furcation, through-and-through involvement with complete loss of bone between the roots is present. Note that the opening of a Class III furcation is covered by gingiva. A Class IV furcation exhibits through-and-through involvement as a result of complete loss of bone between roots and a clearly visible entrance.
question
If a person is taking a beta-blocker antagonist (β-adrenergic blocker) such as propranolol (Inderal) or atenolol (Tenormin), tachycardia would be unlikely because the blood-thinning ability of beta-blocking agents makes an increased pulse rate unlikely.
answer
The statement is correct, but the reason is NOT. RATIONALE: A person taking a beta-blocker antagonist would be unlikely to experience an increased heart rate, called tachycardia. The ability of beta-blockers to block the effects of the hormone epinephrine, also known as adrenaline, prevents the heart rate from increasing. As the heart beats more slowly and with less force, blood pressure is reduced. Note that beta-blockers do not have anticoagulant capability; they do not thin the blood.
question
Michael, a 12-year-old Asian adolescent, is scheduled for his first dental hygiene care appointment and dental examination in 5 years. His mother reports that he is generally in good health but that he takes Adderall XR 30 mg/day for attention deficit hyperactivity disorder and still exhibits clumsiness, irritability, and an inability to focus for even short periods. He is not doing well in school, and the teachers have recommended that he be held back a year. His mother also reports that the "goose egg" on his forehead was sustained while he was roughhousing under the kitchen table. She says that he is always getting bumps and bruises because of his inability to control his behavior. There are no oral manifestations directly associated with attention deficit hyperactivity disorder (ADHD), because early gingivitis is associated with poor oral hygiene.
answer
Both the statement and reason are correct but NOT related. RATIONALE: Although both the statement and the reason are correct, they are not related. It is clearly true that early gingivitis is associated with poor oral hygiene, but there is no documented evidence of oral manifestations of attention deficit hyperactivity disorder (ADHD).
question
The therapeutic goal of nonsurgical periodontal therapy (NSPT) is to establish gingival health through the elimination of causative factors. Oral prophylaxis is performed in patients with slight to moderate loss of periodontal support.
answer
Both statements are false. RATIONALE: Establishing gingival health through the elimination of causative factors is the therapeutic goal of prophylaxis. By contrast, nonsurgical periodontal therapy (NSPT) strives to alter or eliminate periodontal pathogens, address contributing risk factors, and arrest progression of periodontitis. Patients with slight to moderate loss of periodontal support do not benefit from prophylaxis; nonsurgical periodontal therapy should be recommended instead.
question
Immunity acquired by vaccination is BEST classified as
answer
artificial active. RATIONALE: Immunity by vaccination results from the host's immune system actively responding to an artificially induced infection, whereas actual infection with a pathogen is categorized as natural active immunity. As with active immunity, immunity that is considered passive can be natural or artificial. For example, natural passive immunity results from the passage of antibodies from mother to baby. As its name implies, artificial passive immunity results from an injection of an immune serum.
question
A 69-year-old man is a new patient to your practice. He states that he has a heart condition but has never had a heart attack. He periodically experiences swelling of the lower legs and is taking a medication called digoxin (Lanoxin). He is also taking a "water pill." His vital signs are normal, but you do notice some edema around the ankles. Questions 43 through 45 refer to this situation. The "water pill" he is referring to is probably
answer
a diuretic agent. RATIONALE: Diuretics are helpful in controlling fluid retention and edema. This patient's condition is most likely congestive heart failure (CHF). Digoxin (Lanoxin) is a diuretic that is commonly prescribed for patients with congestive heart failure. A beta-blocking agent, prescribed to maintain a low heart rate, is not usually used in CHF. Rather, calcium channel-blocking agents are often used in CHF, but they are not diuretics. Angiotensin-converting enzyme (ACE) inhibitors are high blood pressure drugs that widen or dilate the blood vessels to improve blood flow to the heart. Although they are sometimes used in CHF, ACE inhibitors are not diuretics.
question
The corpus luteum is maintained for the first 10 weeks of pregnancy by
answer
human chorionic gonadotropin (hCG) RATIONALE: The corpus luteum is protected and maintained during the first 10 weeks of pregnancy by human chorionic gonadotropin (hCG). This hormone is secreted by trophoblast cells during the first trimester of pregnancy. Luteinizing hormone (LH), secreted from the anterior pituitary, is not responsible for maintaining the corpus luteum. Estradiol, a form of the female hormone estrogen, is secreted by the granulosa cells as follicles grow. Although these substances are essential during pregnancy, it is hCG that maintains the corpus luteum during the first 10 weeks of pregnancy. Progesterone, which is negligible in the blood before ovulation and rises rapidly to reach a peak during the luteal phase, is essential to a healthy pregnancy. However, the corpus luteum is maintained during the first 10 weeks by hCG.
question
While making a dental hygiene diagnosis on a patient in whom you suspect aggressive periodontitis, you consider that all of the following are likely features EXCEPT one. Which one is the EXCEPTION?
answer
Occurs in individuals with compromised health RATIONALE: Aggressive periodontitis occurs in persons who are otherwise healthy. Rapid attachment loss and bone loss (which may be self-limiting) are distinguishing features of aggressive periodontitis. A common feature of aggressive periodontitis is its tendency to aggregate (occur) within families. Note that familial aggregation is not the same as hereditary etiology. Aggressive periodontitis may be localized or generalized. Generalized aggressive periodontitis is most common before age 30 but can also occur in older persons.
question
A radiolucent area in the mandibular anterior region of a 33-year-old black woman should be suspected of being a traumatic mandibular cyst or a median mandibular cyst because both of these lesions are common and have race, age, and gender predilections.
answer
NEITHER the statement NOR the reason is correct. RATIONALE: A radiolucent area in the mandibular anterior region of a 33-year-old black woman should be suspected of being a periapical cemental dysplasia because of the characteristic historical and radiographic features of this lesion. Together with this patient's demographic features, the radiographic findings define the diagnosis for periapical cemental dysplasia. Note that a median mandibular cyst is a rare lesion that is considered odontogenic in origin. Neither this cyst nor traumatic bone cyst has a race, gender, or age predilection.
question
The main subdivisions of the autonomic nervous system are the sympathetic and parasympathetic nervous systems. The autonomic nervous system operates without conscious control.
answer
Both statements are true. RATIONALE: The sympathetic and parasympathetic nervous systems are the two main subdivisions of the autonomic nervous systems. Note that the autonomic nervous system is part of the peripheral nervous system. The autonomic nervous system operates without conscious control and carries on automatically.
question
Under civil law, a health care provider can elect to terminate the patient-practitioner relationship after giving the patient notice and an opportunity to secure an alternative source of future treatment. Contractual responsibilities of the health care provider include the use of standard drugs and materials, as well as the option for use of off-label treatment options.
answer
The first statement is true, the second is false. RATIONALE: Civil law defines mechanisms that enable a health care provider to terminate the patient-practitioner relationship. They include giving the patient notice of the intent to terminate the relationship and providing the patient with an opportunity to secure alternative treatment sources. Contractual responsibilities of the health care provider do NOT include providing an option for use of off-label treatment options. (An example of off-label use of a product is the use of a local anesthetic to irrigate a wound site.) Note that the contractual responsibilities of health care providers include use of standard drugs, materials, and techniques.
question
Each of the following is a major ethical theory EXCEPT one. Which one is the EXCEPTION?
answer
Nonmaleficence RATIONALE: Nonmaleficence is not a major ethical theory. Rather, it is a universal principle mandating that the provider not use treatment to injure or wrong a patient. Nonmaleficence is often referred to as "inflict no harm." Three major ethical theories are the teleologic theory (also known as utilitarian theory), deontologic ethics, and virtue ethics.
question
A medication that is contraindicated for this pregnant woman is A. lidocaine. B. erythromycin. C. amoxicillin. D. tetracycline.
answer
tetracycline. RATIONALE: Use of the broad-spectrum antibiotic tetracycline to control this client's acute gingival infection is contraindicated, because it may cause discoloration of the deciduous and/or permanent teeth of her unborn child. This effect occurs during mineralization of the primary teeth, beginning around 4 months' gestation. The effect of tetracycline on the permanent teeth occurs near or after birth. Although it is best to avoid unnecessary medications during pregnancy, the local anesthetic lidocaine is considered safe, as are the antibiotics erythromycin and amoxicillin.
question
The second trimester of pregnancy is considered the safest time in which to perform elective dental procedures because gingival conditions are generally more stable during this time period.
answer
The statement is correct, but the reason is NOT. RATIONALE: Dental treatment during the second trimester is the best time to perform elective dental procedures. During this time, the client is typically past the stage of morning sickness and the fetus is no longer highly susceptible to injuries and malformations. The third trimester is not advisable, because prolonged chair time can cause supine hypotension. It is also likely that the client will be generally uncomfortable during the third trimester. Note that although the second trimester is the safest and most comfortable time for the client, dental treatment can be performed at any time during pregnancy if necessary. Most sources believe that pregnancy gingivitis is at its worst during the second and third trimesters.
question
This client's gingival condition is BEST described as
answer
acute/chronic gingivitis. RATIONALE: Although pregnancy may have triggered the severe gingival inflammatory response, the presence of heavy supragingival and subgingival deposits confirms that she has not received dental care for quite some time. This confirms the presence of acute/chronic gingivitis. Necrotizing ulcerative gingivitis is not present; the telltale signs of loss of interdental papilla and acute pain are absent. Although increased hormone levels may have worsened the severity of the gingival response, generalized heavy calculus indicates that this condition existed before the pregnancy and therefore does not represent pregnancy-associated gingivitis. The radiographically apparent horizontal bone loss is associated not with an acute disease but rather with a chronic condition.
question
All of the following should be included in the dental hygiene care plan for this client EXCEPT one. Which one is the EXCEPTION?
answer
Use of disclosing solution to show client areas she is missing RATIONALE: Use of disclosing solution should not be used as an educational tool in this case, because the client is visually impaired. It may, however, be used for the clinician to gather information. Fluoride therapy will be beneficial, because the client has numerous areas of recession and dentinal hypersensitivity. Review of both toothbrushing and flossing techniques is highly indicated for inclusion in the dental hygiene care plan for every client.
question
In the photos, the brownish area on the facial marginal gingiva of #22 is MOST likely
answer
normal melanin pigmentation. RATIONALE: Melanin is the pigment that gives color to the skin, eyes, hair, mucosa, and gingiva. It is hereditary and developmental and is most commonly observed in a variety of forms in people of color. The tissue does not look cyanotic, as it would if subgingival calculus were present. An amalgam tattoo is unlikely, because there are no amalgam restorations in this client's mouth. However, she is missing several teeth, which could have had amalgam restorations. Regardless, the most likely explanation for the brownish area of concern is normal melanin pigmentation. The brownish area cannot be dental caries, because it appears on the gingiva. Dental caries occurs only on tooth surfaces.
question
The recession on the maxillary canines and premolars was most likely caused by vigorous toothbrushing because trauma from aggressive toothbrushing is a leading cause of gingival recession.
answer
Both the statement and reason are correct and related. RATIONALE: Although many factors contribute to gingival recession, including trauma and inflammation, trauma from aggressive toothbrushing is a leading cause. Certain types of anatomy predispose patients to gingival recession. Another common cause is trauma caused by lip and tongue piercings.
question
At the 6-week reevaluation of the nonsurgical periodontal therapy, the presence of gingival bleeding surrounding tooth #10 indicates the necessity of a periapical radiograph. At the 6-week reevaluation of nonsurgical periodontal therapy, the presence of gingival bleeding surrounding tooth #10 indicates the necessity of avoiding periodontal probing and scheduling another reevaluation in 6 weeks.
answer
Both statements are false. RATIONALE: Although radiographs may be taken in a pregnant woman if necessary, there is no indication to do so in this situation. The best course of action during this reevaluation appointment is to update the client's periodontal assessment, including bleeding points, probing depths, and attachment levels. This will enable the dental hygienist to assess the clinical health of the tissues and forms the basis for subsequent treatment planning.
question
Reduction of dentinal hypersensitivity occurs as a result of paresthesia of the odontoblastic processes because the reaction between desensitizing agents and calcium forms crystals that occlude the dentinal tubules.
answer
The statement is NOT correct, but the reason is correct. RATIONALE: Paresthesia, a feeling of "pins and needles" that often results from poor circulation, does not contribute to reduction of dentinal hypersensitivity. Reduction of dentinal hypersensitivity occurs as a result of the reaction of desensitizing agents with calcium from within the dentinal tubules. This reaction forms crystals that occlude (block) the dentinal tubules and diminish transmission of pain. For example, the reaction of potassium oxalate and sodium fluoride with calcium from within the dentinal tubules forms calcium oxalate crystals and calcium fluoride crystals, respectively. These crystals reduce dentinal hypersensitivity by occluding the dentinal tubules.
question
Radiographic cone cuts are most likely the result of incorrect horizontal angulation of the position-indicating device (PID). A visible grid effect on a dental radiograph is caused by double exposure of the film.
answer
Both statements are false. RATIONALE: Radiographic cone cuts are the result of incorrect placement of the position-indicating device (PID). After processing, cone cuts appear as a clear zone on traditional radiographs. Incorrect horizontal angulation results in overlapping images of proximal teeth. A visible grid on a dental radiograph, often called the herringbone effect, results when the film is placed in the mouth backward. This causes the embossed pattern of the lead foil inside the packet to appear on the processed film. A double exposure results in indistinct images or dark x-rays.
question
Under Angle's classification of malocclusion, the occlusal relationship of the maxillary and mandibular first molars in centric occlusion would be classified as
answer
Class I. RATIONALE: In Class I malocclusion, the molar relationship is such that the mesiobuccal cusp of the maxillary permanent first molar occludes with the buccal groove of the mandibular permanent first molar. Additionally, one or more of the following may be present: crowded alignment of the anterior teeth; abnormal buccolingual tooth position; or the premature occlusal contacts. In class II, division I malocclusion, the buccal groove of the mandibular first permanent molar is distal to the mesiobuccal cusp of the maxillary first permanent molar by at least the width of a premolar. The mandible is retruded, and all maxillary incisors are protruded. This is not true in this client. In class II, division II malocclusion, the buccal groove of the mandibular first permanent molar is distal to the mesiobuccal cusp of the maxillary first permanent molar by at least the width of a premolar. The mandible is retruded, and one or more maxillary incisors are retruded. This is not true with this client. In a class III occlusal relationship, the buccal groove of the mandibular first permanent molar is mesial to the mesiobuccal cusp of the maxillary first permanent molar by at least the width of a premolar. This is not true in this client.
question
The condition of the gingiva on the facial surface of tooth #10 is best described as chronic diffuse gingivitis because chronic gingivitis is associated with long-term damage.
answer
The statement is NOT correct, but the reason is correct. RATIONALE: As shown in the photos, the condition of the gingiva on the facial surface of tooth #10 is best described as acute diffuse gingivitis. (However, the hygienist should recognize that there is an underlying chronic aspect to this patient's condition, as confirmed by the heavy supragingival and subgingival calculus.) The bright-red color in the photos denotes an acute inflammatory condition resulting from increased capillary dilation in the area. By contrast, chronic conditions are marked by less redness (erythema) and may even have bluish (cyanotic) tendencies. Also significant: The acute inflammatory phase is associated with faster onset, likely brought on by the hormonal shifts of pregnancy. Conversely, chronic conditions have a longer timeline. Furthermore, the condition is described as diffuse because it extends beyond the papilla and marginal tissues into the attached gingiva.
question
Concern for her unborn child would likely be an effective motivational appeal for this patient. The pain that has resulted from neglecting her teeth does not appear to be a strong motivating factor for this patient.
answer
Both statements are true. RATIONALE: From the other information given for this case, it is evident that this client is truly concerned about the health of her unborn child and could be motivated to improve her own health because it affects that of her child. Because pain is not reported in her history or statements, this does not appear to be a strong motivating factor for the dental hygienist to use when presenting oral hygiene instruction.
question
The client reports that she does not like milk and does not eat breakfast. All of the following foods should be recommended to increase her intake of calcium EXCEPT one. Which one is the EXCEPTION? A. Low-fat yogurt B. Salmon with bones C. Fresh spinach D. Cucumbers
answer
Cucumbers RATIONALE: Cucumbers do not provide a rich source of calcium. Low-fat yogurt is a good suggestion for increasing this client's calcium intake. Salmon with bones is a rich source of calcium and would be beneficial for this client. Fresh spinach is high in calcium content and would be an excellent recommendation for this client.
question
The client tells you that she has smoked cigarettes for a long time and asks whether smoking will affect her baby. All of the following adverse effects can occur in the fetus as a result of the mother's smoking habit EXCEPT one. Which one is the EXCEPTION?
answer
Intellectual disability RATIONALE: Low birth weight, premature birth, and miscarriage, as well as stillbirth and infant mortality, are possible adverse effects of smoking during pregnancy. Although intellectual disability is associated with alcohol use during pregnancy, it is not directly associated with smoking.
question
During the initial visit, the client reports that periodontal probing is painful and asks you to stop. Which of the following actions would be the BEST approach to follow?
answer
Discontinuing probing and waiting for the acute disease stage to resolve before attempting to take periodontal measurements RATIONALE: The best action is to discontinue probing and provide remedies, such as at-home care, that will foster resolution of gingival inflammation. This will enable an opportunity in the near future during which accurate periodontal assessment may be comfortably obtained. Explaining the importance of periodontal probing and asking her to concentrate harder will make the visit unpleasant. Not only is this not advisable, but it is also counterproductive and may prevent her from returning. Administering nitrous oxide and oxygen sedation is not advised, because administering drugs to a pregnant client should be avoided unless absolutely necessary. Administering a local anesthetic agent is not advised, because administering drugs to a pregnant client should be avoided unless absolutely necessary.
question
While asking many questions regarding her health and that of her baby, this client asks when her baby's teeth will begin to develop. Which of the following is correct?
answer
4 to 5 weeks in utero RATIONALE: The tooth buds develop between weeks 4 and 5 in utero. Initial mineralization occurs between weeks 9 and 12th weeks. The remaining selections are incorrect because tooth buds do not develop in utero between weeks 1 and 3, 6 and 7, or 8 and 10.
question
Potassium oxalate, sodium fluoride, and chlorhexidine can be recommended to relieve this client's sensitivity to cold and brushing because randomized clinical trials have documented that these three agents are effective in reducing dentinal hypersensitivity.
answer
NEITHER the statement NOR the reason is correct. RATIONALE: Although randomized clinical trials have documented that both potassium oxalate and sodium fluoride are effective in reducing dentinal hypersensitivity, chlorhexidine does not reduce dentinal sensitivity (or hypersensitivity). Chlorhexidine is used as an antiplaque and antigingivitis agent.
question
The contour of the gingival papilla around the mandibular teeth is BEST described as
answer
bulbous. RATIONALE: The contour of the mandibular gingival papilla is best described as bulbous because, as opposed to the other alternatives, the papillae fill the gingival embrasure spaces. However, note that bulbous papillae no longer possess a healthy pyramidal shape. Bulbous interdental gingival tissues exhibit inflammation (swelling) and rounding rather than the sharp peaks of healthy papillae. The contour of the tissues cannot be described as blunted, because blunted papillae, characterized by interdental voids, no longer fill the gingival embrasure. The contour of the tissues is not described as cratered, because the hallmarks of cratering, interdental necrosis and ulcerated "cratered" papillae, do not exist in this area. Note that cratered gingiva is also referred to as reverse gingival architecture. The gingival contour cannot be described as rolled, because rolled contours refer to lifesaver-shaped enlargements of the gingival margin.
question
Which of the following is likely to provide the MOST effective manner to teach toothbrushing technique to this client?
answer
Chair-side demonstration and discussion that enables the patient to experience correct technique intraorally RATIONALE: Demonstrating toothbrushing in a manner that allows this client to experience the tactile sensations of the correct technique will provide the most effective learning experience. Presentation of a pamphlet, interactive video, or online PowerPoint is inappropriate for a visually impaired client.
question
Each of the following considerations should be implemented in the treatment of a visually impaired individual EXCEPT one. Which one is the EXCEPTION?
answer
Grasp the client's arm to guide him or her. RATIONALE: Rather than having an arm grasped, most visually impaired people prefer to rest a hand on your arm so you can guide them. Providing verbal notice of approaching obstacles such as steps, furniture, or people is important. Politely asking about the client's preferred manner of escort enables you to learn what works best. Hazards should be moved for any client, especially the visually impaired person. Speaking before touching the client is common courtesy and will help you avoid startling the individual. Informing the client of your location is common courtesy and provides important information that helps an individual feel oriented and at ease.
question
After successful dental hygiene therapeutic interventions, the appropriate continued care interval for this client is 1 month because type 1 diabetes requires frequent monitoring of periodontal status.
answer
The statement is NOT correct, but the reason is correct. RATIONALE: Although frequent monitoring of periodontal status in patients with type 1 diabetes is advised, a continued care interval of 1 month is too frequent. Three months is the recommended interval for a patient with these risk factors.
question
Risk factors leading to a periodontal abscess include all of the following EXCEPT one. Which one is the EXCEPTION?
answer
Traumatic occlusion RATIONALE: Traumatic occlusion alone does not usually cause a periodontal abscess. Research supports the role of traumatic occlusion in the progression of inflammatory periodontal infection. Impaction of foreign objects in the sulcus is a risk factor for periodontal abscess. Incomplete scaling and root planing, resulting in retained calculus, can be a risk factor for periodontal abscess. The presence of intrabony pockets is a risk factor for periodontal abscess.
question
Which of the following methods should be used to remove soft deposits from this client's teeth?
answer
Toothbrush RATIONALE: Selective toothbrushing with a soft toothbrush is indicated for this client because of the many areas of exposed cementum and because there is very little extrinsic tooth staining present. There is no rationale for use of the airbrasive polisher on this client. A rubber prophy cup is not necessary to remove soft deposits. Use of an ultrasonic scaler to remove soft deposits from the teeth is not recommended, nor is it optimal therapy.
question
What precautions should be taken before the use of an ultrasonic scaler on this client?
answer
Checking the type of pacemaker (shielded or unshielded) RATIONALE: Ultrasonic devices should not be used on or near clients with unshielded cardiac pacemakers, because they may interfere with the proper function of the pacemaker. The dental hygienist should verify the status of the pacemaker. If it is shielded, the ultrasonic scaler can be used safely. Most pacemakers are now shielded. Premedication is not required for a client with a pacemaker, nor is it required for a patient with diabetes that is under control. This patient could benefit from fluoride, and there is no reason to refrain from using fluoride on this client. The client's vital signs are within normal levels, and there is no reason to reassess them at each appointment.
question
During treatment, the client experiences hypoglycemia. What action should be taken?
answer
Giving client a sugar source RATIONALE: The client should be given a source of sugar, such as orange juice, a sweet drink, cake icing, or hard candy. CPR is not indicated in a person who is breathing and has a pulse. Lowering the back of the dental chair and elevating the feet is not an appropriate action for a client experiencing hypoglycemia. It is more important that the client be in a position to safely be administered an oral source of sugar. Forcing fluids is an inappropriate action for this client and is dangerous.
question
Periodontal assessment reveals that this client has furcation involvement in several teeth. On tooth #19, the furcation invasion allows the probe to extend more than 1 mm horizontally but not completely through the furcation. Which of the following is the correct furcation classification?
answer
Class II furcation RATIONALE: Class II furcation invasion occurs when the bone loss allows the probe to extend more than 1 millimeter horizontally into the furcation. However, there is still some bone intact between the roots, and the probe will not pass completely through to the opposite furcation. Class I furcation involvement is defined as slight bone loss in the furca. The Nabers probe does not penetrate between the roots of the teeth, and the radiographic evidence of furcation involvement is usually not found. Class III furcation involvement occurs when the interradicular bone is absent but the entrance(s) to the furcation is (are) covered by gingival tissue. Involvement cannot be visualized clinically but appears as a distinct area of radiolucency on radiographs. In class IV furcation involvement, the interradicular bone is destroyed and the gingiva has receded apically, exposing the opening(s) to the furcation. It can be visualized clinically and radiographically.
question
In the photos, tooth #24 is in __________version.
answer
linguo RATIONALE: The photographs reveal that the central axis tooth #24 has moved lingually, which is the definition of a tooth in linguoversion. Another direction of tooth movement is mesioversion, in which a tooth moves closer to the midline. Conversely, a tooth in distoversion has moved in a distal direction, or farther from the midline. A tooth in facioversion has moved in the opposite direction of the tooth in the photo. More specifically, a tooth in facioversion has moved toward the labial or facial tissues.
question
Diseases or conditions that could manifest in this client with type 1 diabetes include all of the following EXCEPT one. Which one is the EXCEPTION?
answer
Cyanosis of nailbeds RATIONALE: Bluish discoloration of the nailbed is an early sign of cyanosis resulting from inadequate blood oxygenation and is not a direct manifestation of type 1 diabetes. Polydipsia, polyuria, and polyphagia are referred to as the three big symptoms of diabetes. Respectively, these terms are defined as: increased thirst and fluid intake; the need to urinate frequently; and increased appetite. Over time, diabetes damages the small blood vessels in the retina. Known as diabetic retinopathy, this type of blindness occurs in 80% of people with type 1 diabetes. Diabetes causes coronary heart disease by damaging the circulatory system; cardiovascular disease is the leading cause of mortality in people with diabetes. Poor blood circulation and nerve damage associated with diabetes cause ulcers in the lower extremities (particularly the lower legs and feet). The decreased healing capacity of the lower extremities, coupled with diabetic circulatory insufficiency, too often results in a crisis requiring amputation to save the client's life.
question
During the appointment, the client begins to perspire profusely. She reports that she is feeling weak, and her heart seems to be racing. What is the MOST likely cause of this reaction?
answer
Hypoglycemia (insulin reaction) RATIONALE: Hypoglycemia, the result of too much insulin in the blood and too little glucose, may occur if the client has failed to eat as normal pattern while continuing to take her regular dose of insulin. (Hypoglycemia is also referred to as insulin reaction). Signs of a diabetic coma (the preferred term is hyperglycemia) include dry, flushed skin and a weak pulse; there may also be a fruity odor to the breath. (These signs are not exhibited by this client.) This client has had extensive dental treatment with no prior anxiety problems; it is unlikely that her signs are caused by anxiety. Hyperglycemia, which can lead to diabetic coma, is also called high blood sugar. Signs of hyperglycemia include dry, flushed skin and a weak pulse. The breath of a client with hyperglycemia may have a fruity odor.
question
The crescent-shaped radiolucent area to the distal of #13 MOST likely represents which of the following?
answer
Recent extraction site RATIONALE: Because teeth #14, #15, and #16 are missing, and because of its shape and location, this crescent-shaped radiolucency is most likely the site of a recently extracted tooth. An intraradicular cyst usually appears as a round to ovoid radiolucency. The sinuses are located superior to the extraction site. This radiolucency, in the location of a missing tooth, is unlikely to be due to a bony defect. It is much more likely to be the site of a recently extracted tooth.
question
Which of the following measures is the MOST appropriate tool with which to assess the client's periodontal status?
answer
Gingival index RATIONALE: The gingival index (GI) is a measure of gingival inflammation, including bleeding, rather than the amount of deposit, calculus, or dental caries history. The plaque index (PI) is a measure of the presence of bacterial plaque but does specifically assess periodontal status. The calculus index (CI) is an assessment only of calculus; it does not address periodontal disease. The decayed, missing, and filled surfaces (DMFS) index is a measure of dental caries activity, not periodontal status.
question
Which of the following terms BEST describes the palatal tissue surrounding tooth #3 in the photograph?
answer
Cyanotic RATIONALE: The palatal tissue surrounding tooth #3 in the photograph can best be described as cyanotic. The hallmark of cyanotic tissue is the bluish tinge, caused by an excessive concentration of reduced hemoglobin in the blood. The palatal tissue surrounding tooth #3 in the photograph is not bulbous. Although bulbous gingival tissue may fill the gingival embrasure, it is rounded rather than having a sharp peak. The palatal tissue surrounding tooth #3 does not have blunted characteristics. Blunting refers to gingiva that has lost its pyramidal shape and no longer fills the gingival embrasure space. The palatal tissue surrounding tooth #3 does not have fibrotic characteristics. Although fibrotic tissue may appear pink, internal inflammation is present.
question
The wearing away of the tooth surfaces on the facial surfaces of the client's maxillary teeth can BEST be described as which of the following?
answer
Abrasion RATIONALE: Abrasion is the mechanical wearing away of tooth substance by forces other than mastication. The most common cause is the use of an abrasive dentifrice applied with vigorous horizontal toothbrushing. Attrition is the wearing away of a tooth as a result of tooth-to-tooth contact. Bruxism is the stress-induced behavior of grinding the teeth together. Note that bruxism can result in both attrition and abfraction. Although periodontal surgery affects both the gingiva and bone structure, it does not directly cause wear to the surfaces of the teeth.
question
The black spots on the left border of the mandible are the result of
answer
Improper film handling RATIONALE: It appears that a periapical film packet or some other object came into contact with this film during the developing process. Improper developing technique would affect the entire film, not just one spot. Film exposed to light would be completely dark. This artifact does not appear to be a fingerprint.
question
During your evaluation of soft tissues, you notice firm, irregular ridges on the anterior half of the hard palate. Which of the following is the MOST likely cause?
answer
Rugae RATIONALE: Rugae are firm irregular ridges of masticatory mucosa located on the anterior half of the hard palate. When prominent, rugae may be easily burned or traumatized. Candidiasis appears as multiple or diffuse white fungal lesions that appear thick and patchy and do not rub off with lateral pressure. Ulcerative stomatitis in situ is an infectious disease of the mouth characterized by swollen, spongy gums, ulcers, and loose teeth. Herpetic lesions consist of vesicles that rapidly rupture, forming painful ulcers. When present on the gingiva, herpetic lesions are often diffusely enlarged, erythematous, and ulcerated.
question
What is the preferred instrument to root-plane the distal surface of the mesial root of tooth #30?
answer
Gracey 13/14 curet RATIONALE: Because this area has class I furcation involvement, the Gracey 13/14 curet will adapt well to the distal surface of the mesial root. The mini and micro versions of this instrument may also prove helpful. Some of the new diamond furcation instruments should also be considered. The Gracey 11/12 curet is designed for use on mesial surfaces of posterior teeth. The Columbia 13/14 is a universal curet and could work in this area but is not the preferred instrument. The Gracey 4R/4L, not the preferred instrument for this area, is a universal curet designed to remove heavy calculus.
question
Of the medications taken by this client, which is used to control blood pressure?
answer
Monopril (fosinopril sodium) RATIONALE: Monopril is an angiotensin-converting enzyme (ACE) inhibitor and is used to control high blood pressure by dilating the arterial and venous vessels. Zocor is a cholesterol-lowering agent (e.g., statins). Sudafed (pseudoephedrine), available in both over-the-counter and prescription formulations, is used to treat sinus conditions. Zantac (ranitidine) is prescribed to treat symptoms of gastroesophageal reflux disease (GERD).
question
The professionally administered fluoride therapy that would be most beneficial for this client is
answer
sodium fluoride varnish (5%). RATIONALE: Sodium fluoride varnish (5%) is the preferred fluoride therapy for this client, because this formulation does not etch tooth-colored (porcelain, composite) restorations. Note that fluoride varnishes are the preferred delivery system for reasons of safety, ease of application, and opportunity for longer contact with the tooth surface. Acidulated phosphate fluoride could cause etching of tooth-colored (porcelain, composite) restorations. Foams are not recommended because of the lack of evidence efficacy. Stannous fluoride has astringent characteristics and causes burning of the tissues. Stannous fluoride may also pose problems in regard to the client's cleft palate. Sodium fluoride rinse is not a viable selection, because it contains a low concentration of available fluoride.
question
The indistinct vermilion border shown in photograph #1 is MOST likely a result of
answer
Sun exposure RATIONALE: Loss of the vermilion border is common in fair-skinned people who spend a great deal of time in the sun. This client has recently been treated for basal cell carcinoma and frequently plays golf in the sun. An indistinct vermilion border is not part of the normal aging process and in this case is associated with basal cell carcinoma. Surgery to correct cleft lip causes a scar, not an indistinct vermilion border. The client does not report a history of herpetic lesions; further, herpetic lesions would not cause loss of the vermilion border.
question
Which of the following methods is NOT an acceptable way of cleaning the client's partial denture in the treatment area?
answer
Polishing on a dental lathe RATIONALE: Before the completion of restorative procedures, this client wore a partial denture. Polishing of dental appliances on a dental lathe is not recommended because undue pressure may be put on the appliance, causing it to break. Also, because the denture is not sterilized and contains bacteria from the client's mouth, it should not be introduced to the polishing wheel. Scaling the client's partial denture by hand in the treatment room is an acceptable procedure. Using an ultrasonic instrument to clean the patient's partial denture in the treatment room is acceptable. Use of a denture brush while in the treatment room is an acceptable method of cleaning the patient's partial denture.
question
Which of the following aids would be MOST helpful to this client in removing interproximal plaque biofilm?
answer
Interproximal brush RATIONALE: Because this client has numerous challenges to self-care, such as open interdental embrasure spaces (missing teeth, blunted papillae) and dental implants, the interproximal brush would be the most beneficial. For many clients, flossing is challenging, and hence less effective, than the use of interdental brushes. An interdental stimulator could be beneficial, but the interproximal brush is the best choice for this client. Many clients find floss aids difficult to use, making them a less-than-ideal selection for this client.
question
What recommendation should be made to this client regarding his angular cheilosis/cheilitis?
answer
Use a lubricant frequently. RATIONALE: Angular cheilitis and its more chronic form, angular cheilosis, are inflammatory conditions that present with erythema and painful cracking, scaling, and bleeding at the corners of the mouth. (See photographs.) Frequent lubrication helps soothe the area and prevent further damage. Avoiding excessive sun exposure is a good recommendation for all clients, especially those with a history of skin cancer, but will not improve angular cheilitis/cheilosis. This client should use sunblock, because he has a history of basal cell carcinoma, but the sunblock will not affect the angular cheilitis/cheilosis. Rinsing with chlorhexidine has no effect on angular cheilitis/cheilosis.
question
The initial blood pressure reading on the client at this continued care appointment was 145/98 mm Hg. Which of these guidelines should be followed for this client?
answer
Recheck blood pressure before dental or dental hygiene therapy at consecutive appointments; if all readings exceed guidelines, seek medical consultation. RATIONALE: The blood pressure reading at this continued care appointment indicate that this client has stage I hypertension (systole of 140-159 OR diastole of 90-99). Although blood pressure in this range is above normal, it is not considered a medical emergency. Because the client is taking blood pressure medication, he should be asked whether he has been taking his medication and for the date of his last physician visit. The correct protocol for stage I hypertension is to check blood pressure at consecutive appointments. The selection that advises no unusual precautions is incorrect; this is the protocol for either normal blood pressure or prehypertension. The selection that advises rechecking blood pressure in 5 minutes and requiring medical consultation if the reading remains high describes the protocol for stage II hypertension. The selection that advises immediate medical consultation if the reading remains high after rechecking in 5 minutes describes the protocol for managing hypertensive crisis.
question
What mechanical technique should be used to remove the soft deposits on this client's teeth?
answer
Selective polishing with a soft toothbrush RATIONALE: Because this client has very light staining and bacterial plaque on the teeth, selective polishing with a soft toothbrush should be adequate. Devices that cause aerosolization, such as an airbrasive polishing device, should not be used because of the client's cleft palate and frequent sinus infections. Polishing with a rubber cup is not necessary in this client and could be considered overtreatment. An irrigating device will produce aerosolization and could cause sinus complications.
question
In the photo in which #20 has been extracted, the dark area on the gingiva to the mesial of #20 is MOST likely
answer
An amalgam tattoo RATIONALE: This dark area in question is most likely an amalgam tattoo. Note that it is located at the site of an earlier extraction and that a large restoration is present just posterior to tooth #20. Although this anomaly could be a melanoma, the fact that it is in the area of a prior extraction site and is also near a large restoration make an amalgam tattoo more plausible. Nothing in the client's history is suggestive of Kaposi sarcoma. This area has none of the blood vessel trauma associated with a hematoma.
question
Your primary concern in planning treatment for this client is
answer
Aspiration of liquids into the sinus RATIONALE: The client's history of cleft lip and cleft palate, as well as frequent sinus infections, should alert the dental hygienist to exercise caution when spraying water or air to prevent forced entry into the palatal opening into the sinus cavity. Any speech difficulties the client might have will not affect treatment. Tooth sensitivity should not be a primary concern in the planning of care for this client. Aside from the need to prevent the flow of water or air into the sinus cavity during treatment, the client's sinus infections do not pose an immediate problem in care planning.
question
What term BEST describes the anterior occlusion in this child?
answer
Open bite RATIONALE: The anterior occlusion seen in this child, best described as open bite, occurs when there is a lack of occlusal or incisal contact between the appropriate maxillary and mandibular teeth. Open bite occurs because teeth from either or both arches have failed to erupt to the line of occlusion. The teeth cannot be brought together when the jaws are closed; the resultant space is visible in the line of occlusion. Overjet refers to the horizontal distance between the labioincisal surfaces of the mandibular incisors and the linguoincisal surfaces of the maxillary incisors. Overbite is the vertical distance by which the maxillary incisors overlap the mandibular incisors. In underjet, the maxillary incisors are lingual to the mandibular incisors, with measurable horizontal distance evident between the incisal edges of the maxillary incisors and the incisal edges of the mandibular incisors. Note that underjet can be considered the "opposite" of overjet.
question
The child's mother asks you what symptoms she should watch for incase her daughter has a "bad reaction" to fluoride. The MOST common initial adverse reaction to fluoride toxicity is
answer
nausea. RATIONALE: Nausea is the MOST common initial adverse reaction to fluoride toxicity. Diarrhea can be a reaction to fluoride, but nausea is the most common. Abdominal cramping may occur as a reaction to fluoride, but nausea is the most common reaction. Respiratory distress may occur as a reaction to fluoride, but nausea is the most common reaction.
question
At the 6-month continued-care appointment, the client's mother reports that she had their drinking water tested and learned that the fluoride concentration is 0.2 ppm. Which of the following fluoride supplement dosages would be prescribed for this preschool-age child?
answer
0.5 mg F RATIONALE: The dosage schedule for fluoride supplementation released by the American Dental Association in 2010 is based on both the child's age and the concentration of fluoride in the drinking water source. For a 4-year-old with 0.2 ppm fluoride in her drinking water, the recommended supplemental dose is 0.50 mg/day. As with many health-care recommendations, dosages are occasionally adjusted. The dental hygienist should confirm the current recommendation. A fluoride dose of 0.25 mg/day is too low for this client and will not provide optimal protection. A dose of 0.75 mg/day is too high for this client and could cause fluorosis. A dose of 1.0 mg/day is too high for this client and could cause fluorosis.
question
What is the lethal level of fluoride consumption for a child this age?
answer
0.5 to 1.0 g F RATIONALE: The lethal level of fluoride for this 4-year-old child is 0.5 to 1.0 g. Lethal levels of fluoride for infants and the average adult (respectively) are 0.25 to 0.75 g and 4 to 5 g. Six to 10 g of fluoride is a lethal dose for a person of any age.
question
The narrow radiolucent line that extends vertically between teeth #E and #F on the maxillary anterior radiograph is
answer
a median palatine suture. RATIONALE: The median palatine suture appears as a narrow radiolucent band running superiorly from the crest of the alveolar ridge between the central incisors. Nutrient canals are more common on the mandible but may also be seem within maxillary sinuses. The incisive foramen is a small circular radiolucent area between the maxillary central incisors. A bend in the film is more typically found on the corners and does not have this distinct location or appearance.
question
The blunted root tips on tooth #O and #P that are visible on the mandibular anterior radiograph are due to
answer
physiologic root resorption. RATIONALE: Physiologic root resorption, a physiologic event for the primary teeth, is the response of the primary tooth to pressure from the erupting permanent tooth. Improper film placement did not cause the blunted root tips; this film is correctly placed. Root fracture did not cause the blunted root tips of teeth #O and #P. Note that there is no fracture line visible in this area of the radiograph. The blunted root tips of teeth #O and #P are not due to a pathologic condition; shedding of the deciduous teeth is a natural process.
question
Application of dental sealants on the first permanent molars as soon as they erupt should be recommended to this child's parent. After the placement of sealants and improvement of self-care, dietary counseling for caries control should be recommended to this child's parent.
answer
The first statement is true, the second is false. RATIONALE: This client's history of early childhood caries (ECC) indicates that dental sealants should be applied to the first permanent molars as soon as they are fully erupted. The parent should be advised that deep pits and fissures on molars are highly susceptible to occlusal caries. ECC is also associated with a high level of Streptococcus mutans transmitted from mother to infant, combined with frequent carbohydrate consumption. In addition, the child would benefit from fluoride varnish therapy. Dietary counseling for caries control should be recommended to this client's parents immediately.
question
This client lives in the desert Southwest in a community that does not have water fluoridation. What is the optimal amount of fluoride for the municipal water supply in this area?
answer
0.7 ppm F RATIONALE: The U.S. Department of Health and Human Services recommendation (2015) for the optimal fluoride level in drinking water to prevent tooth decay is 0.7 mg fluoride/L of water, more commonly stated as parts per million (ppm). This recommendation updates and replaces the previous recommended range, issued in 1962, of 0.7 to 1.2 mg/L (ppm) that was based on climate zones. One part per million of fluoride in community-fluoridated water is no longer the recommended concentration for any climate. Two parts per million is too high and could cause fluorosis. Two and a half (2.5) parts per million is too high and could cause fluorosis.
question
The root and pulp chamber of tooth #20 appears considerably larger than those of the other mandibular premolars because hypercementosis has occurred.
answer
The statement is correct, but the reason is NOT. RATIONALE: The photographs and radiograph of the mandibular teeth, as well as the incline of the occlusal plane of this tooth, make it obvious that tooth #20 is rotated about 90 degrees from its intended position. This change in orientation accounts for the larger radiographic appearance of the root and pulp chamber. Hypercementosis would appear as a radiopaque area at the apex of the tooth.
question
The dental hygiene care plan for this client should include all of the following EXCEPT one. Which one is the EXCEPTION?
answer
Home use of disclosing tablets RATIONALE: Use of disclosing tablets or solution is contraindicated for this client because of the loss of enamel and exposure of large areas of dentin on many teeth. Because dentin is more absorbent than enamel, it is likely to absorb color from disclosing solution, further staining the teeth. To motivate this client, in place of disclosing tablets/solution, the hygienist could use the generalized bleeding points as a visual image. A thorough review of toothbrushing technique is a necessary component of the dental hygiene care plan for this client. Home application of fluoride is a necessary component of the dental hygiene care plan for this client. An oral cancer self-examination is a necessary component of the dental hygiene care plan for this client.
question
Amelogenesis imperfecta is a form of enamel dysplasia resulting from unknown factors. The dentin and pulp of teeth with amelogenesis imperfecta develop normally.
answer
The first statement is false, the second is true. RATIONALE: The cause of amelogenesis imperfecta is well established. It is an inherited condition affecting the tooth enamel that results from a malfunction within the tooth germ. Development of the dentin and pulp is not disrupted by amelogenesis imperfecta. Note that dentinogenesis imperfecta, also an inherited condition, causes a disturbance in dentin formation.
question
The color of the attached gingiva and alveolar mucosa in the anterior region is MOST likely due to which of the following?
answer
Normal melanin pigmentation RATIONALE: This tissue change is most likely a result of normal melanin pigmentation. The client is not taking any medications and does not report using a therapeutic mouthrinse. Because he uses only spit tobacco, tobacco stomatitis would not be a valid reason for the tissue color. Tobacco stomatitis is white, not brown. Drug-induced gingival enlargement is incorrect, because the client is not taking any medications. Therapeutic mouthrinses such as chlorhexidine gluconate can cause extrinsic tooth staining but do not cause darkening of the gingival tissues. Cetylpridinium chloride can cause slight extrinsic staining of the teeth but does not stain the gingival tissues.
question
Which one of the following bacterial plaque biofilm removal devices should NOT be recommended to this client?
answer
Sonic/ultrasonic toothbrush RATIONALE: Because of this client's amelogenesis imperfecta, use of a sonic or ultrasonic toothbrush is not advised. The vibrations from the ultrasonic/sonic brushstrokes, combined with the pressure applied by the client, could cause flaking of the remaining fragile enamel. A manual toothbrush should be recommended, but the bristles should be soft and the client should be instructed in proper technique. An end-tuft toothbrush is highly appropriate for this client. For areas in which access is possible, an interdental brush can be recommended to this client.
question
The localized lesion on the buccal mucosa of the client's mandibular lip is best described as
answer
corrugated and plaque-like. RATIONALE: The surface of this lesion has a corrugated (wrinkled) surface with a plaque-like consistency. Note that it is slightly raised, with a broad, flat top and a "pasted on" appearance. Papules are circumscribed solid elevations less than 0.5 cm in diameter that contain no visible fluid. The lesion in question does not have the characteristics of a papule. A fissured surface, irregular by definition, has a deeply cracked appearance, not the wrinkled appearance of this lesion. Vesicles are circumscribed elevations of skin that are less than 0.5 cm in diameter and contain a clear fluid (serum). The fluid gives the lesion a clear or translucent, slightly white appearance.
question
The cause of the localized wrinkled white lesion in the vestibule and on the left labial mucosa of the mandibular lip is MOST likely
answer
use of smokeless tobacco/spit tobacco. RATIONALE: The client's daily use of smokeless tobacco/spit tobacco is most likely the cause of this lesion. Regular use of spit tobacco is known to cause homogenous leukoplakia of the oral mucosa. Tartar-control toothpaste may cause generalized sloughing of the oral mucosa but will not cause a localized lesion like the one depicted in the photograph. Habitual lip biting produces a scarred appearance rather than the wrinkled white lesion depicted in the photograph. Irritation resulting from malocclusion is an unlikely cause; there is no basis to identify this as a causative factor.
question
The bone loss on the distal surface of tooth #29 can BEST be described as vertical because on adjacent teeth the rate of bone loss is similar and even decline of the alveolar bone crest is noted.
answer
The statement is NOT correct, but the reason is correct. RATIONALE: The bone loss on the distal surface of tooth #29 is best described as horizontal bone loss. This is because the rates of loss of bone are similar on adjacent teeth and there is an even decline in the level of the alveolar crest. By contrast, vertical bone loss is characterized by a reduction in the height of crestal bone that is irregular in pattern and more commonly localized than generalized. The dental hygienist should be aware that an intraosseous defect, which is not present in this area, is characteristic of advanced periodontitis and usually involves tooth mobility and localized areas of furcation involvement. Another related term, interdental crater, describes the irreversible tissue loss of necrotizing ulcerative gingivitis and necrotizing ulcerative periodontitis.
question
How is the anomaly present on the root of tooth #31 BEST described?
answer
Taurodont RATIONALE: A taurodont is a tooth in which the pulp chamber is elongated and enlarged and extends deeply into the roots. The teeth appear normal in all aspects clinically, but radiographs reveal the abnormally large pulp chamber and the more apical location of the furcation. Dilaceration is a severe distortion of a crown or root caused by trauma during tooth formation. It usually manifests as a severely angulated root. Fusion is the formation of a single tooth from the union of two adjacent tooth buds. The two buds are united through the enamel and dentin and occasionally the pulp. Concrescence is the fusion of two teeth at the root through the cementum only. The teeth originally involved were separated but later joined as a result of excessive cementum disposition.
question
Which of the following dental hygiene services should be provided for this client?
answer
Scaling and oral debridement with hand instruments RATIONALE: Hand scaling and debridement are necessary to protect the fragile enamel. Sonic or ultrasonic scalers may cause damage to fragile enamel. Although rubber cup polishing should be used selectively in all patients, this procedure is contraindicated in patients with amelogenesis imperfecta because it is likely to remove portions of the remaining fragile enamel. The airbrasive polisher may cause loss or flaking/damage of the fragile remaining enamel.
question
During the course of dental hygiene care, the client becomes rigid, his eyes roll upward, and he begins to have a seizure. Which of the following is the MOST appropriate course of action?
answer
Placing the chair in a supine position and, if possible, turning the client to the side and letting the seizure occur RATIONALE: It is best to leave the client alone and move things out of the client's way. If possible, turn the client on his side to minimize aspiration of secretions. Never force or place anything in the person's mouth; this may cause breakage of the teeth. It is best to leave the client alone and move things out of the way. Trying to place an object in the client's mouth could result in injury to the client or the fingers of the person offering assistance. It is not necessary to call for emergency help unless status epilepticus occurs and the seizure lasts longer than 5 minutes.
question
The client's generalized gingival enlargement is most likely a result of his heavy subgingival calculus because calculus deposits cause gingival enlargement.
answer
NEITHER the statement NOR the reason is correct. RATIONALE: Although calculus can contribute to gingival enlargement by retaining plaque biofilm, it does NOT cause gingival enlargement. The generalized gingival enlargement (formerly called gingival hyperplasia) is a side effect of the medication this client takes for his seizure disorder. His medical history indicates that he takes phenytoin sodium, a generic name of a drug used to control seizures. Some common brand names for this drug are Dilantin, Dilantin Kapseals, Diphenylan, and Dilantin Infatabs. One common side effect of this drug is gingival enlargement. The dental hygienist should note that the term epilepsy is no longer used to describe symptoms of chronic or recurrent brain dysfunction. The correct current terminology is seizure disorder.
question
Risk factors in this client that contribute to drug-induced gingival enlargement include all of the following EXCEPT one. Which one is the EXCEPTION? A. Mouth breathing B. Heavy calculus C. Aspirin D. Heavy bacterial plaque biofilm
answer
Aspirin RATIONALE: Although phenytoin sodium, which the client takes for his seizure disorder, is a direct cause of gingival enlargement, aspirin is not a contributing factor. Drug-induced gingival enlargement is exacerbated by the irritating and drying effects of mouth breathing as well as heavy deposits of calculus and bacterial plaque biofilm. In fact, meticulous oral hygiene can sometimes alleviate drug-induced gingival enlargement. Other contributing factors include overhanging or defective restorations, large carious lesions, and other plaque retentive factors.
question
Short appointments early in the day may reduce the likelihood of a seizure in a susceptible client in the oral health care setting. Should this client experience a seizure, the hygienist should position the client on his back once seizure activity subsides.
answer
The first statement is true, the second is false. RATIONALE: To reduce the risk of a seizure in a susceptible client, the scheduling of short appointments early in the day is advised. After the seizure is over, the client should be positioned on his side for drainage of secretions. Note that care is required if a back or neck injury is suspected.
question
The horizontal radiolucent band in the apical third of the root of tooth #8 represents
answer
a fracture. RATIONALE: The adjacent tooth, #9, was lost in an accident many years ago. It is highly probably that this accident also caused a fracture of the root of tooth #8. Also note the radiolucency at the apexes of teeth #8 and #9, again likely due to trauma from the accident. The horizontal radiolucent band is not an artifact, because it can be explained by the client's history. The nasopalatine canal, a relatively long narrow structure, is located in the midline of the maxilla and not in the area of interest. A bend in the film does not have this appearance and would not appear in the middle of a film.
question
Which of the following actions during dental hygiene care of this patient could precipitate a seizure?
answer
Flashing the overhead light in the client's eyes RATIONALE: For some individuals, certain situations or stimuli can increase the likelihood of a seizure. These include stress, limited sleep, improper diet, and flickering lights or sound. Some sources of flickering lights include television, video games, and areas of shadows and light. The overhead dental light falls into this category. Touching a particular area of the client's lip will not precipitate a seizure. Placing the client in the supine position (with the legs elevated) is advised during a seizure; it will not precipitate a seizure. Although speaking to the client in a rapid manner may be disorienting to the client, it will not precipitate a seizure.
question
The dental hygiene care plan for this client should include the use of
answer
a sonic/ultrasonic scaler. RATIONALE: For the comfort of both client and clinician, the initial removal of heavy deposits is best accomplished with the use of a sonic/ultrasonic scaler. Selective polishing with an airbrasive polisher is not necessary, because as the sonic/ultrasonic scaler removes heavy deposits, extrinsic staining is coincidentally removed. Unless prescribed by the dentist or physician, antianxiety medication is not indicated. Selective polishing with a rubber cup is not necessary, because as the sonic/ultrasonic scaler removes heavy deposits, extrinsic staining is coincidentally removed.
question
The radiolucency associated with the apex of tooth #10 MOST LIKELY represents
answer
a periapical abscess. RATIONALE: Radiographically, a periapical abscess is a radiolucency associated with the apex of a tooth. The normal lamina dura and periapical space are lost, because this infectious process destroys bone. The anterior palatal foramen is a round or ovoid radiolucency in the midline of the maxilla that is not associated with a tooth. The coronoid notch is a scooped-out concavity of bone located distal to the coronoid process on the ramus of the mandible. The mental foramen appears as a small, dark (radiolucent) circle near the apex of the mandibular second premolar.
question
The mandibular lingual protuberances near the premolars and canines are MOST likely
answer
tori. RATIONALE: Mandibular tori, common among adults, appear as hard bony prominences on the lingual alveolar process in the canine and premolar region. They usually appear bilaterally and are radiopaque radiographically. An exostosis is a bony growth projecting from the surface of bone. Although the area in question is a bony growth, when it occurs bilaterally on the lingual of mandibular teeth it is more properly called a torus. Sialoliths are calcifications of the salivary gland; they appear opaque on radiography. A sialolith would not be this large, nor would it appear bilaterally as do the protuberances in these photos. Hyperplasia is an increase in the size of the cells and tissue. The increase in size in this case is related to bone mass, not to the gingival tissues.
question
Which of the following is the BEST approach to planning hard deposit removal on this client?
answer
Scaling and debriding a segment to completion RATIONALE: Scaling and debriding a segment of the mouth to completion is the best approach to removing hard deposits in this client. Such thorough treatment of a limited area includes root planing, removes all depots, and enables the healing process to begin. In contrast, gross scaling often leaves many areas of subgingival deposit and may result in the formation of periodontal abscesses. After tissue shrinkage occurs, subgingival calculus retained after gross scaling is likely to be more difficult to remove during future visits. Complete gross scaling during the first appointment may leave subgingival deposit that inhibits healing and causes periodontal abscesses. Administration of a local anesthetic agent may or may not be indicated for this client. The client should be consulted and informed consent obtained. There is no rationale for treating only the anterior teeth during the last appointment.
question
The round to ovoid radiopacities present in the right and left parts of the first panogram represent which of the following?
answer
Image of client's earrings RATIONALE: The large ovoid bilateral radiopacities on the right and left sides of the first panogram represent jewelry, most likely earrings, that were not removed before exposure of film or image capture (digital). Although it is not entirely incorrect to refer to these opacities as artifacts, in this case the dental hygienist can state with confidence the exact cause (earrings left on during radiographic exposure). A processing error would not cause such distinct ovoid bilateral radiopacities. These two distinct bilateral radiopacities do not resemble a napkin chain; they very much appear to be jewelry, most likely earrings, that were not removed.
question
Which one of the following is recommended to remove the staining and deposits on the lingual surfaces of the mandibular anterior teeth?
answer
Ultrasonic scaler RATIONALE: Although some opinions suggest that scaling and root planing should be performed on HIV-positive/AIDS patients with the use of hand-activated instruments alone because power scaling instruments generate aerosols, to date there have been no documented cases of HIV infection through aerosol exposure. A study analyzing dental treatment issues concluded that with the exception of patients with bleeding disorders, there is no reason to take special precautions when treating HIV/AIDS patients who are healthy enough to be seen in an outpatient setting. Dental hygienists may prefer to perform therapeutic scaling and root planing procedures with the use of ultrasonic instrumentation because treatment time is generally shorter and because the treatment can often be performed without t injected local anesthetic agents, thereby reducing the chance of needle punctures. Use of an air-abrasive polishing system is ineffective in hard deposit removal. The Gracey 1/2 area-specific curet is designed to be used on anterior teeth and can be used to root plane the root surfaces of these teeth. Sickle scalers are designed for supragingival calculus removal and are not effective in root planning in deep pockets.
question
At the client's 4-week re-evaluation after scaling and root planing therapy, there is no appreciable decrease in periodontal pocket depth. Each of the following EXCEPT one is a possible reason. Which one is NOT a possible reason?
answer
Necrotizing ulcerative periodontitis does not always respond to conventional therapy. RATIONALE: Necrotizing ulcerative periodontitis, as found in immunocompromised persons, does not always respond well to nonsurgical periodontal therapy. Note that necrotizing ulcerative periodontitis is evident in the photo of the left posterior molars. Although pocket depth did not INCREASE, note that severe soft tissue necrosis and rapid periodontal attachment destruction is a distinguishing feature of NUP. It is possible that the client did not carry out the oral hygiene self-care recommendations, but it is unlikely that this influenced pocket depths in the space of 4 weeks. A decrease in CD4 lymphocyte count is unlikely to have influenced pocket depth in 4 weeks. A change in medications by the client's physician is unlikely to have influenced pocket depth in just 4 weeks.
question
The white lesions evident on the right and left lateral border of this client's tongue are MOST likely which of the following?
answer
Hairy leukoplakia RATIONALE: Oral hairy leukoplakia is a white lesion found predominantly on the lateral margins of the tongue. Virtually all persons who exhibit hairy leukoplakia are HIV positive. The surface may be smooth, corrugated, or markedly folded. Oral candidiasis will produce multiple or diffuse white curd-like lesions that appear variably thick and patchy and do not rub off with lateral pressure. Lichen planus is characterized by the chronic occurrence of multiple lesions affecting the skin, mucous membranes, or both. It is usually seen in middle-aged persons and may or may not be symptomatic. Herpetic lesions consist of vesicles that rapidly rupture, forming painful ulcers.
question
The gingival condition on the facial gingiva of teeth #23 through #26 can be described as linear gingival erythema. The periodontal condition on the facial surface of tooth #17 is BEST described as HIV-associated periodontitis.
answer
The first statement is true, the second is false. RATIONALE: The facial gingiva surrounding teeth #23 through #26 exhibit symptoms of linear gingival erythema. This condition is marked by a profoundly red (erythemic) band along the gingival margin. It is usually accompanied by mild to moderate pain and responds poorly to conventional treatment. The correct classification for this client's periodontal condition is necrotizing ulcerative periodontitis (NUP), a disease that occurs in persons with HIV infection, AIDS, and other immunocompromising diseases. Characterized by severe soft tissue necrosis and rapid destruction of periodontal attachment and bone, NUP may lead to exposure of alveolar bone, as is evident in this client's mandibular left molars. Note that there is not a periodontal disease classification called HIV-associated periodontitis.
question
The anterior teeth in panoramic radiograph #1 appear abnormally small. This appearance represents which of the following errors?
answer
Client positioned too far forward in machine RATIONALE: In panoramic radiograph #1, the teeth appear abnormally small because the client was positioned too far forward in the machine. The rule of thumb: The closer the object is to the film, the smaller its image. Positioning the client too far back in the machine causes the anterior teeth to appear elongated. Client movement causes a blurred image. Tipping the client's head to the side results in an image with foreshortened teeth on one side and elongated teeth on the other side.
question
Diflucan has been prescribed for
answer
oral candidiasis. RATIONALE: Diflucan (fluconazole) is an antifungal drug that is used to treat oropharyngeal candidiasis and chronic mucocutaneous candidiasis. Because patients with HIV/AIDS typically take multiple medications, diflucan is prescribed as a preventive measure to reduce the risk of fungal infections. Herpetic lesions are caused by a virus and are therefore not affected by antifungal drugs. Pneumonia can be caused by either a virus or bacteria and is not affected by antifungal drugs. A client's CD4+ count is not affected by antifungal drugs.
question
The density in panogram #1 could be improved by increasing the kilovoltage. The contrast in panogram #1 could be improved by increasing the milliamperage.
answer
Both statements are false. RATIONALE: Radiographic density, the overall blackness or darkness of an image, is controlled by the milliamperage (mA) setting. Images with the correct density enable the dental professional to distinguish radiolucent black areas (air spaces); radiopaque white areas such as enamel, dentin, bone, and amalgam restorations; and grey areas of soft tissue. Radiographic contrast, the ability to distinguish various shades of gray, is controlled by the kilovolt peak (kVp) setting. The ideal contrast of a film clearly shows the white of radiopaque metal restorations, the radiolucent black of air, and the many shades of gray between.
question
Which of the following suggestions is the BEST way to help this client remove interproximal bacterial plaque biofilm?
answer
Floss holder RATIONALE: There are many types of floss holders. Most are very easy to use and do not require as much manual dexterity as flossing, interdental brushes, and end-tuft brushes do. Floss, whether waxed or unwaxed, requires a higher level of manual dexterity than does a floss holder. An interdental brush is not indicated for this patient, because even though his papillae are inflamed, they are intact in most areas. Intact papillae that fill the interdental space can easily be blunted by interdental brushes and other self-care aids that do not fit easily. An end-tuft toothbrush would not be effective in removing interproximal plaque from this client's teeth.
question
The proper premedication protocol for this patient is 2.0 g of oral amoxicillin 30 to 60 minutes before dental hygiene therapy because the 2007 American Heart Association guidelines include mitral valve prolapse as a condition requiring antibiotic premedication before dental procedures.
answer
NEITHER the statement NOR the reason is correct. RATIONALE: Antibiotic premedication is no longer recommended for patients with mitral valve prolapse. In 2007 the American Heart Association issued new guidelines for antibiotic prophylaxis for dental patients. Note that it is not clear whether this patient has a more advanced form of mitral valve prolapse called mitral valve prolapse with regurgitation. This more advanced condition required antibiotic premedication before 2007. This client may not take amoxicillin for any reason, because it is penicillin to which he is allergic. Hygienists should be aware that for patients who are not allergic to penicillin and for whom antibiotic premedication is recommended, the protocol stated in the question is correct.
question
Zinc chloride is an appropriate temporary restorative material for tooth #14. Zinc chloride provides thermal insulation and delivers medication to the pulp.
answer
Both statements are false. RATIONALE: Zinc chloride is not a temporary restorative material. An appropriate temporary restorative material for tooth #14 is zinc oxide-eugenol. This cement liner with both antimicrobial and sedative properties will be beneficial for this client. Zinc chloride neither provides thermal insulation nor delivers medication to the pulp. However, as a component of anticalculus agents, zinc chloride has the ADA Seal of Approval for its ability to inhibit supragingival calculus accumulation.
question
The area denoted by the arrow in the radiograph between teeth #23 and #26 is most likely the incisive foramen because this radiopaque structure is located in the midline of the mandible apical to the mandibular central incisors.
answer
The statement is NOT correct, but the reason is correct. RATIONALE: The structure denoted in the radiograph by the arrow is a collection of spiny radiopacities known as the genial tubercles. Often imaged as a circle, this radiographic landmark is located in the midline apical to the mandibular central incisors. (Teeth #24 and #25 are missing.) Note that the incisive foramen is an oval radiolucency location between the maxillary incisors.
question
Punishment, such as taking away a privilege or snack, is an effective motivator to suggest to his caregiver because this type of deprivation will help the client focus on his self-care and be more appreciative of his caregiver's efforts.
answer
NEITHER the statement NOR the reason is correct. RATIONALE: Punishment is neither an acceptable nor an effective motivator. For any patient, particularly a patient with Down syndrome, motivational strategies involving positive reinforcement and rewards are most effective. Suggestions for his caregiver might include use of a powered toothbrush, a reward system, working with him in short sessions, and providing positive feedback for even the smallest improvement. Punishment is counterproductive and damaging and will have far-reaching negative consequences.
question
Which of the following motivational appeals would be MOST successful in helping this client improve his self-care?
answer
Reward system RATIONALE: This client will most likely be motivated by a reward system. Because he is a big Elvis fan, perhaps some small reward, such as pictures of his hero, could be given to encourage him to improve his oral self-care. Appealing to his pride in his appearance could also be used to motivate this patient, but an award system has a higher likelihood of success. This client is cared for by his sister, and it is unlikely that saving money would be motivational. Linking reduced tooth decay with improved self-care is unlikely to be as effective as an award system.
question
The enlargement extending across the hard palate from tooth #7 to tooth #10 is the
answer
palatine rugae. RATIONALE: The palatine rugae are numerous transverse ridges of fibrous tissue that extend across the anterior portion of the hard palate. The palatine fovea is in the center of the palate, near the soft palate. The canine eminence is not located on the facial aspect of the maxillary canine teeth. The incisive papilla is located anterior to the palatine rugae, between teeth #8 and #9.
question
Which of the following terms describes the radiographic technique error in the mandibular anterior periapical image?
answer
Foreshortening RATIONALE: In the image of the mandibular anterior teeth, incorrect radiographic procedure produced an image with foreshortened teeth and oral structures: The central ray was not projected perpendicular to the film/receptor. Note that the teeth appear small and the lower edge of the film, where the oral structures should be visible, appears radiolucent. This results from too much vertical angulation. This may also result if the film or receptor is not placed parallel to the long axis of the teeth. Elongation would cause the roots to appear long. Cone cutting would produce an clear/opaque semicircle or rectangle near the corner or edge of the film. Backward film placement would produce a grid-like appearance called the herringbone pattern/effect. Note that this does not occur with digital radiography.
question
Which of the following home care products and methods could be recommended to help this client manage his dental caries?
answer
Brushing with a prescription fluoride gel RATIONALE: A prescription fluoride paste, used under supervision, should be recommended to manage this client's caries activity. Brushing with an alcohol-free mouthrinse will not prevent dental caries. Further, rinses are problematic for people with Down syndrome because swishing and expectorating is problematic. As a result they often swallow the product and suffer from stomach upset and/or fluoride toxicity. Antiplaque rinses have little effect on dental caries activity.
question
Which of the following is MOST likely the major cause of the client's weight loss?
answer
Poor nutrition RATIONALE: This client's weight loss can most likely be attributed to poor nutrition. Her inability to open her mouth and the shifting of her jaw compound the detrimental effects of xerostomia. Because chewing food is difficult, she should be counseled on dietary and nutritional requirements. The dental hygienist should encourage vitamin and nutritional supplements to preserve/enhance strength and prevent further weight loss. Although she has lost her lower teeth, the client can still consume nutritional foods. Xerostomia has contributed to the client's weight loss, but it is not the most likely cause. Squamous cell carcinoma, a complicating factor, is not the primary cause of her weight loss.
question
All of the following should be recommended to this client to augment her intake of nutrients EXCEPT one. Which one is the EXCEPTION?
answer
Drinking tomato juice RATIONALE: Tomato juice should be avoided because the high salt and acid content of most commercial brands is irritating to oral tissues dried by xerostomia. The client will be able to consume a wider variety of nutrients if food is first pureed in a blender. Use of nutritional supplements will provide the client with beneficial nutrients that she might not otherwise consume. Milkshakes, provided that they are low in sugar, are a good recommendation because they are soothing to dry tissues, provide minerals such as calcium, and have a pleasing taste.
question
Referring to the pre-extraction panograph, the MOST likely cause of the radiolucent area in the angle of the mandible distal to tooth #31 is
answer
carcinoma. RATIONALE: Invasion of bone by malignant neoplastic lesions produces heterogeneous radiolucency with poorly delineated margins. Destructive invasion of bone is evidenced by areas of ragged cortical erosion. Often present are "pockets" of radiolucency that are sometimes described as moth-eaten in appearance. Osteoporosis appears as a generalized radiolucency and lacks the poorly delineated margins of carcinoma. Sclerotic bone formation, because of its density, is radiopaque in appearance. An ossifying fibroma, most frequently occurring in premolar region of the mandible, appears as a mixed radiolucent/radiopaque lesion.
question
Cervical caries in this client is MOST likely caused by
answer
xerostomia. RATIONALE: Although numerous factors have contributed to cervical caries, xerostomia puts this client at extreme risk for dental caries. Her history of severe xerostomia is most likely responsible for the numerous carious lesions in the cervical areas of her remaining dentition. The relative absence of lubrication by saliva deprives the dentition of a vital defense against the multitude of microorganisms that promote demineralization, decalcification, and decay. Diet may be a contributing factor, but xerostomia is a more serious risk factor. Poor brushing may be a contributing factor, but xerostomia is the more serious risk factor. Chemotherapy in itself is not a cause of cervical caries, although it indirectly contributes to the decay process.
question
Yogurt is a good recommendation for a nutrient-rich snack because it is easy to eat and low-sugar/low-fat varieties are readily available.
answer
Both the statement and reason are correct and related. RATIONALE: Yogurt is easy to eat and provides a large amount of calcium and protein, and low-sugar/low-fat varieties are readily available. The hygienist should be aware that ice cream is not advisable because it is cariogenic. Cheese may also be beneficial if the client is able to chew it.
question
The radiopaque areas evident on the roots of tooth #20, #21, and #27 through #29 represent
answer
mandibular tori. RATIONALE: The supplemental findings for this client indicate that she has bilateral mandibular tori. These benign bony growths appear radiographically as radiopaque areas superimposed on the roots of mandibular teeth. Osteoporosis, a condition that causes bones to become weak and brittle, appears radiographically as areas of radiolucency. Carcinoma typically appears as a radiolucent area with poorly delineated (ill-defined) borders. The internal oblique ridge, located on the lingual surface of the mandible, extends from the level of the roots of the last molar to the floor of the mouth. This ridge is not present in the areas of concern.
question
When the original lesion was discovered, the clinician could expect to see all of the following findings EXCEPT one. Which one is the EXCEPTION?
answer
Painful enlargement of regional lymph nodes RATIONALE: In such a case, painful enlargement of regional lymph nodes is not typical. In cases of painful enlarged regional lymph nodes, metastasis from the lesion to distant sites has usually occurred. Bone pain that is poorly localized (i.e., extending beyond the original lesion), is an expected symptom of carcinoma within bone. An area of induration, also described as sclerotic or hardened, is to be expected with carcinoma in a bone. A rough, heterogeneous (non-uniform) appearance of a lesion is an expected condition of carcinoma within bone.
question
The most appropriate interval for continued care for this client is
answer
3 months. RATIONALE: Because severe xerostomia puts this client at extremely high risk for dental caries, 3 months is an appropriate interval between her continued care appointments. This client has a very low plaque score, so a 1-month interval between continued care visits is too short. Because a 3-month interval is appropriate for this client, 4 months is too long between continued care visits. Because both xerostomia and the history of oral cancer put this client at extremely high risk, a 6-month interval between continued care appointments is excessively long.
question
As part of her instruction in oral cancer self-examination, this client should be encouraged to report each of the following to her dentist immediately EXCEPT one. Which one is the EXCEPTION? A. Sores that do not heal within 2 weeks B. Red or white patches in her mouth C. Loss of feeling/numbness in the head/neck region D. Bad odor in the mouth
answer
Bad odor in the mouth RATIONALE: Although oral malodor has dental and periodontal significance, it is not necessarily an indication of a precancerous condition and is usually related to a sinus infection, respiratory infection, or poor oral hygiene. Sores that do not heal within 2 weeks should be reported to her dentist immediately, because this is a warning sign of oral cancer. Red or white patches in the mouth should be immediately reported to the dentist, because they are indicative of oral cancer and should be evaluated. Loss of feeling or numbness in the head and neck region should immediately be reported to her dentist, because this finding is often associated with oral cancer.
question
The client's xerostomia is so severe that swallowing food is difficult. All of the following suggestions are appropriate EXCEPT one. Which one is the EXCEPTION? A. Take sips of water before taking a bite of food. B. Eat juicy fruits and suck on ice chips. C. Use a saliva substitute and avoid caffeine. D. Take smaller bites.
answer
Take smaller bites RATIONALE: Although taking smaller bites is wise, the hygienist should be aware that the amount of food placed in the client's mouth is not the main factor contributing to her swallowing difficulty. The root cause is her lack of saliva. This deficiency prevents the moistening of food necessary to initiate the critical first stage of the digestive process. Advising this client to take sips of water before taking a bite of food is a proactive suggestion that will be of great benefit. Advising this client to eat juicy fruits and suck on ice chips is a beneficial strategy. An important caveat, because xerostomia places this client at high risk for dental caries: Proper preventive strategies should accompany any recommendation of ingestion of juicy fruits. Advising the client to use a saliva substitute and to avoid the drying effects of caffeine will both be of great benefit to this client.
question
This client's mother confides that she often lets her son take a sippy cup with fruit juice to bed with him because it helps him fall asleep. Each of the following is a sound nutritional concept EXCEPT one. Which one is the EXCEPTION? A. Taking a sippy cup to bed is fine, but use of a baby bottle should be avoided. B. Fluoride is the most effective means of preventing and controlling dental caries. C. Prolonged at-will bottle feeding is a predisposing factor for dental caries. D. Fruit juices typically contain added sweetener
answer
Taking a sippy cup to bed is fine, but use of a baby bottle should be avoided. RATIONALE: Nocturnal bottle feeding, including the use of a sippy cup, should be avoided after the first primary tooth begins to erupt. It is important that the parent understand that both formula and fruit juice are acidogenic and increase the potential for early childhood caries (ECC). The dental hygienist should ensure that the mother understands the critical role of fluoride in the prevention of dental decay. Because this child does not benefit from systemic fluoride (i.e., fluoridated water), the dental hygienist should provide recommendations to ensure appropriate fluoride protection. Fruit juices, as well as formula, are high in dietary sugars, also referred to as fermentable carbohydrates. Each time a fermentable carbohydrate is consumed, demineralization of enamel can occur.
question
When addressing toothbrushing procedures, the dental hygienist should advise the mother to
answer
dispense no more than a pea-size amount, or a slight smear, of fluoridated dentifrice. RATIONALE: The dental hygienist should advise the child's mother to dispense no more than a pea-size amount, or a slight smear, of fluoride dentifrice onto the toothbrush bristles. The mother should understand that her supervision while dispensing fluoride dentifrice is a critical function. Children ages 6 years and under can easily consume excess fluoride, increasing the risk of both fluorosis and toxicity. Covering the entire head of the toothbrush with fluoride dentifrice results in an excess quantity that could be ingested, increasing the risk of both fluorosis and toxicity. A fluoridated dentifrice is recommended for a 5-year-old child. A half-inch strip of fluoridated dentifrice is too much, increasing the risk that it will be ingested and increasing the likelihood of both fluorosis and toxicity.
question
Which of the following dental indices is an effective measure of dental caries activity in this young child?
answer
deft (decayed, exfoliated, filled teeth index) RATIONALE: The deft index, a variation of the DMFT, is used to measure observable caries experience in primary teeth. Note that deft measures the number of teeth with visible decay, a need for extraction, and restorative fillings. (The "e" stands for the need for extraction.) Missing teeth are not considered in the deft index. The gingival index (GI) is used to assess the severity of gingivitis on the basis of color, consistency, and bleeding on probing; GI is not a measure of caries activity. The plaque index (PI) is an assessment of the thickness of plaque in the gingival area; this index is not a measure of dental caries. The Oral Hygiene Index-Simplified (OHI-S) is used to assess oral cleanliness by estimating the extent of tooth surface covered with debris and/or calculus; the OHI-S index is not used to measure dental caries.
question
The child's palatal vault is unusually deep, and the maxillary anterior teeth protrude slightly. What is the MOST likely cause of these features?
answer
Thumb-sucking RATIONALE: The continued pressure of thumb-sucking against the palate and maxillary teeth during early childhood can cause anterior open bite and overjet, labial flare of the maxillary anterior teeth, and a high palatal vault. Mouth breathing can cause gingival enlargement but not a deep palate and protruded incisors. Tongue thrusting contributes to protrusion of the anterior teeth, not to a deep palatal vault. Bruxism affects tooth structure loss (attrition), not the soft tissues.
question
This child lives in a rural area in a northeastern state. His family's drinking water supply is well water that does not contain the appropriate amount of fluoride. What is the recommended fluoride level for a person living in this climate?
answer
0.7 ppm F RATIONALE: Earlier recommendations were based on a range of 0.7 to 1.2 mg fluoride/L water, depending on the climate in a geographic area. However, in 2011 both the U.S. Department of Health and Human Services (HHS) and the U.S. Environmental Protection Agency (EPA) adjusted their recommendations to 0.7 mg fluoride/L water (also expressed as parts per million [ppm]). Note that this new level applies to all geographic areas, regardless of climate or amount of water residents might consume. This revision was designed to balance dental caries prevention with the risk of fluorosis. The remaining selections deliver either too little or too much fluoride per liter of water.
question
Because this child is only 2½ years old, which of the following is an effective strategy to manage his care?
answer
Using age-appropriate terms RATIONALE: To successfully manage treatment for a young child, it is essential to establish a common ground for communication. Using words and phrases that the child can understand will improve the child's trust in the clinician. Restraints should be reserved as a last resort for client management. Quick and efficient care is only successful if you first obtain the client's cooperation. Effective communication skills are necessary for this to occur. Asking the parent to remain in the reception area may or may not be effective.
question
The lesion at the apex of the maxillary primary left central incisor is painful on palpation/depression of the tooth in the socket, and thick yellowish exudate can be expressed from a fistula deep within the vestibule. Which of the following is the MOST likely lesion?
answer
Periapical abscess RATIONALE: The presence of both purulence and pain indicate that this lesion is a periapical abscess. This acute, localized bacterial infection causes severe pain as a result of inflammation. A periapical abscess may develop directly from an inflammatory response within the pulp, or it may develop in an area of previously existing chronic inflammation. A periodontal abscess generally develops from accumulation of exudate or impaction of food within a prominent periodontal defect. No periodontal defect is present at this site. A periapical cyst occurs in association with the root of a nonvital tooth. Although this tooth is almost certainly nonvital, most periapical cysts are asymptomatic and are discovered on radiographic examination. This case has symptoms. A periapical granuloma is a localized mass of chronic granulation tissue that forms at the opening of a pulp canal, generally at the apex of a nonvital tooth root. This is a chronic process, and most cases are completely asymptomatic, which does not describe this case.
question
While you are talking to this child's mother, she tells you that she had problems taking the baby bottle away from her child. All of the following liquids could have contributed to the child's ECC EXCEPT one. Which one is the EXCEPTION? A. Diluted milk B. Nonfluoridated water C. Organic baby formula D. Unsweetened apple juice
answer
Nonfluoridated water RATIONALE: Nonfluoridated water is preferable to any juice, milk, or formula product, all of which contain sugar. Consumption of sugary or acidic liquids at bedtime promotes growth of Streptococcus mutans. This microorganism is known to cause dental caries when allowed to remain in a child's mouth. Prolonged feeding from a bottle or sippy cup is contraindicated for a child of this age. Note that sugary liquids, whether natural or refined, contain fermentable carbohydrates. Milk, whether diluted or full strength, contains lactic acid and contributes to dental caries during prolonged feeding (remaining in the mouth for prolonged periods of time, such as bottle feeding). Organic baby formula contains fermentable carbohydrates and can contribute to dental caries during prolonged feeding. Unsweetened apple juice contains acid that contributes to dental caries when allowed to remain in the child's mouth.
question
All of the following are important considerations in the planning of care for this client EXCEPT one. Which one is the EXCEPTION? A. Frequent breaks B. Frequent opportunity to relax her jaw C. Frequent sips of water D. Frequent use of compressed air
answer
Frequent use of compressed air RATIONALE: Frequent use of compressed air on this client is not necessary and may cause discomfort, because her tissues are already dry. Frequent breaks will make the client more comfortable, because these will provide time for her temporomandibular joint (TMJ) to relax. Relaxing her jaw frequently will relieve tension on her TMJ. Frequent sips of water (or rinsing with a mouthwash designed for persons with xerostomia) will help alleviate the dryness and discomfort associated with xerostomia.
question
In the panograph, the well-defined radiolucent area on the mesial surface of tooth #7 is
answer
a composite restoration. RATIONALE: Composite restorations appear radiographically as well-defined areas of radiolucency. Amalgam restorations, which appear radiopaque, are not usually placed in anterior teeth. Also, the photos of this client do not reveal amalgam restorations on the anterior teeth. The photos of this client do not reveal gold restorations, which appear radiopaque, on the anterior teeth. Carious lesions appear radiolucent; because they are a disease process they do not display the well-defined borders that are characteristic of composite restorations.
question
The gingival margin around teeth #22 through #28 can BEST be described as
answer
festooned. RATIONALE: Festooned gingiva is an enlargement of the marginal gingiva with the formation of a lifesaver-like gingival prominence. Displayed well in these photographs, the total gingiva is often very narrow, with the presence of apparent recession. Cratering of the gingival margin refers to interdental necrosis with ulceration of the papillae, which produces a craterlike defect. Clefting creates an area of localized recession that may extend several millimeters toward, or even through, the mucogingival junction. Note that clefting is typically V-shaped, apostrophe-shaped, or slit-like. Marginal gingiva that has receded from apical migration of the junctional epithelium results in exposed root surface.
question
Complications from Sjögren's syndrome include all of the following EXCEPT one. Which one is the EXCEPTION? A. Fissured tongue B. Intraoral yeast infections C. Burning mucosa D. Increase in dental caries E. Enlargement of the salivary glands
answer
Enlargement of the salivary glands RATIONALE: Complications of Sjögren's syndrome, or SS, do not include enlargement of the tongue. Oral manifestations of this multisystem inflammatory disorder of the exocrine glands include fissured tongue, intraoral yeast infections, and burning mucosa. It is xerostomia, a classic manifestation of SS, that causes an increase in the likelihood of dental caries.
question
Xerostomia is one of the main symptoms of Sjögren's syndrome. Xerostomia is a common side effect of Librax and Premarin, but neither Prozac nor Zantac is known to cause dry mouth.
answer
The first statement is true, the second is false. RATIONALE: Xerostomia, or dry mouth, is one of the main symptoms of Sjögren's syndrome (SS). Almost every patient with SS experiences some degree of dry mouth. All of the drugs taken by this patient are associated with some degree of xerostomia.
question
Which of the following instruments should be used to scale the distal surface of tooth #25?
answer
Gracey 1/2 curet RATIONALE: The Gracey 1/2 is the BEST selection for two reasons. The area-specific design of the Gracey 1/2 is specifically designed to adapt to the contours of anterior teeth, and its longer shank permits access to a 5-mm pocket. By contrast, the Columbia 13/14 curet is universal (not area-specific) and designed to scale all aspects of all teeth. However, compared with the Gracey 1/2, the shorter shank of the Columbia 13/14 will not adequately access a 5-mm pocket. Both the Gracey 11/12 and Gracey 13/14 are area-specific curets designed for adaptation to posterior teeth. Although the Gracey 11/12 adapts to the facial, lingual, and mesial surfaces of posterior teeth, the Gracey 13/14 adapts to distal surfaces of posterior teeth.
question
Acidulated phosphate fluoride gel is the optimal fluoride for this patient. Because of her xerostomia, this patient is not a candidate for fluoride therapy.
answer
Both statements are false. RATIONALE: Neutral sodium fluoride varnish is the choice because of its efficacy; acidulated phosphate fluoride can etch porcelain surfaces. Because of her xerostomia, she is at extremely high risk of caries. This client should definitely undergo fluoride therapy, both because of her history of extensive dental work and her xerostomia.
question
Which of the following is an additional hazard to the clinician performing motor-driven rubber cup or air-abrasive extrinsic stain-removal procedures?
answer
Production of contaminated aerosols during extrinsic stain removal RATIONALE: Use either rotary instruments or air abrasive extrinsic stain-removal techniques produces contaminated aerosols that permeate the operatory and pose a hazard to the clinician. All of the remaining selections are problematic and should be minimized, but they pose hazards to the client rather than the clinician. Heat generated during extrinsic stain removal, as well as removal of surface-layer fluoride, can be minimized by using less pressure and speed (RPM) in the operation of stain-removal devices. Use of highly abrasive polishing agents is contraindicated in most cases because these agents can damage both tooth and gingival tissues.
question
Which of the following should this client be MOST encouraged to use as a regular part of her home care regimen?
answer
Topical fluoride gel RATIONALE: This client should be encouraged to use a topical fluoride gel daily, because xerostomia puts her at extremely high risk for dental caries. She has extensive restorative dentistry, which further substantiates this recommendation. There is no reason for this client to use chlorhexidine mouthrinse. Furthermore, chlorhexidine will stain her tooth-colored crowns. Fluoridated toothpaste is beneficial, but topical fluoride gel will provide more protection. There is no therapeutic value to flavored mouthrinses unless they carry the ADA Seal of Acceptance.
question
Taking into account all aspects of this client's history with her current oral conditions and periodontal maintenance guidelines, what is the BEST interval for her continued care?
answer
3 months RATIONALE: Because of the client's history of Sjögren syndrome, xerostomia, and extensive restorative dental treatment and the extremely high risk for dental caries, a 3-month continued care interval is the most appropriate interval. Two months is probably too frequent, in view of the client's good oral hygiene habits. Four months is too long for this client. Six months is definitely too long for this client.
question
While you are using a toothbrush to polish this client's teeth, she asks why you are not using the "electric polisher" as her last dental hygienist did. Which of the following statements is NOT true of the effects of polishing?
answer
Abrades and removes only cementum and dentin RATIONALE: Polishing abrades and removes structure from all tooth surfaces, including enamel. As much as 4 microns may be removed in 30 seconds with pumice paste. Lost tooth substance is cumulative and is not replaced. Loss of structure may become significant with repeated polishing. It is true that polishing produces bacteremia. One important consideration is that polishing may deliver the polishing agent into ulcerated gingiva. All patients should be aware that the heat produced during polishing can cause both discomfort and damage.
question
In the planning of educational services for this client, the practice that should receive the highest priority is
answer
oral cancer self-examination. RATIONALE: Although each of these practices will benefit the client, her past history of melanoma should receive the highest priority. Instructions in oral cancer self-examination techniques may prove lifesaving. Home fluoride, though very important, is not the highest priority for this client. Although flossing techniques should be reviewed, the client already reports flossing two or three times a day. Saliva substitutes will benefit this client but are not as important as oral cancer self-exam.
question
The scattered red projections that are prominent near the tip of this client's tongue are called
answer
fungiform papillae. RATIONALE: Fungiform papillae are mushroom-shaped red projections that are scattered among the filiform papillae on the dorsal surface of the tongue. Fungiform papillae contain taste buds responsible for sensing sweet, sour, and salty stimuli. Circumvallate papillae are located in a V formation on the posterior section of the dorsal surface of the tongue and contain taste buds for sensing bitter stimuli. The filiform papillae are whitish hair-like projections that cover the dorsal surface of the tongue. Foliate papillae are projections found on the posterior lateral borders of the tongue that contain the taste buds responsible for sensing sour and acidic stimuli.
question
The dental implants placed in this client's mouth are
answer
endosteal (endosseous) implants. RATIONALE: These are endosteal screw-type dental implants. Note that endosteal implants are placed within the bone and are the most commonly used type. They are typically screw-like, cylindrical, or plate-like in shape. Subperiosteal implants are made of a custom-fabricated metal framework that is placed over the bone of the mandible or maxilla. Subperiosteal implants may be preferable for patients with shallow jawbones. Transosteal implants consist of a metal plate that is placed in the lower border of the mandible and has pins extending toward the occlusal surface. Micromini implants are a variation of the endosteal implant design but are smaller, narrower, and more screw-like in appearance. Micromini implants are more temporary in nature and designed to be easily removed.
question
Should she need to take an antibiotic, this client should be advised to discontinue her oral contraceptives, because oral contraceptives may decrease the effectiveness of an antibiotic.
answer
NEITHER the statement NOR the reason is correct. RATIONALE: Although some antibiotics decrease the effectiveness of oral contraceptives, the reverse is not true.
question
The radiolucent area to the distal of tooth #31 represents a
answer
recent tooth extraction site. RATIONALE: The radiolucent area is the result of a recent extraction. The bone has not yet regenerated, which further explains why the shape of the radiolucency is that of a recently extracted tooth. The radiolucency is not a periapical abscess, because a tooth is not present in this site. Nothing in the history, evaluation, radiographic findings, or diagnosis indicates the presence of a carcinoma. There is no indication that this radiolucency represents a radicular cyst; it bears the shape of a recently extracted tooth.
question
The radiolucent area at the tip of tooth #29 is an exostosis. The mental foramen is visible in the area of tooth #2 just apical to the retained root tips.
answer
The first statement is true, the second is false. RATIONALE: The radiolucent area at the tip of tooth number 29 is the mental foramen. By contrast, an exostosis appears radiopaque. Note that the mental foramen is located between the mandibular premolars and appears radiographically as a radiolucent area near the apices. The mental foramen is also visible at the apex of tooth number 20. Although tooth number 2 has retained root tips, the mental foramen is not located in this area.
question
The recession along the cervical one third of teeth #21 and 28 is MOST likely due to which of the following?
answer
High lateral frenal attachment RATIONALE: The lateral frenum is located in the buccal vestibule and connects the lips to the alveolar ridge. If these attachments affix too high on the alveolar gingiva, they can cause excessive tension. Gingival recession on the facial surfaces of the teeth in that area is the predictable result. Improper toothbrushing usually affects more than one tooth in the arch. Calculus is present but is not the primary cause of gingival recession. Improper flossing will not cause labial recession. Improper flossing typically does not cause gingival recession, but it can cause traumatic gingival damage, such as Stillman's clefting.
question
Which of the following aids would be MOST helpful to this client in removing supragingival plaque biofilm?
answer
Power toothbrush RATIONALE: This client has arthritis in her hands and most likely has problems using a manual toothbrush because of its small handle. Most power toothbrushes have large handles, which make them easier to use for people with limited manual dexterity. Many clients with arthritis in their hands have difficulty using dental floss; again, a power toothbrush greatly enhances their oral hygiene performance. Although interdental stimulators are beneficial for many, for this client they will be too small and difficult to use.
question
Which of the following probes is preferred to detect furcation involvement?
answer
Nabers probe RATIONALE: The Nabers probe, curved and calibrated, is preferred for detecting, measuring, and classifying periodontal furcation involvement in multi-rooted teeth. The Marquis probe, the first color-coded probe, is available in both straight and curved designs but is not specifically designed for furcations. The Williams probe is a straight probe and therefore not beneficial in investigating furcations. The Michigan-O probe is also a straight probe and is therefore not the preferred probe for furcations.
question
If the physician determines that this client should receive prophylactic antibiotic premedication, what is the proper dosage?
answer
2 g amoxicillin orally 30 minutes to 1 hour before the procedure RATIONALE: For patients and procedures requiring prophylactic antibiotic administration before dental hygiene and dental procedures, the dosage recommended by the American Heart Association is 2 g amoxicillin administered 30 minutes to 1 hour before the procedure. In people who are unable to take penicillin, the dentist or physician will select from cephalexin, clindamycin, azithromycin, and clarithromycin. The dosage varies according to the medication. Five hundred milligrams of amoxicillin 30 minutes to 1 hour before the procedure, followed by 250 mg every 6 hours for eight doses, is incorrect. Two grams of clindamycin 30 min to 1 hour before the procedure, followed by 500 mg every 6 hours for eight doses, is incorrect. Three g of amoxicillin 30 minutes to 1 hour before the procedure, followed by 1.5 g 6 hours after the initial dose, is incorrect.
question
For which of the following reasons is a medical consultation with the client's physician recommended for persons with systemic lupus erythematosus (SLE)?
answer
Heart valve pathology RATIONALE: Multiple cardiac complications, including valvular damage and immunosuppression, are common in persons with systemic lupus erythematosus (SLE). Consultation with the client's physician before treatment is required to establish the need for prophylactic antibiotic premedication. A medical consultation is not required for arthritis, but the dental hygienist should be aware that Plaquenil (hydroxychloroquine) is administered for rheumatoid arthritis, which is typically more serious than osteoarthritis. Rheumatoid arthritis is a systemic inflammatory disease with autoimmune links. The health history should reflect the type of arthritis. Raynaud's syndrome is a condition exhibited in some individuals who have systemic lupus. It is characterized by abnormal spasms of the blood vessels (vasospasm) of the extremities, especially in response to cold. Raynaud's syndrome is not life threatening and does not require consultation with the physician. Periodontal inflammation by itself does not require a medical consultation.
question
The radiopaque areas on both the mesial and distal surfaces of #4 as well as the distal surface of #5 are interproximal calculus. The radiopaque areas on the mesial and distal of #13 represent dental caries.
answer
The first statement is true; the second is false. RATIONALE: In radiography, an areas of radiopacity will obstruct (attenuate) the passage of x-rays and thus appears light. The radiopaque areas visible on the mesial and distal surfaces of tooth #4 and the distal surface of tooth #5 represent interproximal calculus that is dense enough to interfere with the passage of x-radiation. Thus the areas appear whitish. To be visible on radiographs, calculus must be fairly significant in mass. Another distinguishing feature is that the borders of the mass often appear irregular. The radiopacities visible on the mesial and distal surfaces of tooth #13 also represent interproximal calculus. By contrast, x-rays readily pass through a carious lesion, giving it a radiolucent (dark) appearance.
question
The treatment plan for this patient should include ultrasonic scaling because heavy subgingival calculus is present in several areas.
answer
The statement is NOT correct, but the reason is correct. RATIONALE: Although heavy subgingival calculus is present, this patient is not a candidate for ultrasonic scaling because of the extensive demineralization of tooth structure.
question
An in-office application of sodium fluoride solution (2%) would be a better selection for this client than an application of acidulated phosphate fluoride (1.23%) because the APF formulation may be astringent and cause burning of the oral tissues.
answer
NEITHER the statement NOR the reason is correct. RATIONALE: Because this client has numerous areas of decalcification, the better selection for this client is acidulated phosphate fluoride (APF) therapy. Research shows that APF is more effective than sodium fluoride for decay prevention. Note however, that sodium fluoride gel may be used if the client has dentinal hypersensitivity. An astringent quality and burning of the tissues is associated with stannous fluoride preparations.
question
Complete scaling and root planing in this client will be complicated because of
answer
numerous areas of dental caries. RATIONALE: Although all of the factors mentioned will complicate scaling and root planing, the numerous areas of rampant decay and demineralization will pose the greatest challenge. Because many of these areas are located at the gingival margin, complete instrumentation in these areas will pose difficulty, even with the administration of a local anesthetic agent. Inadequate oral hygiene will not complicate scaling and root planing procedures. Although root sensitivity could be a problem, it is not listed in the client's history. The dental hygienist can elect to use pain control measures should root sensitivity pose a problem. Excessive bleeding could be a problem, but the numerous areas of dental caries will more problematic.
question
Class II mobility, noted on teeth #23 and #24, is defined as
answer
moderate mobility greater than 1 mm of horizontal displacement in a facial-lingual direction RATIONALE: Class II mobility is defined as moderate mobility, with more than 1 mm of horizontal displacement in a facial-lingual direction. Barely discernible movement, often referred to as normal or physiologic mobility, is expected in health. Class I mobility is defined as slight mobility with up to 1 mm of horizontal displacement in a facial-lingual direction. Class III mobility is defined as severe mobility, with vertical as well as horizontal movement greater than 1 mm possible in all directions. In Class III mobility the tooth may be depressible in the socket.
question
The cause of this client's xerostomia is MOST likely
answer
medications. RATIONALE: Xerostomia is a common side effect of many medications, including those listed in his medical history: Plaquenil, prednisone, and Feldene. All medications should be investigated before treatment to determine whether any factors would affect dental/dental hygiene care and oral disease risk. Although lupus erythematosus does not cause xerostomia, it contributes, because medications used to treat it can cause dry mouth. High blood pressure does not cause xerostomia, but it contributes, because the medications used to treat it can cause dry mouth. The client's age is not a contributing factor to xerostomia.
question
Four weeks after initial scaling and root planing, a reevaluation is performed. Adherence to recommended home care techniques is evaluated, and new periodontal measurements are taken. Many areas exhibit both a reduction in bleeding sites and probing depths. Which of the following BEST explains the reason for this change?
answer
Decrease in the disease process RATIONALE: Because bleeding on probing is a reliable sign of disease, the absence of bleeding on probing, along with a decrease in pocket depth, would serve as proof of an overall diminution of the periodontal disease process. Normal tissue shrinkage is most likely a result of the decreased disease process. Improved home care technique is highly likely to be a factor in the decrease of the periodontal disease process. An increase in anaerobic bacteria would worsen the disease process.
question
For teeth #14 through #19, the severity of this client's periodontal status is classified as
answer
advanced chronic periodontitis. RATIONALE: The correct periodontal classification for teeth #14 through #19 is advanced periodontitis, as evidenced by major loss of alveolar bone support accompanied by increased tooth mobility, furcation involvement in multiple-rooted teeth, and average probing depths of 6 mm or more. Early chronic periodontitis is characterized by slight bone loss, normal gingival contour, and an average attachment loss of 1 to 2 mm. Moderate chronic periodontitis is characterized by noticeable loss of bone support, with the possibility of tooth mobility and an average 3 to 4 mm of attachment loss.
question
A fluoridated mouthrinse without alcohol is recommended for patients with xerostomia because alcohol-free fluoridated rinses prevent further drying of the tissues and oral fungal infections.
answer
The statement is correct, but the reason is NOT. RATIONALE: To prevent dental caries and further drying of the tissues, patients with xerostomia can benefit from a fluoridated mouthrinse. Newer research has shown that mouthrinses containing alcohol do not dry the oral tissues, but controversy persists. Fluoridated mouthrinses, with or without alcohol, do not prevent oral fungal infections.
question
A panoramic radiograph is advantageous in orthodontic evaluation because this technology provides early detection of small carious lesions, allowing them to be restored before the placement of orthodontic appliances.
answer
NEITHER the statement NOR the reason is correct. RATIONALE: Panoramic radiology is advantageous for orthodontic evaluation because it provides excellent visualization of deciduous, permanent, and supernumerary teeth, as well as potential pathologies. Note that panoramic radiology does not enable detection of small carious lesions or moderate calculus. Note also that the angulation of the x-ray beam during panoramic radiology prevents early detection of vertical bone loss.
question
The white area on the facial of tooth #3 is MOST likely
answer
decalcification/demineralization. RATIONALE: The white area in question is most likely decalcification, which is also called demineralization. (Calcium is a mineral.) Areas of demineralization/decalcification appear dull and chalky; however, illumination reveals a loss of translucency in the enamel. Although calculus is a likely explanation, closer inspection reveals that the white area is a defect within the body of the tooth. Calculus would appear an external substance deposited on top of the surface of the tooth. Fluorosis is unlikely, because this area of whiteness is isolated. Fluorosis is a more generalized condition. Trauma would not cause this type of white area.
question
By which of the following factors is the white area on the facial of tooth #3 MOST likely caused?
answer
Poor brushing habits and presence of plaque biofilm RATIONALE: Decalcification occurs in areas where bacterial plaque biofilm collects and the client finds it difficult to clean. Chlorhexidine mouthrinse does not cause white areas; with extended use, it may cause brownish extrinsic stain. The gingival margin has not receded, nor does gingival recession alone cause white marks on teeth. Weak tooth structure can cause surface irregularities, but this area is more consistent with decalcification resulting from retention of bacterial plaque biofilm.
question
Radiographically the mandibular canal appears as a radiolucent band outlined by two radiopaque lines on the right and left sides of the mandible. The lingual foramen appears as a tiny radiopaque opening near the coronoid process.
answer
The first statement is true, the second is false. RATIONALE: The mandibular canal appears bilaterally as a radiolucent band outlined by two radiopaque lines that represent the surrounding walls of thin cortical bone. Located slightly apical to the roots of the teeth, the mandibular canal extends from the molars to the mental foramen. The lingual foramen appears as a tiny radiolucent hole or opening on the internal surface of the mandible at the midline. (The lingual foramen is surrounded by the genial tubercles, which appear radiopaque.) Note that the coronoid process is located above the angle of the mandible.
question
What type of occlusion is present in the anterior area as viewed looking directly onto the teeth of the client?
answer
Overbite RATIONALE: Overbite, the characteristic of the anterior maxillary teeth to overlap the anterior mandibular teeth in a vertical dimension, is the vertical distance by which the maxillary incisors overlap the mandibular incisors. An overbite is considered normal when the incisal edges of the maxillary teeth are within the incisal third of the mandibular teeth. A scissors-like function of the incisors is established by a normal overbite. However, in the anterior-oriented photo of this client, the incisal edges of the maxillary teeth are within the cervical third of the mandibular teeth. This overbite is classified as deep or severe. Additionally, the incisal edges of the mandibular teeth are in contact with the maxillary lingual gingival tissue, which further classifies this overbite as very deep. Overjet is a measure of the horizontal distance between the labioincisal surfaces of the mandibular incisors and the linguoincisal surfaces of the maxillary incisors. Crossbite occurs when the mandibular or maxillary teeth are lingual or facial to their normal position. Underjet, a measure of horizontal distance, occurs when the maxillary incisors are lingual to the mandibular incisors.
question
At a re-care appointment, you place a dental sealant on tooth #30. When the client rinses, the sealant comes out. Which of the following factors is the MOST likely cause of the failure of the sealant placement?
answer
Moisture contamination RATIONALE: The leading cause of sealant failure is contamination of the etched surface with saliva, water, or moisture before sealant application. Phosphoric acid, a leading etchant, typically has no expiration date. Rinsing a dental sealant after polymerization does not affect its retention. Omission of a post-fluoride application does not affect sealant retention. An important distinction: Fluoride application BFORE sealant placement is contraindicated because the mechanical bonding necessary for retention is disturbed.
question
Orthodontic appliances will be placed on this client's teeth in the next few weeks. Judging from his current photographs, which of the following devices would be MOST likely to be successfully implemented at the beginning of his orthodontic treatment?
answer
Orthodontic bi-level toothbrush RATIONALE: An orthodontic bi-level toothbrush is MOST likely to be successfully implemented by this client because it is designed to move into and around orthodontic appliances. Its bristle configuration, three rows wide with a shorter middle row, enhances adaptation between orthodontic appliances and the gingival margin. Although other oral hygiene adjuncts can be effectively implemented later, the orthodontic bi-level toothbrush is a good starting device for a 10-year-old client. Although interdental brushes can be very effective, be mindful that this client is 10 years old and they may be difficult for him to maneuver. Floss threaders, though highly effective for many clients, are not the best selection for this client so early in the orthodontic process. Dental water jets are indicated for clients with fixed orthodontic appliances, but they require personal training in proper technique. Research suggests that although oral irrigation improves gingival health compared with regular oral hygiene, it does not reduce plaque biofilm.
question
All of the following are benefits of a custom athletic mouthguard EXCEPT one. Which one is the EXCEPTION?
answer
Decreasing the risk of dental caries RATIONALE: Custom athletic mouthguards (also called custom mouth protectors) are an established mechanism for decreasing shock to the temporomandibular joint (TMJ) and mandibular condyle. It is important that the dental hygienist help clients understand that prevention of such injuries significantly decreases incidence of traumatic brain injury (concussion). To a large extent, custom mouth protectors prevent dislocation or fracture of maxillary anterior teeth and minimize lacerations of soft tissues. Despite these benefits, custom athletic mouthguards have no effect on caries risk.
question
Which of the following terms describes this client's occlusal classification on the right molar side?
answer
Crossbite RATIONALE: This client's occlusal classification on the right side is described as crossbite because the maxillary molar is lingual to its normal position. Underjet describes anterior occlusal patterns and occurs when the maxillary incisors are lingual to the mandibular incisors. Overbite is the vertical distance by which the maxillary incisors overlap the mandibular incisors. In overjet, the maxillary incisors are labial to the mandibular incisors.
question
After taking the client's alginate impressions, you put them in the lab, and it is several hours before you are able to pour them. Which of the following will MOST likely to happen to the impressions?
answer
Syneresis RATIONALE: Syneresis, the loss of water by evaporation due to exposure to air, results in shrinkage of alginate. For this reason, alginate impressions should be immediately poured into a gypsum product. Solation, the transformation of a gel into a sol (solution), occurs with hydrocolloid impression material. Gelation, transformation from a sol (solution) to a gel, occurs while the alginate impression is in the mouth. Imbibition causes an alginate impression to absorb additional water and expand. As a means of preventing imbibition, alginate impressions should not be stored in water.
Get an explanation on any task
Get unstuck with the help of our AI assistant in seconds
New